SlideShare a Scribd company logo
1-1 
Chapter 01 
The Role of the Public Accountant in the American Economy 
True / False Questions 
1. 
Independent audits of today place more emphasis on sampling than did the audits of the 19th 
century. 
True False 
2. 
The American Institute of Certified Public Accountants issues CPA certificates and permits CPAs 
to practice. 
True False 
3. 
A company is either audited by the GAO or internal auditors, but not both. 
True False 
4. 
The SEC does not pass on the merits of the securities that are registered with the agency. 
True False 
5. 
The American Institute of Certified Public Accountants has the primary authority to establish 
accounting standards. 
True False 
6. 
An annual peer review is a requirement of the AICPA. 
True False 
7. 
Many small companies elect to have their financial statements reviewed by a CPA firm, rather 
than incur the cost of an audit. 
True False 
8. 
Staff assistants in CPA firms generally are responsible for planning and coordinating audit 
engagements. 
True False
9. 
The Sarbanes-Oxley Act requires that auditors of certain publicly traded companies in the United 
States perform an integrated audit that includes providing assurance on both the financial 
statements and on compliance with laws and regulations. 
True False 
10. 
Auditing is frequently only a small part of the practice of local CPA firms. 
1-2 
True False 
Multiple Choice Questions 
11. 
A summary of findings rather than assurance is most likely to be included in a(n): 
A. 
Agreed-upon procedures report. 
B. 
Compilation report. 
C. 
Examination report. 
D. 
Review report. 
12. 
The Statements on Auditing Standards have been issued by the: 
A. 
Auditing Standards Board. 
B. 
Financial Accounting Standards Board. 
C. 
Securities and Exchange Commission. 
D. 
Federal Bureau of Investigation. 
13. 
The risk associated with a company's survival and profitability is referred to as: 
A. 
Business Risk. 
B. 
Information Risk. 
C. 
Detection Risk. 
D. 
Control Risk. 
14. 
Historically, which of the following has the AICPA been most concerned with providing? 
A. 
Professional standards for CPAs. 
B. 
Professional guidance for regulating financial markets. 
C. 
Standards guiding the conduct of internal auditors. 
D. 
Staff support to Congress.
15. 
The organization charged with protecting investors and the public by requiring full disclosure of 
financial information by companies offering securities to the public is the: 
A. 
Auditing Standards Board. 
B. 
Financial Accounting Standards Board. 
C. 
Government Accounting Standards Boards. 
D. 
Securities and Exchange Commission. 
16. 
An engagement in which a CPA firm arranges for a critical review of its practices by another CPA 
1-3 
firm is referred to as a(n): 
A. 
Peer Review Engagement. 
B. 
Quality Control Engagement. 
C. 
Quality Assurance Engagement. 
D. 
Attestation Engagement. 
17. 
The serially-numbered pronouncements issued by the Auditing Standards Board over a period of 
years are known as: 
A. 
Auditing Statements of Position (ASPs). 
B. 
Accounting Series Releases (ASRs). 
C. 
Statements on Auditing Standards (SASs). 
D. 
Statements on Auditing Principles (SAPs). 
18. 
The Government Accountability Office (GAO): 
A. 
Is primarily concerned with rapid processing of all accounts payable incurred by the federal 
government. 
B. 
Conducts operational audits and reports the results to Congress. 
C. 
Is a multinational organization of professional accountants. 
D. 
Is primarily concerned with budgets and forecasts approved by the SEC. 
19. 
The risk that information is misstated is referred to as: 
A. 
Information risk. 
B. 
Inherent risk. 
C. 
Relative risk. 
D. 
Business risk.
20. 
The risk that a company will not be able to meet its obligations when they become due is an 
1-4 
aspect of: 
A. 
Information risk. 
B. 
Inherent risk. 
C. 
Relative risk. 
D. 
Business risk. 
21. 
Which of the following attributes most clearly differentiates a CPA who audits management's 
financial statements as contrasted to management? 
A. 
Integrity. 
B. 
Competence. 
C. 
Independence. 
D. 
Keeping informed on current professional developments. 
22. 
The attest function: 
A. 
Is an essential part of every engagement by the CPA, whether performing auditing, tax work, or 
other services. 
B. 
Includes the preparation of a report of the CPA's findings. 
C. 
Requires a consideration of internal control. 
D. 
Requires a complete review of all transactions during the period under examination. 
23. 
Attestation risk is limited to a low level in which of the following engagement(s)? 
A. 
Both examinations and reviews. 
B. 
Examinations, but not reviews. 
C. 
Reviews, but not examinations. 
D. 
Neither examinations nor reviews. 
24. 
When compared to an audit performed prior to 1900, an audit today: 
A. 
Is more likely to include tests of compliance with laws and regulations. 
B. 
Is less likely to include consideration of the effectiveness of internal control. 
C. 
Has bank loan officers as the primary financial statement user group. 
D. 
Includes a more detailed examination of all individual transactions.
25. 
Which of the following are issued by the Securities and Exchange Commission? 
1-5 
A. 
Accounting Research Studies. 
B. 
Accounting Trends and Techniques. 
C. 
Industry Audit Guides. 
D. 
Financial Reporting Releases. 
26. 
Which of the following is not correct relating to the Sarbanes-Oxley Act? 
A. 
It toughens penalties for corporate fraud. 
B. 
It restricts the types of consulting CPAs may perform for audit clients. 
C. 
It created the Public Company Accounting Oversight Board (PCAOB) as a replacement for the 
Financial Accounting Standards Board. 
D. 
It eliminates a significant portion of the accounting profession's system of self-regulation. 
27. 
An operational audit differs in many ways from an audit of financial statements. Which of the 
following is the best example of one of these differences? 
A. 
The usual audit of financial statements covers the four basic statements, whereas the 
operational audit is usually limited to either the balance sheet or the income statement. 
B. 
The boundaries of an operational audit are often drawn from an organization chart and are not 
limited to a single accounting period. 
C. 
Operational audits do not ordinarily result in the preparation of a report. 
D. 
The operational audit deals with pre-tax income. 
28. 
The review of a company's financial statements by a CPA firm: 
A. 
Is substantially less in scope of procedures than an audit. 
B. 
Requires detailed analysis of the major accounts. 
C. 
Is of similar scope as an audit and adds similar credibility to the statements. 
D. 
Culminates in issuance of a report expressing the CPA's opinion as to the fairness of the 
statements. 
29. 
Which statement is correct with respect to continuing professional education (CPE) requirements 
of members of the AICPA? 
A. 
Only members employed by the AICPA are required to take such courses. 
B. 
Only members in public practice are required to take such courses. 
C. 
Members, regardless of whether they are in public practice, are required to meet such 
requirements. 
D. 
There is no requirement for members to participate in CPE.
30. 
The FDIC Improvement Act requires that management of large financial institutions engage 
auditors to attest to assertions by management about the effectiveness of the institution's internal 
controls over: 
1-6 
A. 
Compliance with laws and regulations. 
B. 
Financial reporting. 
C. 
Effectiveness of operations. 
D. 
Efficiency of operations. 
31. 
Passage of the Sarbanes-Oxley Act led to the establishment of the: 
A. 
Auditing Standards Board. 
B. 
Accounting Enforcement Releases Board. 
C. 
Public Company Accounting Oversight Board. 
D. 
Securities and Exchange Commission. 
32. 
Which of the following professionals has primary responsibility for the performance of an audit? 
A. 
The managing partner of the firm. 
B. 
The senior assigned to the engagement. 
C. 
The manager assigned to the engagement. 
D. 
The partner in charge of the engagement. 
33. 
Which of the following types of services is generally provided only by CPA firms? 
A. 
Tax audits. 
B. 
Financial statement audits. 
C. 
Compliance audits. 
D. 
Operational audits. 
34. 
The right to practice as a CPA is given by which of the following organizations? 
A. 
State Boards of Accountancy. 
B. 
The AICPA. 
C. 
The SEC. 
D. 
The General Accounting Office.
35. 
Which of the following terms best describes the audit of a taxpayer's tax return by an IRS auditor? 
1-7 
A. 
Operational audit. 
B. 
Internal audit. 
C. 
Compliance audit. 
D. 
Government audit. 
36. 
Inquiries and analytical procedures ordinarily form the basis for which type of engagement? 
A. 
Agreed-upon procedures. 
B. 
Audit. 
C. 
Examination. 
D. 
Review. 
37. 
Which of the following best describes the reason why independent auditors report on financial 
statements? 
A. 
A management fraud may exist and it is more likely to be detected by independent auditors. 
B. 
Different interests may exist between the company preparing the statements and the persons 
using the statements. 
C. 
A misstatement of account balances may exist and is generally corrected as the result of the 
independent auditors' work. 
D. 
Poorly designed internal control may be in existence. 
38. 
Governmental auditing often extends beyond examinations leading to the expression of opinion on 
the fairness of financial presentation and includes audits of efficiency, economy, effectiveness, 
and also: 
A. 
Accuracy. 
B. 
Evaluation. 
C. 
Compliance. 
D. 
Internal control. 
39. 
Operational auditing is primarily oriented toward: 
A. 
Future improvements to accomplish the goals of management. 
B. 
The accuracy of data reflected in management's financial records. 
C. 
The verification that a company's financial statements are fairly presented. 
D. 
Past protection provided by existing internal control.
40. 
A typical objective of an operational audit is for the auditor to: 
A. 
Determine whether the financial statements fairly present the entity's operations. 
B. 
Evaluate the feasibility of attaining the entity's operational objectives. 
C. 
Make recommendations for improving performance. 
D. 
Report on the entity's relative success in attaining profit maximization. 
41. 
An integrated audit performed under the Sarbanes-Oxley Act requires that auditors report on: 
1-8 
A. 
Option A 
B. 
Option B 
C. 
Option C 
D. 
Option D 
Matching Questions
42. 
Accountants are regulated by a variety of organizations. Match the following statements with the 
most directly related organizations. Organizations may be used once or not at all. 
1-9 
1. Formed to improve standards of 
financial accounting for state and local 
government entities 
State Boards of 
Accountancy. 
____ 
2. Issue CPA certificates 
Government Accounting 
Standards Board. 
____ 
3. Develop accounting standards for 
public and nonpublic companies 
American Institute of 
Certified Public 
Accountants. 
____ 
4. Develop accounting standards for 
the U.S. Government 
Financial Accounting 
Standards Board. 
____ 
5. Issue auditing standards for public 
companies 
Federal Accounting 
Standards Advisory Board. 
____ 
6. Prepares the CPA exam 
Public Company 
Accounting Oversight 
Board. 
____ 
Essay Questions 
43. 
The Sarbanes-Oxley Act of 2002 made significant reforms for public companies and their auditors. 
a. Describe the events that led up to the passage of the Act. 
b. Describe the major changes made by the Act.
44. 
Many people confuse the responsibilities of the independent auditors and the client's management 
with respect to audited financial statements. 
a. Describe management's responsibility regarding audited financial statements. 
b. Describe the independent auditors' responsibility regarding audited financial statements. 
c. Evaluate the following statement: "If the auditors disagree with management regarding an 
accounting principle used in the financial statements the auditors should express their views in the 
notes to the financial statements." 
45. 
An investor is considering investing in one of two companies. The companies have very similar 
reported financial position and results of operations. However, only one of the companies has its 
financial statements audited. 
a. Describe what creates the demand for an audit in this situation. Include a discussion of how 
audited financial statements facilitate this investment transaction, and the effect of the audit on 
business risk and information risk. 
b. Identify the potential consequences to the company of not having its financial statements 
audited. 
1-10
Chapter 01 The Role of the Public Accountant in the American Economy 
Answer Key 
1-11 
True / False Questions 
1. 
Independent audits of today place more emphasis on sampling than did the audits of the 19th 
century. 
TRUE 
AACSB: Analytic 
AICPA BB: Industry 
AICPA FN: Decision Making 
Blooms: Remember 
Difficulty: 2 Medium 
Learning Objective: 01-04 Explain why audits are demanded by society. 
Topic: Financial Statement Audits 
2. 
The American Institute of Certified Public Accountants issues CPA certificates and permits 
CPAs to practice. 
FALSE 
AACSB: Analytic 
AICPA BB: Industry 
AICPA FN: Decision Making 
Blooms: Remember 
Difficulty: 2 Medium 
Learning Objective: 01-07 Explain the regulatory process for auditors of public companies and auditors of nonpublic companies. 
Topic: Public Accounting Profession 
3. 
A company is either audited by the GAO or internal auditors, but not both. 
FALSE 
AACSB: Analytic 
AICPA BB: Industry 
AICPA FN: Decision Making 
Blooms: Remember 
Difficulty: 1 Easy 
Learning Objective: 01-06 Contrast the various types of audits and types of auditors. 
Topic: Financial Statement Audits 
4. 
The SEC does not pass on the merits of the securities that are registered with the agency. 
TRUE 
AACSB: Analytic
1-12 
AICPA BB: Industry 
AICPA FN: Decision Making 
Blooms: Remember 
Difficulty: 2 Medium 
Learning Objective: 01-07 Explain the regulatory process for auditors of public companies and auditors of nonpublic companies. 
Topic: Public Accounting Profession 
5. 
The American Institute of Certified Public Accountants has the primary authority to establish 
accounting standards. 
FALSE 
AACSB: Analytic 
AICPA BB: Industry 
AICPA FN: Decision Making 
Blooms: Remember 
Difficulty: 1 Easy 
Learning Objective: 01-07 Explain the regulatory process for auditors of public companies and auditors of nonpublic companies. 
Topic: Public Accounting Profession 
6. 
An annual peer review is a requirement of the AICPA. 
FALSE 
AACSB: Analytic 
AICPA BB: Industry 
AICPA FN: Decision Making 
Blooms: Remember 
Difficulty: 2 Medium 
Learning Objective: 01-07 Explain the regulatory process for auditors of public companies and auditors of nonpublic companies. 
Topic: Public Accounting Profession 
7. 
Many small companies elect to have their financial statements reviewed by a CPA firm, rather 
than incur the cost of an audit. 
TRUE 
AACSB: Analytic 
AICPA BB: Industry 
AICPA FN: Decision Making 
Blooms: Remember 
Difficulty: 1 Easy 
Learning Objective: 01-07 Explain the regulatory process for auditors of public companies and auditors of nonpublic companies. 
Topic: Public Accounting Profession 
8. 
Staff assistants in CPA firms generally are responsible for planning and coordinating audit 
engagements. 
FALSE 
AACSB: Analytic 
AICPA BB: Industry 
AICPA FN: Decision Making 
Blooms: Remember 
Difficulty: 1 Easy
Learning Objective: 01-08 Describe how public accounting firms are typically organized and the responsibilities of auditors at the 
1-13 
various levels in the organization. 
Topic: Public Accounting Profession 
9. 
The Sarbanes-Oxley Act requires that auditors of certain publicly traded companies in the 
United States perform an integrated audit that includes providing assurance on both the 
financial statements and on compliance with laws and regulations. 
FALSE 
AACSB: Analytic 
AICPA BB: Industry 
AICPA FN: Decision Making 
Blooms: Remember 
Difficulty: 2 Medium 
Learning Objective: 01-06 Contrast the various types of audits and types of auditors. 
Topic: Financial Statement Audits 
10. 
Auditing is frequently only a small part of the practice of local CPA firms. 
TRUE 
AACSB: Analytic 
AICPA BB: Industry 
AICPA FN: Decision Making 
Blooms: Remember 
Difficulty: 2 Medium 
Learning Objective: 01-08 Describe how public accounting firms are typically organized and the responsibilities of auditors at the 
various levels in the organization. 
Topic: Public Accounting Profession 
Multiple Choice Questions 
11. 
A summary of findings rather than assurance is most likely to be included in a(n): 
A. 
Agreed-upon procedures report. 
B. 
Compilation report. 
C. 
Examination report. 
D. 
Review report. 
AACSB: Analytic 
AICPA BB: Industry 
AICPA FN: Decision Making 
Blooms: Remember 
Difficulty: 2 Medium 
Learning Objective: 01-02 Identify assurance services that involve attestation. 
Topic: Attest Function
12. 
The Statements on Auditing Standards have been issued by the: 
A. 
Auditing Standards Board. 
B. 
Financial Accounting Standards Board. 
C. 
Securities and Exchange Commission. 
D. 
Federal Bureau of Investigation. 
1-14 
AACSB: Analytic 
AICPA BB: Industry 
AICPA FN: Decision Making 
Blooms: Remember 
Difficulty: 1 Easy 
Learning Objective: 01-07 Explain the regulatory process for auditors of public companies and auditors of nonpublic companies. 
Topic: Public Accounting Profession 
13. 
The risk associated with a company's survival and profitability is referred to as: 
A. 
Business Risk. 
B. 
Information Risk. 
C. 
Detection Risk. 
D. 
Control Risk. 
AACSB: Analytic 
AICPA BB: Industry 
AICPA FN: Decision Making 
Blooms: Remember 
Difficulty: 1 Easy 
Learning Objective: 01-04 Explain why audits are demanded by society. 
Topic: Financial Statement Audits 
14. 
Historically, which of the following has the AICPA been most concerned with providing? 
A. 
Professional standards for CPAs. 
B. 
Professional guidance for regulating financial markets. 
C. 
Standards guiding the conduct of internal auditors. 
D. 
Staff support to Congress. 
AACSB: Analytic 
AICPA BB: Industry 
AICPA FN: Decision Making 
Blooms: Remember 
Difficulty: 2 Medium 
Learning Objective: 01-07 Explain the regulatory process for auditors of public companies and auditors of nonpublic companies. 
Topic: Public Accounting Profession
15. 
The organization charged with protecting investors and the public by requiring full disclosure of 
financial information by companies offering securities to the public is the: 
A. 
Auditing Standards Board. 
B. 
Financial Accounting Standards Board. 
C. 
Government Accounting Standards Boards. 
D. 
Securities and Exchange Commission. 
1-15 
AACSB: Analytic 
AICPA BB: Industry 
AICPA FN: Decision Making 
Blooms: Remember 
Difficulty: 2 Medium 
Learning Objective: 01-07 Explain the regulatory process for auditors of public companies and auditors of nonpublic companies. 
Topic: Public Accounting Profession 
16. 
An engagement in which a CPA firm arranges for a critical review of its practices by another 
CPA firm is referred to as a(n): 
A. 
Peer Review Engagement. 
B. 
Quality Control Engagement. 
C. 
Quality Assurance Engagement. 
D. 
Attestation Engagement. 
AACSB: Analytic 
AICPA BB: Industry 
AICPA FN: Decision Making 
Blooms: Remember 
Difficulty: 1 Easy 
Learning Objective: 01-07 Explain the regulatory process for auditors of public companies and auditors of nonpublic companies. 
Topic: Public Accounting Profession 
17. 
The serially-numbered pronouncements issued by the Auditing Standards Board over a period 
of years are known as: 
A. 
Auditing Statements of Position (ASPs). 
B. 
Accounting Series Releases (ASRs). 
C. 
Statements on Auditing Standards (SASs). 
D. 
Statements on Auditing Principles (SAPs). 
AACSB: Analytic 
AICPA BB: Industry 
AICPA FN: Decision Making 
Blooms: Remember 
Difficulty: 1 Easy 
Learning Objective: 01-07 Explain the regulatory process for auditors of public companies and auditors of nonpublic companies. 
Topic: Public Accounting Profession
18. 
The Government Accountability Office (GAO): 
A. 
Is primarily concerned with rapid processing of all accounts payable incurred by the federal 
1-16 
government. 
B. 
Conducts operational audits and reports the results to Congress. 
C. 
Is a multinational organization of professional accountants. 
D. 
Is primarily concerned with budgets and forecasts approved by the SEC. 
AACSB: Analytic 
AICPA BB: Industry 
AICPA FN: Decision Making 
Blooms: Remember 
Difficulty: 1 Easy 
Learning Objective: 01-06 Contrast the various types of audits and types of auditors. 
Topic: Financial Statement Audits 
19. 
The risk that information is misstated is referred to as: 
A. 
Information risk. 
B. 
Inherent risk. 
C. 
Relative risk. 
D. 
Business risk. 
AACSB: Analytic 
AICPA BB: Industry 
AICPA FN: Decision Making 
Blooms: Remember 
Difficulty: 1 Easy 
Learning Objective: 01-04 Explain why audits are demanded by society. 
Topic: Financial Statement Audits 
20. 
The risk that a company will not be able to meet its obligations when they become due is an 
aspect of: 
A. 
Information risk. 
B. 
Inherent risk. 
C. 
Relative risk. 
D. 
Business risk. 
AACSB: Analytic 
AICPA BB: Industry 
AICPA FN: Decision Making 
Blooms: Remember 
Difficulty: 1 Easy 
Learning Objective: 01-04 Explain why audits are demanded by society. 
Topic: Financial Statement Audits
21. 
Which of the following attributes most clearly differentiates a CPA who audits management's 
financial statements as contrasted to management? 
A. 
Integrity. 
B. 
Competence. 
C. 
Independence. 
D. 
Keeping informed on current professional developments. 
1-17 
AACSB: Analytic 
AICPA BB: Industry 
AICPA FN: Decision Making 
Blooms: Remember 
Difficulty: 1 Easy 
Learning Objective: 01-01 Describe the nature of assurance services. 
Topic: Assurance Services 
22. 
The attest function: 
A. 
Is an essential part of every engagement by the CPA, whether performing auditing, tax 
work, or other services. 
B. 
Includes the preparation of a report of the CPA's findings. 
C. 
Requires a consideration of internal control. 
D. 
Requires a complete review of all transactions during the period under examination. 
AACSB: Analytic 
AICPA BB: Industry 
AICPA FN: Decision Making 
Blooms: Remember 
Difficulty: 2 Medium 
Learning Objective: 01-02 Identify assurance services that involve attestation. 
Topic: Attest Function 
23. 
Attestation risk is limited to a low level in which of the following engagement(s)? 
A. 
Both examinations and reviews. 
B. 
Examinations, but not reviews. 
C. 
Reviews, but not examinations. 
D. 
Neither examinations nor reviews. 
AACSB: Analytic 
AICPA BB: Industry 
AICPA FN: Decision Making 
Blooms: Remember 
Difficulty: 1 Easy 
Learning Objective: 01-02 Identify assurance services that involve attestation. 
Topic: Attest Function
24. 
When compared to an audit performed prior to 1900, an audit today: 
A. 
Is more likely to include tests of compliance with laws and regulations. 
B. 
Is less likely to include consideration of the effectiveness of internal control. 
C. 
Has bank loan officers as the primary financial statement user group. 
D. 
Includes a more detailed examination of all individual transactions. 
1-18 
AACSB: Analytic 
AICPA BB: Industry 
AICPA FN: Decision Making 
Blooms: Remember 
Difficulty: 2 Medium 
Learning Objective: 01-04 Explain why audits are demanded by society. 
Topic: Financial Statement Audits 
25. 
Which of the following are issued by the Securities and Exchange Commission? 
A. 
Accounting Research Studies. 
B. 
Accounting Trends and Techniques. 
C. 
Industry Audit Guides. 
D. 
Financial Reporting Releases. 
AACSB: Analytic 
AICPA BB: Industry 
AICPA FN: Decision Making 
Blooms: Remember 
Difficulty: 2 Medium 
Learning Objective: 01-07 Explain the regulatory process for auditors of public companies and auditors of nonpublic companies. 
Topic: Public Accounting Profession 
26. 
Which of the following is not correct relating to the Sarbanes-Oxley Act? 
A. 
It toughens penalties for corporate fraud. 
B. 
It restricts the types of consulting CPAs may perform for audit clients. 
C. 
It created the Public Company Accounting Oversight Board (PCAOB) as a replacement for 
the Financial Accounting Standards Board. 
D. 
It eliminates a significant portion of the accounting profession's system of self-regulation. 
AACSB: Analytic 
AICPA BB: Industry 
AICPA FN: Decision Making 
Blooms: Remember 
Difficulty: 2 Medium 
Learning Objective: 01-05 Describe how the credibility of the accounting profession was affected by the large number of 
companies reporting accounting irregularities in the beginning of this century. 
Topic: Financial Statement Audits
27. 
An operational audit differs in many ways from an audit of financial statements. Which of the 
following is the best example of one of these differences? 
A. 
The usual audit of financial statements covers the four basic statements, whereas the 
operational audit is usually limited to either the balance sheet or the income statement. 
B. 
The boundaries of an operational audit are often drawn from an organization chart and are 
not limited to a single accounting period. 
C. 
Operational audits do not ordinarily result in the preparation of a report. 
D. 
The operational audit deals with pre-tax income. 
1-19 
AACSB: Analytic 
AICPA BB: Industry 
AICPA FN: Decision Making 
Blooms: Remember 
Difficulty: 2 Medium 
Learning Objective: 01-06 Contrast the various types of audits and types of auditors. 
Topic: Financial Statement Audits 
28. 
The review of a company's financial statements by a CPA firm: 
A. 
Is substantially less in scope of procedures than an audit. 
B. 
Requires detailed analysis of the major accounts. 
C. 
Is of similar scope as an audit and adds similar credibility to the statements. 
D. 
Culminates in issuance of a report expressing the CPA's opinion as to the fairness of the 
statements. 
AACSB: Analytic 
AICPA BB: Industry 
AICPA FN: Decision Making 
Blooms: Remember 
Difficulty: 1 Easy 
Learning Objective: 01-02 Identify assurance services that involve attestation. 
Topic: Attest Function 
29. 
Which statement is correct with respect to continuing professional education (CPE) 
requirements of members of the AICPA? 
A. 
Only members employed by the AICPA are required to take such courses. 
B. 
Only members in public practice are required to take such courses. 
C. 
Members, regardless of whether they are in public practice, are required to meet such 
requirements. 
D. 
There is no requirement for members to participate in CPE. 
AACSB: Analytic 
AICPA BB: Industry 
AICPA FN: Decision Making 
Blooms: Remember 
Difficulty: 2 Medium
Learning Objective: 01-07 Explain the regulatory process for auditors of public companies and auditors of nonpublic companies. 
1-20 
Topic: Public Accounting Profession 
30. 
The FDIC Improvement Act requires that management of large financial institutions engage 
auditors to attest to assertions by management about the effectiveness of the institution's 
internal controls over: 
A. 
Compliance with laws and regulations. 
B. 
Financial reporting. 
C. 
Effectiveness of operations. 
D. 
Efficiency of operations. 
AACSB: Analytic 
AICPA BB: Industry 
AICPA FN: Decision Making 
Blooms: Remember 
Difficulty: 2 Medium 
Learning Objective: 01-04 Explain why audits are demanded by society. 
Topic: Financial Statement Audits 
31. 
Passage of the Sarbanes-Oxley Act led to the establishment of the: 
A. 
Auditing Standards Board. 
B. 
Accounting Enforcement Releases Board. 
C. 
Public Company Accounting Oversight Board. 
D. 
Securities and Exchange Commission. 
AACSB: Analytic 
AICPA BB: Industry 
AICPA FN: Decision Making 
Blooms: Remember 
Difficulty: 2 Medium 
Learning Objective: 01-05 Describe how the credibility of the accounting profession was affected by the large number of 
companies reporting accounting irregularities in the beginning of this century. 
Topic: Financial Statement Audits 
32. 
Which of the following professionals has primary responsibility for the performance of an 
audit? 
A. 
The managing partner of the firm. 
B. 
The senior assigned to the engagement. 
C. 
The manager assigned to the engagement. 
D. 
The partner in charge of the engagement. 
AACSB: Analytic 
AICPA BB: Industry 
AICPA FN: Decision Making 
Blooms: Remember 
Difficulty: 2 Medium
Learning Objective: 01-08 Describe how public accounting firms are typically organized and the responsibilities of auditors at the 
1-21 
various levels in the organization. 
Topic: Public Accounting Profession 
33. 
Which of the following types of services is generally provided only by CPA firms? 
A. 
Tax audits. 
B. 
Financial statement audits. 
C. 
Compliance audits. 
D. 
Operational audits. 
AACSB: Analytic 
AICPA BB: Industry 
AICPA FN: Decision Making 
Blooms: Remember 
Difficulty: 2 Medium 
Learning Objective: 01-02 Identify assurance services that involve attestation. 
Topic: Attest Function 
34. 
The right to practice as a CPA is given by which of the following organizations? 
A. 
State Boards of Accountancy. 
B. 
The AICPA. 
C. 
The SEC. 
D. 
The General Accounting Office. 
AACSB: Analytic 
AICPA BB: Industry 
AICPA FN: Decision Making 
Blooms: Remember 
Difficulty: 2 Medium 
Learning Objective: 01-07 Explain the regulatory process for auditors of public companies and auditors of nonpublic companies. 
Topic: Public Accounting Profession 
35. 
Which of the following terms best describes the audit of a taxpayer's tax return by an IRS 
auditor? 
A. 
Operational audit. 
B. 
Internal audit. 
C. 
Compliance audit. 
D. 
Government audit. 
AACSB: Analytic 
AICPA BB: Industry 
AICPA FN: Decision Making 
Blooms: Remember 
Difficulty: 2 Medium 
Learning Objective: 01-06 Contrast the various types of audits and types of auditors. 
Topic: Financial Statement Audits
36. 
Inquiries and analytical procedures ordinarily form the basis for which type of engagement? 
1-22 
A. 
Agreed-upon procedures. 
B. 
Audit. 
C. 
Examination. 
D. 
Review. 
AACSB: Analytic 
AICPA BB: Industry 
AICPA FN: Decision Making 
Blooms: Remember 
Difficulty: 1 Easy 
Learning Objective: 01-02 Identify assurance services that involve attestation. 
Topic: Attest Function 
37. 
Which of the following best describes the reason why independent auditors report on financial 
statements? 
A. 
A management fraud may exist and it is more likely to be detected by independent auditors. 
B. 
Different interests may exist between the company preparing the statements and the 
persons using the statements. 
C. 
A misstatement of account balances may exist and is generally corrected as the result of 
the independent auditors' work. 
D. 
Poorly designed internal control may be in existence. 
AACSB: Analytic 
AICPA BB: Industry 
AICPA FN: Decision Making 
Blooms: Remember 
Difficulty: 2 Medium 
Source: AICPA 
Topic: Financial Statement Audits 
38. 
Governmental auditing often extends beyond examinations leading to the expression of opinion 
on the fairness of financial presentation and includes audits of efficiency, economy, 
effectiveness, and also: 
A. 
Accuracy. 
B. 
Evaluation. 
C. 
Compliance. 
D. 
Internal control. 
AACSB: Reflective Thinking 
AICPA BB: Critical Thinking 
AICPA FN: Decision Making 
Blooms: Understand 
Difficulty: 3 Hard 
Learning Objective: 01-06 Contrast the various types of audits and types of auditors. 
Source: AICPA
1-23 
Topic: Financial Statement Audits 
39. 
Operational auditing is primarily oriented toward: 
A. 
Future improvements to accomplish the goals of management. 
B. 
The accuracy of data reflected in management's financial records. 
C. 
The verification that a company's financial statements are fairly presented. 
D. 
Past protection provided by existing internal control. 
AACSB: Reflective Thinking 
AICPA BB: Critical Thinking 
AICPA FN: Decision Making 
Blooms: Understand 
Difficulty: 3 Hard 
Learning Objective: 01-06 Contrast the various types of audits and types of auditors. 
Source: AICPA 
Topic: Financial Statement Audits 
40. 
A typical objective of an operational audit is for the auditor to: 
A. 
Determine whether the financial statements fairly present the entity's operations. 
B. 
Evaluate the feasibility of attaining the entity's operational objectives. 
C. 
Make recommendations for improving performance. 
D. 
Report on the entity's relative success in attaining profit maximization. 
AACSB: Reflective Thinking 
AICPA BB: Critical Thinking 
AICPA FN: Decision Making 
Blooms: Understand 
Difficulty: 3 Hard 
Learning Objective: 01-06 Contrast the various types of audits and types of auditors. 
Source: AICPA 
Topic: Financial Statement Audits 
41. 
An integrated audit performed under the Sarbanes-Oxley Act requires that auditors report on: 
A. 
Option A 
B. 
Option B 
C. 
Option C 
D. 
Option D 
AACSB: Analytic
1-24 
AICPA BB: Industry 
AICPA FN: Decision Making 
Blooms: Remember 
Difficulty: 2 Medium 
Learning Objective: 01-06 Contrast the various types of audits and types of auditors. 
Topic: Financial Statement Audits 
Matching Questions 
42. 
Accountants are regulated by a variety of organizations. Match the following statements with 
the most directly related organizations. Organizations may be used once or not at all. 
1. Formed to improve standards of 
financial accounting for state and local 
government entities 
State Boards of 
Accountancy. 
2 
2. Issue CPA certificates 
Government Accounting 
Standards Board. 
1 
3. Develop accounting standards for 
public and nonpublic companies 
American Institute of 
Certified Public 
Accountants. 
6 
4. Develop accounting standards for 
the U.S. Government 
Financial Accounting 
Standards Board. 
3 
5. Issue auditing standards for public 
companies 
Federal Accounting 
Standards Advisory Board. 
4 
6. Prepares the CPA exam 
Public Company 
Accounting Oversight 
Board. 
5 
AACSB: Analytic 
AICPA BB: Industry 
AICPA FN: Decision Making 
Blooms: Remember 
Difficulty: 2 Medium 
Learning Objective: 01-06 Contrast the various types of audits and types of auditors. 
Learning Objective: 01-07 Explain the regulatory process for auditors of public companies and auditors of nonpublic companies. 
Topic: Financial Statement Audits 
Topic: Public Accounting Profession 
Essay Questions
43. 
The Sarbanes-Oxley Act of 2002 made significant reforms for public companies and their 
auditors. 
a. Describe the events that led up to the passage of the Act. 
b. Describe the major changes made by the Act. 
a. The events leading up to the passage of the Sarbanes-Oxley Act include: 
• A large number of misstatements of financial statements, many of which resulted from 
fraudulent financial reporting. Notably including WorldCom and Enron. 
• The conviction of the Big 5 accounting firm of Arthur Andersen on charges of destroying 
evidence. 
b. The major reforms made the Act include: 
• Tougher penalties for fraud. 
• Restrictions on the types of consulting services that may be provided by auditors to their 
public audit clients. 
• The creation of the Public Company Accounting Oversight Board to create auditing standards 
and oversee accounting firms that audit public companies. 
• Requirements for management to make an assertion about the effectiveness of internal 
control. 
• Requirements for auditors of public companies to audit and report on internal control. 
1-25 
AACSB: Reflective Thinking 
AICPA BB: Critical Thinking 
AICPA FN: Decision Making 
Blooms: Understand 
Difficulty: 3 Hard 
Learning Objective: 01-05 Describe how the credibility of the accounting profession was affected by the large number of 
companies reporting accounting irregularities in the beginning of this century. 
Learning Objective: 01-06 Contrast the various types of audits and types of auditors. 
Topic: Financial Statement Audits
44. 
Many people confuse the responsibilities of the independent auditors and the client's 
management with respect to audited financial statements. 
a. Describe management's responsibility regarding audited financial statements. 
b. Describe the independent auditors' responsibility regarding audited financial statements. 
c. Evaluate the following statement: "If the auditors disagree with management regarding an 
accounting principle used in the financial statements the auditors should express their views in 
the notes to the financial statements." 
a. Management has primary responsibility for the fairness of the financial statements and 
internal control. 
b. The auditors are responsible for performing an independent audit of the financial statements 
and issuing a report on them in accordance with generally accepted auditing standards. 
c. The statement if false. The notes to the financial statements should contain only 
representations of management. The auditors should express their reservations in their report. 
1-26 
AACSB: Analytic 
AICPA BB: Industry 
AICPA FN: Decision Making 
Blooms: Remember 
Difficulty: 2 Medium 
Learning Objective: 01-02 Identify assurance services that involve attestation. 
Topic: Attest Function
45. 
An investor is considering investing in one of two companies. The companies have very similar 
reported financial position and results of operations. However, only one of the companies has 
its financial statements audited. 
a. Describe what creates the demand for an audit in this situation. Include a discussion of how 
audited financial statements facilitate this investment transaction, and the effect of the audit on 
business risk and information risk. 
b. Identify the potential consequences to the company of not having its financial statements 
audited. 
a. Audits add credibility to the financial statements of the company. The individual can invest in 
the company knowing that there is a low probability that the financial statements depart 
materially from generally accepted accounting principles. Audited financial statements facilitate 
this transaction by reducing risk related to the investment. Specifically, audits reduce 
information risk--the risk that information used to make the investment decision is misstated-- 
related to the financial statements. Audited financial statements do not directly affect business 
risk, which is the risk that the company will not be able to meet its financial obligations. 
b. The potential consequences of not having an audit are: 
• If the investor is particularly risk averse, he or she may not invest in the company at all. 
• If the investor decides to invest in the company, he or she will not be willing to pay as high a 
price because the investor will want to be compensated for the additional risk that is involved in 
relying upon unaudited financial statements. 
1-27 
AACSB: Analytic 
AICPA BB: Industry 
AICPA FN: Decision Making 
Blooms: Remember 
Difficulty: 2 Medium 
Learning Objective: 01-04 Explain why audits are demanded by society. 
Topic: Financial Statement Audits
1-28 
Chapter 
02 
Professional 
Standards 
Multiple 
Choice 
Questions 
1. 
The 
attestation 
standards 
of 
reporting 
do 
not 
require 
the 
attestation 
report 
to 
include 
a 
statement 
that 
A. 
Provides 
a 
conclusion 
whether 
the 
subject 
matter 
is 
presented 
in 
conformity 
with 
established 
or 
stated 
criteria. 
B. 
Indicates 
that 
the 
practitioner 
has 
significant 
reservations 
about 
the 
engagement. 
C. 
Identifies 
the 
subject 
matter 
or 
assertion 
being 
reported 
on. 
D. 
Indicates 
that 
the 
accountant 
assumes 
no 
responsibility 
to 
update 
the 
report. 
2. 
Control 
risk 
is 
A. 
The 
probability 
that 
a 
material 
misstatement 
could 
not 
be 
prevented 
or 
detected 
by 
the 
entity's 
internal 
control 
policies 
and 
procedures. 
B. 
The 
probability 
that 
a 
material 
misstatement 
could 
occur 
and 
not 
be 
detected 
by 
auditors' 
procedures. 
C. 
The 
risk 
that 
auditors 
will 
not 
be 
able 
to 
complete 
the 
audit 
on 
a 
timely 
basis. 
D. 
The 
risk 
that 
auditors 
will 
not 
properly 
control 
the 
staff 
on 
the 
audit 
engagement. 
3. 
The 
responsibilities 
principle 
under 
generally 
accepted 
auditing 
standards 
does 
not 
include 
which 
of 
the 
following? 
A. 
Competence 
and 
capabilities. 
B. 
Independent 
attitude. 
C. 
Due 
care. 
D. 
Planning 
and 
supervision. 
4. 
Which 
of 
the 
following 
types 
of 
auditors' 
reports 
does 
not 
require 
an 
explanatory 
paragraph 
to 
support 
the 
opinion? 
A. 
Unqualified 
opinion. 
B. 
Adverse 
opinion. 
C. 
Qualified 
opinion. 
D. 
Disclaimer 
of 
opinion.
5. 
Which 
of 
the 
following 
is 
an 
element 
of 
a 
system 
of 
quality 
control 
that 
should 
be 
considered 
by 
a 
public 
accounting 
firm 
in 
establishing 
its 
quality 
control 
policies 
and 
procedures? 
A. 
Lending 
credibility 
to 
a 
client's 
financial 
statements. 
B. 
Using 
statistical 
sampling 
techniques. 
C. 
Acceptance 
and 
continuance 
of 
clients. 
D. 
Membership 
in 
the 
Center 
for 
Public 
Company 
Audit 
Firms 
(CPCAF). 
1-29 
6. 
Which 
of 
the 
following 
presumptions 
does 
not 
relate 
to 
the 
reliability 
of 
audit 
evidence? 
A. 
The 
more 
effective 
the 
client's 
internal 
control, 
the 
more 
assurance 
it 
provides 
about 
the 
accounting 
data 
and 
financial 
statements. 
B. 
The 
auditors' 
opinion, 
to 
be 
economically 
useful, 
is 
formed 
within 
reasonable 
time 
and 
based 
on 
evidence 
obtained 
at 
a 
reasonable 
cost. 
C. 
Evidence 
obtained 
from 
independent 
sources 
outside 
the 
entity 
is 
more 
reliable 
than 
evidence 
secured 
solely 
within 
the 
entity. 
D. 
The 
independent 
auditors' 
direct 
personal 
knowledge, 
obtained 
through 
observation 
and 
inspection, 
is 
more 
persuasive 
than 
information 
obtained 
indirectly. 
7. 
An 
important 
role 
of 
the 
Public 
Company 
Accounting 
Oversight 
Board 
(PCAOB) 
is 
to 
oversee 
the 
A. 
Issuance 
of 
statements 
by 
the 
Financial 
Accounting 
Standards 
Board. 
B. 
Preparation 
and 
grading 
of 
the 
Uniform 
CPA 
Examination. 
C. 
Peer 
review 
of 
member 
firms 
of 
the 
Private 
Companies 
Practice 
Section. 
D. 
Regulation 
of 
firms 
that 
audit 
public 
companies. 
8. 
Audit 
evidence 
is 
usually 
considered 
sufficient 
when 
A. 
It 
is 
reliable. 
B. 
There 
is 
enough 
quantity 
to 
afford 
a 
reasonable 
basis 
for 
an 
opinion 
on 
financial 
statements. 
C. 
It 
has 
the 
qualities 
of 
being 
relevant, 
objective, 
and 
free 
from 
unknown 
bias. 
D. 
It 
has 
been 
obtained 
through 
random 
selection 
methods.
1-30 
9. 
Which 
of 
the 
following 
is 
not 
considered 
a 
type 
of 
audit 
evidence? 
A. 
The 
company's 
trial 
balance. 
B. 
Auditors' 
calculations. 
C. 
Physical 
observation. 
D. 
Verbal 
statements 
made 
by 
client 
personnel. 
10. 
The 
AICPA 
attestation 
standards 
differ 
from 
the 
AICPA 
responsibilities 
principle, 
performance 
principle 
and 
reporting 
principle 
in 
that: 
A. 
The 
attestation 
standards 
contain 
no 
requirement 
to 
obtain 
an 
understanding 
of 
the 
entity 
and 
assess 
the 
risk 
of 
material 
misstatement. 
B. 
The 
attestation 
standards 
do 
not 
require 
competence 
and 
capabilities. 
C. 
The 
attestation 
standards 
do 
not 
require 
planning 
for 
attestation 
engagements 
or 
supervision 
of 
accountants 
and 
consultants 
who 
perform 
the 
work. 
D. 
The 
attestation 
standards 
do 
not 
require 
a 
report 
that 
states 
the 
character 
of 
the 
engagement. 
11. 
An 
audit 
of 
the 
financial 
statements 
of 
Camden 
Corporation 
is 
being 
conducted 
by 
external 
auditors. 
The 
external 
auditors 
are 
expected 
to: 
A. 
Certify 
the 
correctness 
of 
Camden's 
financial 
statements. 
B. 
Make 
a 
complete 
examination 
of 
Camden's 
records 
and 
verify 
all 
of 
Camden's 
transactions. 
C. 
Give 
an 
opinion 
on 
the 
fair 
presentation 
of 
Camden's 
financial 
statements 
in 
conformity 
with 
the 
applicable 
financial 
reporting 
framework 
(e.g., 
GAAP, 
IFRS). 
D. 
Give 
an 
opinion 
on 
the 
attractiveness 
of 
Camden 
for 
investment 
purposes 
and 
critique 
the 
wisdom 
and 
legality 
of 
its 
business 
decisions. 
12. 
Auditors 
try 
to 
achieve 
independence 
in 
appearance 
in 
order 
to: 
A. 
Maintain 
public 
confidence 
in 
the 
profession. 
B. 
Become 
independent 
in 
fact. 
C. 
Comply 
with 
the 
responsibilities 
principle. 
D. 
Maintain 
an 
unbiased 
mental 
attitude.
13. 
The 
independent 
auditors' 
plan 
prepared 
prior 
to 
the 
start 
of 
field 
work 
is 
appropriately 
considered 
documentation 
of 
A. 
Planning. 
B. 
Supervision. 
C. 
Information 
evaluation. 
D. 
Quality 
assurance. 
14. 
Which 
of 
the 
following 
procedures 
would 
provide 
the 
most 
reliable 
audit 
evidence? 
A. 
Inquiries 
of 
the 
client's 
accounting 
staff 
held 
in 
private. 
B. 
Inspection 
of 
pre-­‐numbered 
client 
shipping 
documents. 
C. 
Inspection 
of 
bank 
statements 
obtained 
directly 
from 
the 
client's 
financial 
institution. 
D. 
Analytical 
procedures 
performed 
by 
auditors 
on 
the 
client's 
trial 
balance. 
15. 
Which 
of 
the 
following 
is 
not 
an 
attestation 
standard? 
A. 
The 
practitioner 
must 
obtain 
sufficient 
evidence 
to 
provide 
a 
reasonable 
basis 
for 
the 
conclusion 
expressed 
in 
the 
report. 
B. 
The 
practitioner 
must 
identify 
the 
subject 
matter 
or 
the 
assertion 
being 
reported 
on 
and 
state 
the 
character 
of 
the 
engagement. 
C. 
The 
practitioner 
must 
adequately 
plan 
the 
work 
and 
must 
properly 
supervise 
any 
assistants. 
D. 
A 
sufficient 
understanding 
of 
the 
client's 
internal 
controls 
shall 
be 
obtained 
to 
plan 
the 
engagement. 
16. 
Which 
of 
the 
following 
would 
most 
likely 
be 
a 
violation 
of 
the 
independence 
requirement 
found 
in 
the 
responsibilities 
principle 
under 
generally 
accepted 
auditing 
standards? 
A. 
An 
auditor 
on 
the 
engagement 
has 
a 
distant 
relative 
who 
is 
employed 
by 
a 
vendor 
that 
does 
a 
significant 
amount 
of 
business 
with 
clients. 
B. 
The 
client's 
Chief 
Executive 
Officer 
graduated 
from 
the 
same 
university 
as 
the 
partner 
in 
charge 
of 
the 
accounting 
firm. 
C. 
An 
auditor 
on 
the 
engagement 
owns 
a 
financial 
interest 
in 
the 
stock 
of 
the 
client. 
D. 
The 
client 
provides 
financial 
support 
to 
a 
number 
of 
charitable 
causes 
that 
also 
receive 
support 
from 
the 
accounting 
firm. 
1-31
17. 
A 
vendor's 
invoice 
received 
and 
held 
by 
the 
client 
would 
be 
considered 
what 
type 
of 
evidence? 
A. 
External. 
B. 
Internal. 
C. 
External-­‐internal. 
D. 
Written 
representation. 
18. 
Which 
of 
the 
following 
statements 
is 
generally 
correct 
about 
the 
appropriateness 
of 
audit 
evidence? 
A. 
Auditors' 
direct 
personal 
knowledge, 
obtained 
through 
observation 
and 
inspection, 
is 
more 
persuasive 
than 
information 
obtained 
indirectly 
from 
independent 
outside 
sources. 
B. 
To 
be 
reliable, 
audit 
evidence 
must 
be 
either 
valid 
or 
relevant, 
but 
need 
not 
be 
both. 
C. 
Client 
accounting 
data 
alone 
may 
be 
considered 
sufficient 
appropriate 
audit 
evidence 
to 
issue 
an 
unqualified 
opinion 
on 
client 
financial 
statements. 
D. 
Appropriateness 
of 
audit 
evidence 
refers 
to 
the 
amount 
of 
corroborative 
evidence 
to 
be 
obtained. 
19. 
The 
standard 
auditors' 
report 
refers 
to 
GAAS 
and 
GAAP 
in 
which 
paragraph? 
A. 
GAAS: 
Scope 
only; 
GAAP: 
Opinion 
only 
B. 
GAAS: 
Introductory 
only; 
GAAP: 
Scope 
and 
opinion 
C. 
GAAS: 
Introductory 
and 
scope; 
GAAP: 
Opinion 
only 
D. 
GAAS: 
Introductory 
only; 
GAAP: 
All 
paragraphs 
20. 
Which 
of 
the 
following 
is 
not 
included 
in 
the 
auditors' 
standard 
report 
representing 
an 
unqualified 
opinion? 
A. 
A 
brief 
indication 
of 
the 
responsibility 
of 
auditors 
and 
management 
for 
the 
financial 
statements. 
B. 
An 
indication 
that 
all 
appropriate 
disclosures 
have 
been 
made 
and 
included 
in 
the 
financial 
statements. 
C. 
An 
indication 
that 
the 
audit 
was 
conducted 
in 
accordance 
with 
standards 
established 
by 
the 
PCAOB. 
D. 
The 
auditors' 
opinion 
on 
the 
fairness 
of 
the 
financial 
statements. 
1-32
21. 
Internal 
evidence 
A. 
Is 
obtained 
directly 
from 
third 
parties 
independent 
of 
the 
client. 
B. 
Originates 
outside 
of 
the 
client's 
system 
but 
has 
been 
received 
and 
processed 
by 
the 
client. 
C. 
Consists 
of 
documents 
that 
are 
produced, 
used, 
and 
stored 
within 
the 
client's 
information 
system. 
D. 
Consists 
of 
representations 
made 
by 
the 
client's 
officers, 
directors, 
owners, 
and 
employees. 
22. 
Which 
of 
the 
following 
presumptions 
is 
correct 
about 
the 
reliability 
of 
audit 
evidence? 
A. 
Information 
obtained 
indirectly 
from 
outside 
sources 
is 
the 
most 
reliable 
form 
of 
audit 
evidence. 
B. 
To 
be 
reliable, 
audit 
evidence 
should 
be 
convincing 
rather 
than 
persuasive. 
C. 
Reliability 
of 
audit 
evidence 
refers 
to 
the 
amount 
of 
corroborative 
evidence 
obtained. 
D. 
An 
effective 
system 
of 
internal 
control 
provides 
more 
assurance 
about 
the 
reliability 
of 
audit 
evidence. 
23. 
When 
auditors 
do 
not 
mention 
consistency 
in 
the 
auditors' 
report, 
a 
reader 
of 
the 
financial 
statements 
may 
infer 
A. 
That 
the 
applicable 
financial 
reporting 
framework 
(e.g., 
GAAP) 
has 
been 
consistently 
observed 
in 
the 
current 
period 
in 
relation 
to 
the 
preceding 
period. 
B. 
That 
no 
material 
departure 
from 
the 
applicable 
financial 
reporting 
framework 
(e.g., 
GAAP) 
has 
been 
detected. 
C. 
That 
no 
reclassification 
of 
items 
or 
change 
in 
classifications 
has 
occurred. 
D. 
Nothing 
about 
application 
of 
accounting 
principles 
within 
the 
period. 
24. 
The 
auditors' 
responsibility 
to 
express 
an 
opinion 
on 
the 
financial 
statements 
is 
A. 
Implicitly 
represented 
in 
the 
auditors' 
standard 
report. 
B. 
Explicitly 
represented 
in 
the 
introductory 
paragraph 
of 
the 
auditors' 
standard 
report. 
C. 
Explicitly 
represented 
in 
the 
scope 
paragraph 
of 
the 
auditors' 
standard 
report. 
D. 
Explicitly 
represented 
in 
the 
opinion 
paragraph 
of 
the 
auditors' 
standard 
report. 
1-33
25. 
Which 
of 
the 
following 
is 
not 
a 
concept 
from 
the 
performance 
principle 
under 
generally 
accepted 
auditing 
standards? 
A. 
The 
auditor 
must 
plan 
the 
work 
and 
properly 
supervise 
any 
assistants. 
B. 
The 
auditor 
must 
express 
an 
opinion 
in 
accordance 
with 
the 
auditor's 
findings. 
C. 
The 
auditor 
must 
obtain 
sufficient 
appropriate 
evidence 
about 
whether 
material 
misstatements 
exist. 
D. 
The 
auditor 
must 
determine 
and 
apply 
an 
appropriate 
materiality 
level 
throughout 
the 
audit. 
26. 
Under 
generally 
accepted 
auditing 
standards, 
which 
of 
the 
following 
reflects 
a 
concept 
related 
to 
the 
responsibilities 
principle? 
A. 
The 
initial 
planning 
of 
the 
audit 
engagement 
should 
occur 
with 
the 
audit 
partner, 
manager, 
senior, 
and 
client 
personnel. 
B. 
The 
confirmation 
of 
accounts 
receivable 
should 
occur 
on 
each 
audit. 
C. 
The 
completion 
of 
an 
internal 
control 
questionnaire. 
D. 
Maintaining 
professional 
skepticism 
and 
exercising 
professional 
judgment. 
27. 
Which 
of 
the 
following 
represent 
audit 
quality 
guides 
that 
remain 
stable 
over 
time 
and 
are 
applicable 
for 
all 
audits? 
A. 
Auditing 
procedures. 
B. 
Auditing 
standards. 
C. 
Due 
care. 
D. 
System 
of 
quality 
control. 
28. 
Which 
of 
the 
following 
situations 
would 
most 
likely 
be 
in 
conflict 
with 
the 
responsibilities 
principle? 
A. 
Auditors 
perform 
the 
engagement 
with 
prudent 
auditors, 
but 
not 
expert 
auditors. 
B. 
Auditors 
obtain 
expertise 
in 
their 
client's 
industry 
as 
they 
are 
conducting 
the 
audit 
examination. 
C. 
Auditors 
are 
directly 
involved 
with 
a 
client 
manager 
in 
a 
strategic 
decision-­‐making 
capacity. 
D. 
Auditors 
fail 
to 
document 
their 
assessment 
of 
control 
risk 
following 
their 
study 
of 
internal 
control. 
1-34
1-35 
29. 
Which 
of 
the 
following 
statements 
is 
not 
true 
with 
respect 
to 
the 
evidence 
that 
would 
be 
gathered 
when 
assessments 
of 
control 
risk 
are 
high? 
A. 
Auditors 
would 
be 
required 
to 
rely 
on 
external 
(rather 
than 
internal) 
forms 
of 
evidence. 
B. 
Auditors 
would 
be 
required 
to 
perform 
procedures 
at 
interim 
periods, 
rather 
than 
at 
year 
end. 
C. 
Auditors 
would 
be 
required 
to 
confirm 
a 
larger 
number 
of 
customer 
accounts 
receivable 
balances. 
D. 
Auditors 
would 
be 
required 
to 
obtain 
more 
evidence 
through 
direct 
personal 
observation. 
30. 
As 
it 
relates 
to 
audit 
evidence, 
appropriateness 
refers 
to 
the 
A. 
Originality 
of 
evidence 
gathered. 
B. 
Quality 
of 
evidence 
gathered. 
C. 
Quantity 
of 
evidence 
gathered. 
D. 
Timeliness 
of 
evidence 
gathered. 
31. 
Which 
of 
the 
following 
information 
would 
not 
be 
included 
in 
the 
auditors' 
standard 
report? 
A. 
The 
names 
of 
the 
financial 
statements 
audited. 
B. 
A 
description 
of 
the 
nature 
of 
an 
audit. 
C. 
An 
indication 
that 
all 
necessary 
disclosures 
have 
been 
presented. 
D. 
An 
opinion 
on 
the 
entity's 
financial 
statements. 
32. 
The 
primary 
purpose 
of 
the 
auditors' 
study 
of 
internal 
control 
for 
a 
nonpublic 
entity 
is: 
A. 
To 
provide 
constructive 
suggestions 
to 
the 
client 
for 
improving 
its 
internal 
control. 
B. 
To 
report 
on 
internal 
control 
as 
required 
by 
Auditing 
Standard 
No. 
5. 
C. 
To 
identify 
and 
detect 
fraud 
and 
irregularities 
perpetrated 
by 
client 
personnel. 
D. 
To 
determine 
the 
nature, 
timing, 
and 
extent 
of 
substantive 
procedures. 
33. 
Which 
reporting 
options 
do 
auditors 
have 
if 
the 
client's 
financial 
statements 
are 
not 
presented 
according 
to 
the 
applicable 
financial 
framework 
(e.g., 
GAAP, 
IFRS)? 
A. 
Unqualified 
or 
disclaimer 
of 
opinion. 
B. 
Qualified 
or 
disclaimer 
of 
opinion. 
C. 
Unqualified 
or 
adverse. 
D. 
Qualified 
or 
adverse.
34. 
Which 
of 
the 
following 
is 
most 
closely 
related 
to 
system 
of 
quality 
control 
regarding 
engagement 
performance? 
A. 
Requiring 
all 
of 
the 
firm's 
personnel 
to 
provide 
a 
summary 
of 
their 
investments 
and 
other 
financial 
relationships. 
B. 
Evaluating 
the 
firm's 
system 
of 
quality 
controls 
on 
a 
periodic 
basis. 
C. 
Utilizing 
standardized 
audit 
plans 
and 
audit 
documentation 
on 
engagements 
in 
a 
particular 
industry. 
D. 
Evaluating 
the 
firm's 
ability 
to 
provide 
a 
quality 
audit 
to 
a 
prospective 
client. 
1-36 
Question 
also 
found 
in 
textbook 
35. 
Which 
of 
the 
following 
categories 
of 
principles 
is 
most 
closely 
related 
to 
gathering 
audit 
evidence? 
A. 
Performance. 
B. 
Reasonable 
assurance. 
C. 
Reporting. 
D. 
Responsibilities. 
36. 
To 
exercise 
due 
care, 
an 
accountant 
should 
A. 
Take 
continuing 
professional 
education 
classes. 
B. 
Report 
whether 
the 
financial 
statements 
are 
in 
accordance 
with 
the 
applicable 
financial 
reporting 
framework 
(e.g., 
GAAP, 
IFRS). 
C. 
Gather 
enough 
audit 
evidence 
to 
have 
complete 
assurance 
that 
there 
is 
enough 
support 
for 
the 
accountant's 
opinion 
on 
the 
financial 
statements. 
D. 
Conduct 
the 
engagement 
in 
accordance 
with 
GAAS 
and 
ensure 
that 
the 
engagement 
is 
completed 
on 
a 
timely 
basis. 
37. 
One 
of 
an 
accounting 
firm's 
basic 
objectives 
is 
to 
provide 
professional 
services 
that 
conform 
to 
professional 
standards. 
Reasonable 
assurance 
of 
achieving 
this 
objective 
can 
be 
obtained 
by 
following 
A. 
Generally 
Accepted 
Auditing 
Standards 
(GAAS). 
B. 
Standards 
within 
a 
system 
of 
quality 
control. 
C. 
Generally 
Accepted 
Accounting 
Practices 
(GAAP). 
D. 
International 
Auditing 
Standards.
38. 
Which 
of 
the 
following 
best 
demonstrates 
the 
concept 
of 
professional 
skepticism? 
A. 
Relying 
more 
extensively 
on 
external 
evidence 
rather 
than 
internal 
evidence. 
B. 
Focusing 
on 
items 
that 
have 
a 
more 
significant 
quantitative 
effect 
on 
the 
entity's 
financial 
statements. 
C. 
Critically 
assessing 
verbal 
evidence 
received 
from 
the 
entity's 
management. 
D. 
Evaluating 
potential 
financial 
interests 
held 
by 
auditors 
in 
the 
client. 
39. 
The 
primary 
purpose 
for 
obtaining 
an 
understanding 
of 
the 
entity's 
environment 
(including 
its 
internal 
control) 
in 
a 
financial 
statement 
audit 
is 
A. 
To 
determine 
the 
nature, 
timing, 
and 
extent 
of 
further 
audit 
procedures 
to 
be 
performed. 
B. 
To 
make 
consulting 
suggestions 
to 
the 
management. 
C. 
To 
obtain 
direct 
sufficient 
appropriate 
audit 
evidence 
to 
afford 
a 
reasonable 
basis 
for 
an 
opinion 
on 
the 
financial 
statements. 
D. 
To 
determine 
whether 
the 
entity 
has 
changed 
any 
accounting 
principles. 
40. 
Ordinarily, 
what 
source 
of 
evidence 
should 
least 
affect 
audit 
conclusions? 
A. 
External. 
B. 
Inquiry 
of 
management. 
C. 
Auditor 
prepared. 
D. 
Inquiry 
of 
entity 
legal 
counsel. 
41. 
The 
most 
persuasive 
evidence 
regarding 
the 
existence 
of 
newly 
acquired 
computer 
equipment 
is 
A. 
Inquiry 
of 
management. 
B. 
Documentation 
prepared 
externally. 
C. 
Observation 
of 
auditee's 
procedures. 
D. 
Physical 
observation. 
42. 
Which 
of 
the 
following 
procedures 
would 
provide 
the 
most 
reliable 
audit 
evidence? 
A. 
Inquiries 
of 
the 
client's 
internal 
audit 
staff 
held 
in 
private. 
B. 
Inspection 
of 
pre-­‐numbered 
client 
purchase 
orders 
filed 
in 
the 
vouchers 
payable 
department. 
C. 
Inspection 
of 
vendor 
sales 
invoices 
received 
from 
client 
personnel. 
D. 
Inspection 
of 
bank 
statements 
obtained 
directly 
from 
the 
client's 
financial 
institution. 
1-37
43. 
Breaux 
& 
Co., 
CPAs 
require 
that 
all 
audit 
documentation 
contain 
the 
initials 
of 
the 
preparer 
and 
the 
reviewer 
in 
the 
top 
right-­‐hand 
corner. 
This 
procedure 
provides 
evidence 
of 
Breaux 
& 
Co., 
CPAs' 
professional 
concern 
regarding 
which 
of 
the 
following? 
A. 
Independence. 
B. 
Adequate 
competence 
and 
capabilities. 
C. 
Adequate 
planning 
and 
supervision. 
D. 
Gathering 
sufficient 
appropriate 
evidence. 
1-38 
44. 
The 
attestation 
standards 
do 
not 
contain 
a 
requirement 
that 
auditors 
obtain 
A. 
Adequate 
knowledge 
in 
the 
subject 
matter 
of 
the 
assertions 
being 
examined. 
B. 
An 
understanding 
of 
the 
auditee's 
internal 
controls. 
C. 
Sufficient 
evidence 
for 
the 
conclusions 
expressed 
in 
an 
attestation 
report. 
D. 
Independence 
in 
mental 
attitude. 
45. 
Which 
of 
the 
following 
concepts 
is 
least 
related 
to 
the 
standard 
of 
due 
care? 
A. 
Independence 
in 
fact 
B. 
Professional 
skepticism 
C. 
Prudent 
auditor 
D. 
Reasonable 
assurance 
46. 
The 
evidence 
considered 
most 
appropriate 
by 
auditors 
is 
best 
described 
as 
A. 
Internal 
documents 
such 
as 
sales 
invoice 
copies 
produced 
under 
conditions 
of 
strong 
internal 
control. 
B. 
Written 
representations 
made 
by 
the 
president 
of 
the 
entity. 
C. 
Documentary 
evidence 
obtained 
directly 
from 
independent 
external 
sources. 
D. 
Direct 
personal 
knowledge 
obtained 
through 
physical 
observation 
and 
mathematical 
recalculation.
47. 
Auditors' 
understanding 
of 
the 
internal 
control 
in 
an 
entity 
contributes 
information 
for 
A. 
Determining 
whether 
members 
of 
the 
audit 
team 
have 
the 
required 
competence 
and 
capabilities 
to 
perform 
the 
audit. 
B. 
Ascertaining 
the 
independence 
in 
mental 
attitude 
of 
members 
of 
the 
audit 
team. 
C. 
Planning 
the 
professional 
development 
courses 
the 
audit 
staff 
needs 
to 
keep 
up 
to 
date 
with 
new 
auditing 
standards. 
D. 
Planning 
the 
nature, 
timing, 
and 
extent 
of 
further 
audit 
procedures 
on 
an 
audit. 
48. 
Which 
of 
the 
following 
elements 
of 
a 
system 
of 
quality 
control 
is 
related 
to 
firms 
receiving 
independence 
confirmations 
from 
its 
professionals 
with 
respect 
to 
clients? 
A. 
Acceptance 
and 
continuance 
of 
clients. 
B. 
Engagement 
performance. 
C. 
Monitoring. 
D. 
Relevant 
ethical 
requirements. 
49. 
Which 
of 
the 
following 
standards 
is 
not 
correctly 
associated 
with 
its 
rule-­‐making 
body? 
A. 
Public 
Company 
Accounting 
Oversight 
Board, 
Auditing 
Standards 
1-39 
B. 
Governmental 
Accounting 
Standards 
Board, 
Government 
Auditing 
Standards 
C. 
Auditing 
Standards 
Board, 
Statements 
on 
Auditing 
Standards 
D. 
International 
Auditing 
and 
Assurance 
Standards 
Board, 
International 
Statements 
on 
Auditing 
50. 
Kramer, 
CPA 
consulted 
with 
an 
independent 
appraiser 
regarding 
the 
valuation 
of 
fine 
art 
for 
a 
not-­‐for-­‐profit 
museum. 
Consultation 
with 
a 
specialist 
in 
this 
case 
would 
A. 
Be 
considered 
proper 
due 
care. 
B. 
Be 
considered 
a 
failure 
to 
follow 
GAAS 
because 
Kramer 
should 
have 
known 
how 
to 
value 
fine 
art 
before 
accepting 
the 
engagement. 
C. 
Not 
be 
considered 
a 
violation 
of 
GAAS 
because 
GAAS 
does 
not 
apply 
to 
not-­‐for-­‐profit 
entities. 
D. 
None 
of 
the 
above.
1-40 
51. 
Which 
of 
the 
following 
topics 
is 
not 
addressed 
in 
the 
auditors' 
report 
for 
a 
public 
entity? 
A. 
Responsibilities 
of 
the 
auditor 
and 
management 
in 
the 
financial 
reporting 
process. 
B. 
Absolute 
assurance 
regarding 
the 
fairness 
of 
the 
entity's 
financial 
statements 
in 
accordance 
with 
the 
applicable 
financial 
reporting 
framework 
(e.g., 
GAAP). 
C. 
A 
description 
of 
an 
audit 
engagement. 
D. 
A 
summary 
of 
the 
auditors' 
opinion 
on 
the 
effectiveness 
of 
the 
entity's 
internal 
control 
over 
financial 
reporting. 
52. 
Which 
of 
the 
following 
is 
a 
conceptual 
difference 
between 
attestation 
standards 
and 
generally 
accepted 
auditing 
standards? 
A. 
The 
attestation 
standards 
provide 
a 
framework 
for 
the 
attest 
function 
beyond 
historical 
financial 
statements. 
B. 
The 
requirement 
that 
the 
practitioner 
be 
independent 
is 
not 
required 
under 
attestation 
standards. 
C. 
The 
attestation 
standards 
do 
not 
permit 
an 
attestation 
engagement 
to 
examine 
prospective 
"what-­‐if" 
financial 
statements. 
D. 
Requirements 
related 
to 
evidence 
are 
not 
included 
in 
the 
attestation 
standards. 
Questions 
also 
found 
in 
Study 
Guide 
53. 
The 
attestation 
standards 
are 
a 
general 
set 
of 
standards 
intended 
to 
guide 
work 
in 
A. 
Audits 
of 
financial 
statements. 
B. 
Financial 
forecasts 
and 
prospective 
financial 
information. 
C. 
Areas 
other 
than 
audits 
of 
financial 
statements. 
D. 
Understanding 
internal 
control. 
54. 
Statements 
on 
Auditing 
Standards 
(SASs) 
are 
considered 
to 
be 
A. 
Specialized 
to 
obtain 
evidence 
to 
render 
an 
opinion. 
B. 
Detailed 
interpretations 
of 
the 
fundamental 
principles. 
C. 
Standards 
for 
preparation 
of 
financial 
statements. 
D. 
Standards 
to 
govern 
the 
quality 
of 
a 
specific 
firm's 
audit 
practice.
1-41 
55. 
Which 
of 
the 
following 
is 
not 
a 
subject 
related 
to 
the 
performance 
principle 
of 
GAAS? 
A. 
Risk 
of 
material 
misstatement 
B. 
Planning 
and 
supervision 
C. 
Sufficient 
appropriate 
evidence 
D. 
Due 
care 
56. 
Which 
of 
the 
following 
statements 
is 
true 
for 
attestation 
standards, 
but 
not 
for 
the 
fundamental 
principles 
of 
generally 
accepted 
auditing 
standards? 
A. 
The 
practitioner 
or 
practitioners 
must 
have 
reason 
to 
believe 
that 
the 
subject 
matter 
is 
capable 
of 
evaluation 
against 
criteria 
that 
are 
suitable 
and 
available 
to 
users. 
B. 
The 
work 
shall 
be 
adequately 
planned 
and 
assistants, 
if 
any, 
are 
to 
be 
properly 
supervised. 
C. 
Due 
care 
shall 
be 
exercised. 
D. 
A 
sufficient 
understanding 
of 
the 
internal 
control 
is 
to 
be 
obtained. 
57. 
The 
quality 
control 
of 
personnel 
management 
in 
a 
public 
accounting 
firm 
includes 
which 
of 
the 
following? 
A. 
Supervision 
appropriate 
for 
the 
competencies 
of 
the 
personnel 
assigned 
to 
the 
work 
is 
important. 
B. 
Professional 
development 
continuing 
education 
should 
be 
provided 
so 
that 
personnel 
will 
have 
the 
knowledge 
required 
to 
enable 
them 
to 
fulfill 
their 
responsibilities. 
C. 
People 
at 
all 
organizational 
levels 
must 
maintain 
independence 
in 
fact 
and 
appearance. 
D. 
When 
accepting 
and 
continuing 
client 
relationships, 
firms 
should 
consider 
their 
own 
competence. 
58. 
Which 
of 
the 
following 
is 
not 
an 
implicit 
message 
in 
the 
opinion 
paragraph 
in 
the 
auditors' 
unqualified 
opinion? 
A. 
The 
accounting 
principles 
in 
the 
financial 
statements 
have 
general 
acceptance. 
B. 
The 
accounting 
principles 
used 
by 
the 
entity 
are 
appropriate 
in 
the 
circumstances. 
C. 
The 
audit 
was 
performed 
in 
accordance 
with 
generally 
accepted 
auditing 
standards. 
D. 
The 
financial 
statements 
are 
accurate 
within 
practical 
materiality 
limits.
59. 
Auditors' 
opinions 
on 
statements 
"taken 
as 
a 
whole" 
would 
not 
include 
A. 
Disclaimers 
of 
opinion. 
B. 
Adverse 
opinions. 
C. 
Qualified 
opinions. 
D. 
Unqualified 
opinions. 
60. 
The 
opinion 
paragraph 
of 
the 
auditors' 
standard 
report 
includes 
a 
statement 
that 
A. 
The 
financial 
statements 
are 
the 
responsibility 
of 
management. 
B. 
The 
audit 
was 
conducted 
in 
accordance 
with 
generally 
accepted 
auditing 
standards. 
C. 
The 
audit 
provides 
a 
reasonable 
basis 
for 
an 
opinion. 
D. 
The 
financial 
statements 
are 
presented 
in 
conformity 
with 
generally 
accepted 
accounting 
principles. 
61. 
The 
auditors' 
standard 
report 
should 
be 
dated 
with 
the 
date 
A. 
The 
report 
was 
delivered 
to 
the 
client. 
B. 
When 
all 
significant 
procedures 
have 
been 
completed 
and 
auditors 
have 
gathered 
sufficient 
appropriate 
evidence. 
C. 
When 
the 
client's 
fiscal 
year 
ended. 
D. 
When 
the 
audit 
was 
completely 
reviewed 
by 
supervisory 
personnel. 
62. 
To 
ensure 
that 
a 
public 
accounting 
firm 
is 
providing 
services 
that 
conform 
to 
professional 
standards, 
the 
firm 
should 
follow 
A. 
The 
performance 
principle 
of 
GAAS. 
B. 
Its 
system 
of 
quality 
controls. 
C. 
Generally 
accepted 
accounting 
principles. 
D. 
International 
auditing 
standards. 
1-42
1-43 
Matching 
Questions 
63. 
For 
each 
of 
the 
matters 
below, 
indicate 
through 
the 
appropriate 
letter 
the 
fundamental 
principle 
to 
which 
the 
matter 
is 
most 
closely 
related. 
1. 
Responsibilities 
principle 
Maintaining 
professional 
skepticism. 
____ 
2. 
Responsibilities 
An 
auditors' 
overall 
conclusion 
of 
the 
fairness 
of 
the 
principle 
client's 
financial 
statements. 
____ 
3. 
Reporting 
principle 
The 
use 
of 
an 
audit 
plan 
to 
identify 
audit 
procedures 
to 
be 
performed 
during 
the 
engagement. 
____ 
4. 
Performance 
principle 
Auditors' 
assessment 
of 
the 
risk 
of 
material 
misstatement. 
____ 
5. 
Performance 
principle 
Accounting 
firm 
policies 
with 
respect 
to 
the 
level 
of 
expected 
continuing 
professional 
education. 
____ 
6. 
Performance 
principle 
Expressing 
an 
opinion 
in 
accordance 
with 
the 
auditor's 
findings. 
____ 
7. 
Reporting 
principle 
Proper 
supervision 
of 
assistants 
on 
the 
audit. 
____ 
Auditors' 
requests 
to 
obtain 
bank 
statements 
directly 
8. 
Performance 
from 
financial 
institutions 
with 
whom 
the 
client 
does 
principle 
business. 
____ 
9. 
Reporting 
principle 
An 
expression 
that 
an 
opinion 
cannot 
be 
expressed. 
____ 
10. 
Performance 
Determining 
and 
applying 
an 
appropriate 
materiality 
principle 
level. 
____ 
True 
/ 
False 
Questions 
Question 
also 
Found 
in 
Study 
Guide 
64. 
Auditors 
may 
be 
independent 
in 
fact 
but 
not 
independent 
in 
appearance. 
True 
False 
65. 
Standards 
for 
accountants 
in 
public 
practice 
are 
limited 
to 
auditing 
services. 
True 
False
66. 
The 
attestation 
standards 
provide 
guidance 
for 
a 
wide 
variety 
of 
attestation 
engagements. 
True 
False 
67. 
The 
AICPA's 
Generally 
Accepted 
Auditing 
Standards 
must 
be 
followed 
on 
all 
audit 
engagements. 
True 
False 
68. 
The 
reporting 
principle 
relates 
to 
a 
firm's 
system 
of 
quality 
control 
criteria 
for 
conducting 
an 
audit. 
True 
False 
69. 
Auditors 
cannot 
effectively 
satisfy 
the 
responsibilities 
principle 
requiring 
due 
care 
if 
they 
have 
not 
also 
satisfied 
the 
performance 
principle. 
True 
False 
70. 
Auditing 
procedures 
are 
quality 
guides 
that 
are 
less 
specific 
than 
auditing 
standards. 
True 
False 
71. 
Auditing 
procedures 
are 
the 
same 
as 
auditing 
standards. 
True 
False 
72. 
The 
concept 
of 
due 
care 
requires 
auditors 
to 
observe 
the 
performance, 
responsibilities 
and 
reporting 
principles. 
True 
False 
73. 
Attestation 
standards 
require 
the 
practitioner 
to 
obtain 
a 
sufficient 
understanding 
of 
the 
client's 
internal 
control. 
True 
False 
1-44
74. 
The 
performance 
principle 
sets 
forth 
the 
quality 
criteria 
for 
conducting 
an 
audit. 
True 
False 
75. 
Auditors 
of 
entities 
registered 
with 
the 
Securities 
and 
Exchange 
Commission 
are 
required 
to 
register 
with 
the 
Public 
Company 
Accounting 
Oversight 
Board 
(PCAOB). 
True 
False 
76. 
Control 
risk 
is 
the 
probability 
that 
a 
material 
misstatement 
(error 
or 
fraud) 
could 
occur 
and 
not 
be 
prevented 
or 
detected 
on 
a 
timely 
basis 
by 
the 
entity's 
external 
auditors. 
True 
False 
77. 
Evidence 
that 
is 
considered 
"appropriate" 
in 
auditing 
means 
that 
all 
underlying 
accounting 
data 
and 
corroborating 
information 
must 
be 
absolutely 
compelling 
to 
auditors. 
True 
False 
78. 
Even 
in 
the 
audit 
of 
historical 
cost 
financial 
statements, 
auditors 
may 
have 
to 
make 
inferences 
about 
the 
future. 
True 
False 
79. 
The 
contents 
of 
the 
auditors' 
report 
are 
guided 
exclusively 
by 
the 
reporting 
principle 
of 
GAAS. 
True 
False 
80. 
The 
auditors' 
standard 
report 
should 
always 
make 
direct 
reference 
to 
consistency 
and 
disclosure. 
True 
False 
1-45
81. 
The 
auditors' 
standard 
report 
should 
either 
contain 
an 
expression 
of 
opinion 
on 
the 
financial 
statements 
taken 
as 
a 
whole 
or 
an 
assertion 
to 
the 
effect 
that 
an 
opinion 
cannot 
be 
expressed. 
True 
False 
82. 
Evidence 
is 
considered 
appropriate 
when 
it 
is 
both 
valid 
and 
relevant. 
True 
False 
83. 
The 
statement 
on 
quality 
control 
standards 
No. 
7 
notes 
that 
the 
purpose 
of 
a 
system 
of 
quality 
control 
is 
to 
provide 
reasonable 
assurance 
that 
the 
firm 
and 
its 
personnel 
issue 
reports 
that 
are 
appropriate 
under 
the 
circumstances. 
True 
False 
1-46 
Fill 
in 
the 
Blank 
Questions 
Question 
also 
found 
in 
Study 
Guide 
84. 
The 
_____________________________ 
standards 
are 
a 
general 
set 
of 
standards 
to 
guide 
attestation 
engagements 
in 
areas 
other 
than 
audits 
of 
financial 
statements. 
________________________________________ 
85. 
Audits 
of 
historical 
financial 
statements 
are 
guided 
by 
a 
broad 
set 
of 
principles 
referred 
to 
as 
_______________________ 
_________________________ 
_____________________________ 
_____________________________. 
________________________________________
86. 
Attestation 
reporting 
is 
different 
because 
attestation 
engagements 
related 
to 
nonfinancial 
information 
do 
not 
require 
information 
to 
be 
presented 
in 
accordance 
with 
_____________________________ 
_____________________________ 
_____________________________ 
_____________________________. 
________________________________________ 
87. 
The 
AICPA's 
fundamental 
principles 
of 
generally 
accepted 
auditing 
standards 
are 
classified 
in 
three 
categories: 
_______________________ 
principle, 
_______________________ 
principle, 
and 
the 
_____________________________ 
principle. 
________________________________________ 
88. 
A(n) 
_____________________________ 
_____________________________ 
is 
a 
list 
of 
auditing 
procedures 
that 
will 
be 
performed 
during 
the 
engagement 
to 
gather 
sufficient 
appropriate 
evidence. 
________________________________________ 
89. 
The 
responsibilities 
principle 
of 
GAAS 
highlights 
the 
importance 
of 
complying 
with 
ethical 
requirements, 
including 
those 
pertaining 
to 
__________________________ 
and 
_____________________________. 
________________________________________ 
90. 
The 
three 
aspects 
of 
practical 
independence 
are 
_____________________________ 
independence, 
_____________________________ 
independence, 
and 
_____________________________ 
independence. 
________________________________________ 
91. 
The 
concept 
of 
_____________________________ 
relates 
to 
financial 
statement 
users' 
perceptions 
of 
auditors' 
independence. 
________________________________________ 
1-47
92. 
_____________________________ 
________________________ 
reflects 
a 
level 
of 
performance 
that 
would 
be 
exercised 
by 
reasonable 
auditors 
in 
similar 
circumstances. 
________________________________________ 
93. 
Since 
audit 
samples 
are 
used, 
audit 
evidence 
is 
considered 
to 
be 
_____________________________, 
rather 
than 
_____________________________. 
________________________________________ 
94. 
The 
auditors' 
report 
must 
state 
whether 
the 
financial 
statements 
are 
presented 
in 
accordance 
with 
__________________________ 
_____________________________ 
_____________________________ 
_____________________________. 
________________________________________ 
95. 
Under 
the 
reporting 
principle 
of 
GAAS, 
the 
auditor 
expresses 
an 
opinion 
in 
accordance 
with 
the 
___________________ 
______________. 
________________________________________ 
96. 
Under 
the 
reporting 
principle 
of 
GAAS, 
the 
report 
will 
contain 
either 
an 
expression 
of 
_____________________________ 
regarding 
the 
financial 
statements, 
taken 
as 
a 
whole, 
or 
an 
assertion 
to 
the 
effect 
that 
an 
opinion 
cannot 
be 
expressed. 
________________________________________ 
97. 
An 
overall 
opinion 
that 
the 
financial 
statements 
present 
the 
financial 
condition, 
results 
of 
operations, 
and 
cash 
flows 
according 
to 
generally 
accepted 
accounting 
principles 
is 
a(n) 
_____________________________ 
opinion. 
________________________________________ 
98. 
If 
a 
material 
departure 
from 
GAAP 
is 
noted, 
auditors 
can 
choose 
between 
a(n) 
__________________________ 
opinion 
or 
a(n) 
___________________________ 
opinion. 
________________________________________ 
1-48
99. 
Auditors' 
indication 
that 
no 
opinion 
is 
given 
is 
referred 
to 
as 
a(n) 
_______________________ 
___________________________________________ 
________________________________________ 
100. 
The 
_____________________________ 
paragraph 
of 
the 
auditors' 
report 
declares 
that 
the 
audit 
was 
conducted 
in 
accordance 
with 
generally 
accepted 
_________________________________________________________ 
________________________________________ 
101. 
A(n) 
_____________________________ 
_____________________________ 
is 
a 
study 
of 
an 
accounting 
firm's 
quality 
control 
policies 
and 
procedures, 
followed 
by 
a 
report 
on 
the 
firm's 
quality 
of 
audit 
practice 
in 
accordance 
with 
the 
system 
of 
quality 
controls. 
________________________________________ 
102. 
The 
_____________________________ 
_____________________________ 
Act 
of 
2002 
created 
the 
Public 
Company 
Accounting 
Oversight 
Board 
(PCAOB). 
________________________________________ 
103. 
The 
PCAOB 
has 
two 
primary 
roles: 
_____________________________ 
and 
_____________________________. 
________________________________________ 
1-49
1-50 
Essay 
Questions 
104. 
Distinguish 
between 
attestation 
standards 
and 
the 
fundamental 
principles 
of 
generally 
accepted 
auditing 
standards 
by 
identifying 
and 
describing 
major 
differences 
between 
the 
two 
sets 
of 
standards.
105. 
Alan 
Fallon 
was 
recently 
promoted 
to 
senior 
accountant. 
He 
was 
put 
in 
charge 
of 
the 
Mellow 
Markets 
audit 
because 
of 
his 
experience 
with 
other 
grocery 
clients. 
Mellow 
Markets 
has 
a 
small, 
but 
growing, 
chain 
of 
natural 
food 
stores. 
This 
is 
the 
first 
year 
Mellow 
Markets 
has 
been 
audited. 
Because 
of 
their 
growth, 
Mellow 
Markets 
needs 
additional 
capital 
and 
intend 
to 
use 
their 
audited 
financial 
statements 
to 
secure 
a 
loan. 
Alan 
has 
been 
assigned 
two 
inexperienced 
staff 
assistants 
for 
the 
audit. 
Because 
this 
is 
his 
first 
engagement 
as 
a 
senior, 
he 
intends 
to 
bring 
the 
job 
in 
on 
budget. 
To 
save 
time, 
he 
provided 
his 
assistants 
with 
a 
copy 
of 
the 
audit 
plan 
for 
Happy 
Time 
Food 
Stores. 
He 
told 
them 
that 
this 
would 
make 
things 
go 
more 
quickly. 
He 
also 
told 
them 
that 
he 
could 
not 
spend 
much 
time 
with 
them 
at 
the 
client's 
place 
of 
business, 
because 
"my 
time 
is 
billed 
out 
at 
such 
a 
high 
rate, 
we'll 
go 
right 
over 
budget." 
However, 
he 
did 
call 
them 
once 
a 
day 
from 
another 
audit 
on 
which 
he 
was 
working. 
After 
beginning 
their 
work, 
the 
assistants 
told 
Alan 
that 
the 
audit 
plan 
did 
not 
always 
match 
up 
with 
what 
they 
found 
at 
Mellow 
Markets. 
Alan 
responded, 
"just 
cross 
out 
whatever 
is 
not 
relevant 
in 
the 
audit 
plan 
and 
don't 
add 
anything 
-­‐ 
it 
will 
only 
make 
us 
go 
over 
the 
budget." 
When 
Alan 
came 
to 
the 
client 
near 
the 
end 
of 
field 
Work, 
one 
assistant 
was 
concerned 
that 
no 
inventory 
observation 
was 
done 
at 
the 
out-­‐of-­‐town 
locations 
of 
Mellow 
Markets 
(the 
audit 
plan 
had 
stipulated 
that 
inventory 
should 
be 
observed 
for 
in-­‐town 
stores 
only). 
Happy 
Time 
had 
only 
one 
out-­‐of-­‐town 
location, 
while 
three 
of 
Mellow 
Markets' 
five 
stores 
were 
in 
other 
cities. 
Alan 
told 
the 
assistant 
to 
get 
inventory 
sheets 
from 
the 
client 
for 
the 
other 
stores 
and 
added 
"make 
sure 
that 
the 
inventory 
balance 
in 
the 
general 
ledger 
agrees 
with 
the 
total 
for 
all 
the 
inventory 
sheets." 
The 
next 
day, 
Alan 
reviewed 
all 
audit 
documentation 
and 
submitted 
the 
job 
for 
review 
by 
the 
manager. 
Required: 
1. 
Describe 
the 
performance 
principle 
of 
GAAS. 
2. 
Do 
you 
believe 
that 
the 
Mellow 
Markets 
audit 
complies 
with 
these 
standards? 
Explain. 
1-51 
Matching 
Questions 
Question 
also 
Found 
in 
Study 
Guide
106. 
Using 
I 
(introductory), 
S 
(scope), 
O 
(opinion), 
A 
(additional), 
or 
N 
(none), 
indicate 
the 
paragraph 
in 
which 
the 
following 
statements 
or 
topics 
would 
be 
included 
in 
the 
auditors' 
report. 
1-52 
1. 
none 
The 
titles 
of 
the 
financial 
statements 
examined 
by 
the 
auditors. 
____ 
2. 
none 
A 
description 
of 
any 
scope 
limitation(s) 
encountered 
during 
the 
audit. 
____ 
3. 
introductory 
A 
statement 
that 
auditors 
were 
independent 
with 
respect 
to 
the 
entity. 
____ 
4. 
additional 
The 
auditors' 
conclusion 
with 
respect 
to 
the 
fairness 
of 
the 
entity's 
financial 
statements. 
____ 
5. 
opinion 
A 
statement 
that 
an 
audit 
was 
conducted 
in 
accordance 
with 
generally 
accepted 
auditing 
standards. 
____ 
6. 
opinion 
A 
statement 
that 
the 
entity's 
management 
is 
responsible 
for 
the 
fairness 
of 
the 
financial 
statements. 
____ 
7. 
introductory 
A 
description 
of 
an 
audit, 
which 
includes 
examining 
evidence 
in 
support 
of 
the 
financial 
statements. 
____ 
8. 
scope 
Reference 
to 
generally 
accepted 
accounting 
principles. 
____ 
A 
description 
of 
any 
specific 
departures 
from 
GAAP 
noted 
9. 
scope 
during 
the 
audit 
that 
were 
material. 
____ 
10. 
additional 
A 
statement 
that 
the 
financial 
statements 
were 
consistently 
prepared 
compared 
to 
those 
of 
prior 
period(s). 
____
Chapter 
02 
Professional 
Standards 
Answer 
Key 
1-53 
Multiple 
Choice 
Questions 
1. 
The 
attestation 
standards 
of 
reporting 
do 
not 
require 
the 
attestation 
report 
to 
include 
a 
statement 
that 
A. 
Provides 
a 
conclusion 
whether 
the 
subject 
matter 
is 
presented 
in 
conformity 
with 
established 
or 
stated 
criteria. 
B. 
Indicates 
that 
the 
practitioner 
has 
significant 
reservations 
about 
the 
engagement. 
C. 
Identifies 
the 
subject 
matter 
or 
assertion 
being 
reported 
on. 
D. 
Indicates 
that 
the 
accountant 
assumes 
no 
responsibility 
to 
update 
the 
report. 
Original 
AACSB: 
Analytic 
AICPA 
BB: 
Legal 
AICPA 
FN: 
Research 
Bloom's: 
Knowledge 
Difficulty: 
Easy 
2. 
Control 
risk 
is 
A. 
The 
probability 
that 
a 
material 
misstatement 
could 
not 
be 
prevented 
or 
detected 
by 
the 
entity's 
internal 
control 
policies 
and 
procedures. 
B. 
The 
probability 
that 
a 
material 
misstatement 
could 
occur 
and 
not 
be 
detected 
by 
auditors' 
procedures. 
C. 
The 
risk 
that 
auditors 
will 
not 
be 
able 
to 
complete 
the 
audit 
on 
a 
timely 
basis. 
D. 
The 
risk 
that 
auditors 
will 
not 
properly 
control 
the 
staff 
on 
the 
audit 
engagement. 
Original 
AACSB: 
Analytic 
AICPA 
BB: 
Legal 
AICPA 
FN: 
Risk 
Analysis 
Bloom's: 
Knowledge 
Difficulty: 
Easy
3. 
The 
responsibilities 
principle 
under 
generally 
accepted 
auditing 
standards 
does 
1-54 
not 
include 
which 
of 
the 
following? 
A. 
Competence 
and 
capabilities. 
B. 
Independent 
attitude. 
C. 
Due 
care. 
D. 
Planning 
and 
supervision. 
Original 
AACSB: 
Analytic 
AICPA 
BB: 
Legal 
AICPA 
FN: 
Research 
Bloom's: 
Knowledge 
Difficulty: 
Easy 
4. 
Which 
of 
the 
following 
types 
of 
auditors' 
reports 
does 
not 
require 
an 
explanatory 
paragraph 
to 
support 
the 
opinion? 
A. 
Unqualified 
opinion. 
B. 
Adverse 
opinion. 
C. 
Qualified 
opinion. 
D. 
Disclaimer 
of 
opinion. 
Original 
AACSB: 
Communication 
AICPA 
BB: 
Legal 
AICPA 
FN: 
Reporting 
Bloom's: 
Knowledge 
Difficulty: 
Easy
5. 
Which 
of 
the 
following 
is 
an 
element 
of 
a 
system 
of 
quality 
control 
that 
should 
be 
considered 
by 
a 
public 
accounting 
firm 
in 
establishing 
its 
quality 
control 
policies 
and 
procedures? 
A. 
Lending 
credibility 
to 
a 
client's 
financial 
statements. 
B. 
Using 
statistical 
sampling 
techniques. 
C. 
Acceptance 
and 
continuance 
of 
clients. 
D. 
Membership 
in 
the 
Center 
for 
Public 
Company 
Audit 
Firms 
(CPCAF). 
1-55 
Original 
AACSB: 
Analytic 
AICPA 
BB: 
Legal 
AICPA 
FN: 
Research 
Bloom's: 
Knowledge 
Difficulty: 
Medium 
6. 
Which 
of 
the 
following 
presumptions 
does 
not 
relate 
to 
the 
reliability 
of 
audit 
evidence? 
A. 
The 
more 
effective 
the 
client's 
internal 
control, 
the 
more 
assurance 
it 
provides 
about 
the 
accounting 
data 
and 
financial 
statements. 
B. 
The 
auditors' 
opinion, 
to 
be 
economically 
useful, 
is 
formed 
within 
reasonable 
time 
and 
based 
on 
evidence 
obtained 
at 
a 
reasonable 
cost. 
C. 
Evidence 
obtained 
from 
independent 
sources 
outside 
the 
entity 
is 
more 
reliable 
than 
evidence 
secured 
solely 
within 
the 
entity. 
D. 
The 
independent 
auditors' 
direct 
personal 
knowledge, 
obtained 
through 
observation 
and 
inspection, 
is 
more 
persuasive 
than 
information 
obtained 
indirectly. 
AICPA 
AACSB: 
Analytic 
AICPA 
BB: 
Legal 
AICPA 
FN: 
Research 
Bloom's: 
Comprehension 
Difficulty: 
Hard
7. 
An 
important 
role 
of 
the 
Public 
Company 
Accounting 
Oversight 
Board 
(PCAOB) 
is 
to 
oversee 
the 
A. 
Issuance 
of 
statements 
by 
the 
Financial 
Accounting 
Standards 
Board. 
B. 
Preparation 
and 
grading 
of 
the 
Uniform 
CPA 
Examination. 
C. 
Peer 
review 
of 
member 
firms 
of 
the 
Private 
Companies 
Practice 
Section. 
D. 
Regulation 
of 
firms 
that 
audit 
public 
companies. 
1-56 
Original 
AACSB: 
Analytic 
AICPA 
BB: 
Legal 
AICPA 
FN: 
Research 
Bloom's: 
Knowledge 
Difficulty: 
Easy 
8. 
Audit 
evidence 
is 
usually 
considered 
sufficient 
when 
A. 
It 
is 
reliable. 
B. 
There 
is 
enough 
quantity 
to 
afford 
a 
reasonable 
basis 
for 
an 
opinion 
on 
financial 
statements. 
C. 
It 
has 
the 
qualities 
of 
being 
relevant, 
objective, 
and 
free 
from 
unknown 
bias. 
D. 
It 
has 
been 
obtained 
through 
random 
selection 
methods. 
AICPA 
AACSB: 
Analytic 
AICPA 
BB: 
Legal 
AICPA 
FN: 
Research 
Bloom's: 
Knowledge 
Difficulty: 
Medium
1-57 
9. 
Which 
of 
the 
following 
is 
not 
considered 
a 
type 
of 
audit 
evidence? 
A. 
The 
company's 
trial 
balance. 
B. 
Auditors' 
calculations. 
C. 
Physical 
observation. 
D. 
Verbal 
statements 
made 
by 
client 
personnel. 
Original 
AACSB: 
Analytic 
AICPA 
BB: 
Legal 
AICPA 
FN: 
Research 
Bloom's: 
Knowledge 
Difficulty: 
Medium 
10. 
The 
AICPA 
attestation 
standards 
differ 
from 
the 
AICPA 
responsibilities 
principle, 
performance 
principle 
and 
reporting 
principle 
in 
that: 
A. 
The 
attestation 
standards 
contain 
no 
requirement 
to 
obtain 
an 
understanding 
of 
the 
entity 
and 
assess 
the 
risk 
of 
material 
misstatement. 
B. 
The 
attestation 
standards 
do 
not 
require 
competence 
and 
capabilities. 
C. 
The 
attestation 
standards 
do 
not 
require 
planning 
for 
attestation 
engagements 
or 
supervision 
of 
accountants 
and 
consultants 
who 
perform 
the 
work. 
D. 
The 
attestation 
standards 
do 
not 
require 
a 
report 
that 
states 
the 
character 
of 
the 
engagement. 
Original 
AACSB: 
Analytic 
AICPA 
BB: 
Legal 
AICPA 
FN: 
Research 
Bloom's: 
Knowledge 
Difficulty: 
Hard
11. 
An 
audit 
of 
the 
financial 
statements 
of 
Camden 
Corporation 
is 
being 
conducted 
by 
external 
auditors. 
The 
external 
auditors 
are 
expected 
to: 
A. 
Certify 
the 
correctness 
of 
Camden's 
financial 
statements. 
B. 
Make 
a 
complete 
examination 
of 
Camden's 
records 
and 
verify 
all 
of 
Camden's 
transactions. 
C. 
Give 
an 
opinion 
on 
the 
fair 
presentation 
of 
Camden's 
financial 
statements 
in 
conformity 
with 
the 
applicable 
financial 
reporting 
framework 
(e.g., 
GAAP, 
IFRS). 
D. 
Give 
an 
opinion 
on 
the 
attractiveness 
of 
Camden 
for 
investment 
purposes 
and 
critique 
the 
wisdom 
and 
legality 
of 
its 
business 
decisions. 
1-58 
Original 
AACSB: 
Communication 
AICPA 
BB: 
Legal 
AICPA 
FN: 
Reporting 
Bloom's: 
Knowledge 
Difficulty: 
Easy 
12. 
Auditors 
try 
to 
achieve 
independence 
in 
appearance 
in 
order 
to: 
A. 
Maintain 
public 
confidence 
in 
the 
profession. 
B. 
Become 
independent 
in 
fact. 
C. 
Comply 
with 
the 
responsibilities 
principle. 
D. 
Maintain 
an 
unbiased 
mental 
attitude. 
Original 
AACSB: 
Ethics 
AICPA 
BB: 
Legal 
AICPA 
FN: 
Research 
Bloom's: 
Knowledge 
Difficulty: 
Medium
13. 
The 
independent 
auditors' 
plan 
prepared 
prior 
to 
the 
start 
of 
field 
work 
is 
appropriately 
considered 
documentation 
of 
A. 
Planning. 
B. 
Supervision. 
C. 
Information 
evaluation. 
D. 
Quality 
assurance. 
1-59 
Original 
AACSB: 
Analytic 
AICPA 
BB: 
Legal 
AICPA 
FN: 
Research 
Bloom's: 
Knowledge 
Difficulty: 
Easy 
14. 
Which 
of 
the 
following 
procedures 
would 
provide 
the 
most 
reliable 
audit 
evidence? 
A. 
Inquiries 
of 
the 
client's 
accounting 
staff 
held 
in 
private. 
B. 
Inspection 
of 
pre-­‐numbered 
client 
shipping 
documents. 
C. 
Inspection 
of 
bank 
statements 
obtained 
directly 
from 
the 
client's 
financial 
institution. 
D. 
Analytical 
procedures 
performed 
by 
auditors 
on 
the 
client's 
trial 
balance. 
AICPA 
AACSB: 
Analytic 
AICPA 
BB: 
Legal 
AICPA 
FN: 
Decision 
Making 
Bloom's: 
Knowledge 
Difficulty: 
Hard
15. 
Which 
of 
the 
following 
is 
not 
an 
attestation 
standard? 
A. 
The 
practitioner 
must 
obtain 
sufficient 
evidence 
to 
provide 
a 
reasonable 
basis 
for 
the 
conclusion 
expressed 
in 
the 
report. 
B. 
The 
practitioner 
must 
identify 
the 
subject 
matter 
or 
the 
assertion 
being 
reported 
on 
and 
state 
the 
character 
of 
the 
engagement. 
C. 
The 
practitioner 
must 
adequately 
plan 
the 
work 
and 
must 
properly 
supervise 
any 
assistants. 
D. 
A 
sufficient 
understanding 
of 
the 
client's 
internal 
controls 
shall 
be 
obtained 
to 
plan 
the 
engagement. 
1-60 
AICPA 
AACSB: 
Analytic 
AICPA 
BB: 
Legal 
AICPA 
FN: 
Research 
Bloom's: 
Knowledge 
Difficulty: 
Medium 
16. 
Which 
of 
the 
following 
would 
most 
likely 
be 
a 
violation 
of 
the 
independence 
requirement 
found 
in 
the 
responsibilities 
principle 
under 
generally 
accepted 
auditing 
standards? 
A. 
An 
auditor 
on 
the 
engagement 
has 
a 
distant 
relative 
who 
is 
employed 
by 
a 
vendor 
that 
does 
a 
significant 
amount 
of 
business 
with 
clients. 
B. 
The 
client's 
Chief 
Executive 
Officer 
graduated 
from 
the 
same 
university 
as 
the 
partner 
in 
charge 
of 
the 
accounting 
firm. 
C. 
An 
auditor 
on 
the 
engagement 
owns 
a 
financial 
interest 
in 
the 
stock 
of 
the 
client. 
D. 
The 
client 
provides 
financial 
support 
to 
a 
number 
of 
charitable 
causes 
that 
also 
receive 
support 
from 
the 
accounting 
firm. 
Original 
AACSB: 
Ethics 
AICPA 
BB: 
Legal 
AICPA 
FN: 
Research 
Bloom's: 
Knowledge 
Difficulty: 
Easy
17. 
A 
vendor's 
invoice 
received 
and 
held 
by 
the 
client 
would 
be 
considered 
what 
type 
of 
evidence? 
A. 
External. 
B. 
Internal. 
C. 
External-­‐internal. 
D. 
Written 
representation. 
1-61 
Original 
AACSB: 
Analytic 
AICPA 
BB: 
Legal 
AICPA 
FN: 
Research 
Bloom's: 
Knowledge 
Difficulty: 
Easy 
18. 
Which 
of 
the 
following 
statements 
is 
generally 
correct 
about 
the 
appropriateness 
of 
audit 
evidence? 
A. 
Auditors' 
direct 
personal 
knowledge, 
obtained 
through 
observation 
and 
inspection, 
is 
more 
persuasive 
than 
information 
obtained 
indirectly 
from 
independent 
outside 
sources. 
B. 
To 
be 
reliable, 
audit 
evidence 
must 
be 
either 
valid 
or 
relevant, 
but 
need 
not 
be 
both. 
C. 
Client 
accounting 
data 
alone 
may 
be 
considered 
sufficient 
appropriate 
audit 
evidence 
to 
issue 
an 
unqualified 
opinion 
on 
client 
financial 
statements. 
D. 
Appropriateness 
of 
audit 
evidence 
refers 
to 
the 
amount 
of 
corroborative 
evidence 
to 
be 
obtained. 
AICPA 
AACSB: 
Analytic 
AICPA 
BB: 
Legal 
AICPA 
FN: 
Decision 
Making 
Bloom's: 
Comprehension 
Difficulty: 
Medium
19. 
The 
standard 
auditors' 
report 
refers 
to 
GAAS 
and 
GAAP 
in 
which 
paragraph? 
A. 
GAAS: 
Scope 
only; 
GAAP: 
Opinion 
only 
B. 
GAAS: 
Introductory 
only; 
GAAP: 
Scope 
and 
opinion 
C. 
GAAS: 
Introductory 
and 
scope; 
GAAP: 
Opinion 
only 
D. 
GAAS: 
Introductory 
only; 
GAAP: 
All 
paragraphs 
1-62 
AICPA 
AACSB: 
Communication 
AICPA 
BB: 
Legal 
AICPA 
FN: 
Reporting 
Bloom's: 
Knowledge 
Difficulty: 
Medium 
20. 
Which 
of 
the 
following 
is 
not 
included 
in 
the 
auditors' 
standard 
report 
representing 
an 
unqualified 
opinion? 
A. 
A 
brief 
indication 
of 
the 
responsibility 
of 
auditors 
and 
management 
for 
the 
financial 
statements. 
B. 
An 
indication 
that 
all 
appropriate 
disclosures 
have 
been 
made 
and 
included 
in 
the 
financial 
statements. 
C. 
An 
indication 
that 
the 
audit 
was 
conducted 
in 
accordance 
with 
standards 
established 
by 
the 
PCAOB. 
D. 
The 
auditors' 
opinion 
on 
the 
fairness 
of 
the 
financial 
statements. 
Original 
AACSB: 
Communication 
AICPA 
BB: 
Legal 
AICPA 
FN: 
Reporting 
Bloom's: 
Knowledge 
Difficulty: 
Medium
21. 
Internal 
evidence 
A. 
Is 
obtained 
directly 
from 
third 
parties 
independent 
of 
the 
client. 
B. 
Originates 
outside 
of 
the 
client's 
system 
but 
has 
been 
received 
and 
processed 
by 
the 
client. 
C. 
Consists 
of 
documents 
that 
are 
produced, 
used, 
and 
stored 
within 
the 
client's 
information 
system. 
D. 
Consists 
of 
representations 
made 
by 
the 
client's 
officers, 
directors, 
owners, 
and 
employees. 
1-63 
Original 
AACSB: 
Analytic 
AICPA 
BB: 
Legal 
AICPA 
FN: 
Research 
Bloom's: 
Knowledge 
Difficulty: 
Medium 
22. 
Which 
of 
the 
following 
presumptions 
is 
correct 
about 
the 
reliability 
of 
audit 
evidence? 
A. 
Information 
obtained 
indirectly 
from 
outside 
sources 
is 
the 
most 
reliable 
form 
of 
audit 
evidence. 
B. 
To 
be 
reliable, 
audit 
evidence 
should 
be 
convincing 
rather 
than 
persuasive. 
C. 
Reliability 
of 
audit 
evidence 
refers 
to 
the 
amount 
of 
corroborative 
evidence 
obtained. 
D. 
An 
effective 
system 
of 
internal 
control 
provides 
more 
assurance 
about 
the 
reliability 
of 
audit 
evidence. 
AICPA 
AACSB: 
Analytic 
AICPA 
BB: 
Legal 
AICPA 
FN: 
Decision 
Making 
Bloom's: 
Comprehension 
Difficulty: 
Hard
23. 
When 
auditors 
do 
not 
mention 
consistency 
in 
the 
auditors' 
report, 
a 
reader 
of 
the 
financial 
statements 
may 
infer 
A. 
That 
the 
applicable 
financial 
reporting 
framework 
(e.g., 
GAAP) 
has 
been 
consistently 
observed 
in 
the 
current 
period 
in 
relation 
to 
the 
preceding 
period. 
B. 
That 
no 
material 
departure 
from 
the 
applicable 
financial 
reporting 
framework 
(e.g., 
GAAP) 
has 
been 
detected. 
C. 
That 
no 
reclassification 
of 
items 
or 
change 
in 
classifications 
has 
occurred. 
D. 
Nothing 
about 
application 
of 
accounting 
principles 
within 
the 
period. 
1-64 
AICPA 
AACSB: 
Communication 
AICPA 
BB: 
Legal 
AICPA 
FN: 
Reporting 
Bloom's: 
Knowledge 
Difficulty: 
Hard 
24. 
The 
auditors' 
responsibility 
to 
express 
an 
opinion 
on 
the 
financial 
statements 
is 
A. 
Implicitly 
represented 
in 
the 
auditors' 
standard 
report. 
B. 
Explicitly 
represented 
in 
the 
introductory 
paragraph 
of 
the 
auditors' 
standard 
report. 
C. 
Explicitly 
represented 
in 
the 
scope 
paragraph 
of 
the 
auditors' 
standard 
report. 
D. 
Explicitly 
represented 
in 
the 
opinion 
paragraph 
of 
the 
auditors' 
standard 
report. 
AICPA 
AACSB: 
Communication 
AICPA 
BB: 
Legal 
AICPA 
FN: 
Reporting 
Bloom's: 
Knowledge 
Difficulty: 
Hard
25. 
Which 
of 
the 
following 
is 
not 
a 
concept 
from 
the 
performance 
principle 
under 
generally 
accepted 
auditing 
standards? 
A. 
The 
auditor 
must 
plan 
the 
work 
and 
properly 
supervise 
any 
assistants. 
B. 
The 
auditor 
must 
express 
an 
opinion 
in 
accordance 
with 
the 
auditor's 
findings. 
C. 
The 
auditor 
must 
obtain 
sufficient 
appropriate 
evidence 
about 
whether 
material 
misstatements 
exist. 
D. 
The 
auditor 
must 
determine 
and 
apply 
an 
appropriate 
materiality 
level 
throughout 
the 
audit. 
1-65 
Original 
AACSB: 
Analytic 
AICPA 
BB: 
Legal 
AICPA 
FN: 
Research 
Bloom's: 
Knowledge 
Difficulty: 
Medium 
26. 
Under 
generally 
accepted 
auditing 
standards, 
which 
of 
the 
following 
reflects 
a 
concept 
related 
to 
the 
responsibilities 
principle? 
A. 
The 
initial 
planning 
of 
the 
audit 
engagement 
should 
occur 
with 
the 
audit 
partner, 
manager, 
senior, 
and 
client 
personnel. 
B. 
The 
confirmation 
of 
accounts 
receivable 
should 
occur 
on 
each 
audit. 
C. 
The 
completion 
of 
an 
internal 
control 
questionnaire. 
D. 
Maintaining 
professional 
skepticism 
and 
exercising 
professional 
judgment. 
Original 
AACSB: 
Analytic 
AICPA 
BB: 
Legal 
AICPA 
FN: 
Research 
Bloom's: 
Knowledge 
Difficulty: 
Medium
27. 
Which 
of 
the 
following 
represent 
audit 
quality 
guides 
that 
remain 
stable 
over 
time 
and 
are 
applicable 
for 
all 
audits? 
A. 
Auditing 
procedures. 
B. 
Auditing 
standards. 
C. 
Due 
care. 
D. 
System 
of 
quality 
control. 
1-66 
Original 
AACSB: 
Analytic 
AICPA 
BB: 
Legal 
AICPA 
FN: 
Research 
Bloom's: 
Knowledge 
Difficulty: 
Easy 
28. 
Which 
of 
the 
following 
situations 
would 
most 
likely 
be 
in 
conflict 
with 
the 
responsibilities 
principle? 
A. 
Auditors 
perform 
the 
engagement 
with 
prudent 
auditors, 
but 
not 
expert 
auditors. 
B. 
Auditors 
obtain 
expertise 
in 
their 
client's 
industry 
as 
they 
are 
conducting 
the 
audit 
examination. 
C. 
Auditors 
are 
directly 
involved 
with 
a 
client 
manager 
in 
a 
strategic 
decision-­‐making 
capacity. 
D. 
Auditors 
fail 
to 
document 
their 
assessment 
of 
control 
risk 
following 
their 
study 
of 
internal 
control. 
Original 
AACSB: 
Analytic 
AICPA 
BB: 
Legal 
AICPA 
FN: 
Research 
Bloom's: 
Application 
Difficulty: 
Medium
1-67 
29. 
Which 
of 
the 
following 
statements 
is 
not 
true 
with 
respect 
to 
the 
evidence 
that 
would 
be 
gathered 
when 
assessments 
of 
control 
risk 
are 
high? 
A. 
Auditors 
would 
be 
required 
to 
rely 
on 
external 
(rather 
than 
internal) 
forms 
of 
evidence. 
B. 
Auditors 
would 
be 
required 
to 
perform 
procedures 
at 
interim 
periods, 
rather 
than 
at 
year 
end. 
C. 
Auditors 
would 
be 
required 
to 
confirm 
a 
larger 
number 
of 
customer 
accounts 
receivable 
balances. 
D. 
Auditors 
would 
be 
required 
to 
obtain 
more 
evidence 
through 
direct 
personal 
observation. 
Original 
AACSB: 
Analytic 
AICPA 
BB: 
Critical 
Thinking 
AICPA 
FN: 
Decision 
Making 
Bloom's: 
Application 
Difficulty: 
Hard 
30. 
As 
it 
relates 
to 
audit 
evidence, 
appropriateness 
refers 
to 
the 
A. 
Originality 
of 
evidence 
gathered. 
B. 
Quality 
of 
evidence 
gathered. 
C. 
Quantity 
of 
evidence 
gathered. 
D. 
Timeliness 
of 
evidence 
gathered. 
Original 
AACSB: 
Analytic 
AICPA 
BB: 
Legal 
AICPA 
FN: 
Research 
Bloom's: 
Knowledge 
Difficulty: 
Easy
31. 
Which 
of 
the 
following 
information 
would 
not 
be 
included 
in 
the 
auditors' 
standard 
report? 
A. 
The 
names 
of 
the 
financial 
statements 
audited. 
B. 
A 
description 
of 
the 
nature 
of 
an 
audit. 
C. 
An 
indication 
that 
all 
necessary 
disclosures 
have 
been 
presented. 
D. 
An 
opinion 
on 
the 
entity's 
financial 
statements. 
1-68 
Original 
AACSB: 
Communication 
AICPA 
BB: 
Legal 
AICPA 
FN: 
Reporting 
Bloom's: 
Knowledge 
Difficulty: 
Easy 
32. 
The 
primary 
purpose 
of 
the 
auditors' 
study 
of 
internal 
control 
for 
a 
nonpublic 
entity 
is: 
A. 
To 
provide 
constructive 
suggestions 
to 
the 
client 
for 
improving 
its 
internal 
control. 
B. 
To 
report 
on 
internal 
control 
as 
required 
by 
Auditing 
Standard 
No. 
5. 
C. 
To 
identify 
and 
detect 
fraud 
and 
irregularities 
perpetrated 
by 
client 
personnel. 
D. 
To 
determine 
the 
nature, 
timing, 
and 
extent 
of 
substantive 
procedures. 
Original 
AACSB: 
Analytic 
AICPA 
BB: 
Legal 
AICPA 
FN: 
Risk 
Analysis 
Bloom's: 
Knowledge 
Difficulty: 
Easy
Whittington -principles of auditing 19e
Whittington -principles of auditing 19e
Whittington -principles of auditing 19e
Whittington -principles of auditing 19e
Whittington -principles of auditing 19e
Whittington -principles of auditing 19e
Whittington -principles of auditing 19e
Whittington -principles of auditing 19e
Whittington -principles of auditing 19e
Whittington -principles of auditing 19e
Whittington -principles of auditing 19e
Whittington -principles of auditing 19e
Whittington -principles of auditing 19e
Whittington -principles of auditing 19e
Whittington -principles of auditing 19e
Whittington -principles of auditing 19e
Whittington -principles of auditing 19e
Whittington -principles of auditing 19e
Whittington -principles of auditing 19e
Whittington -principles of auditing 19e
Whittington -principles of auditing 19e
Whittington -principles of auditing 19e
Whittington -principles of auditing 19e
Whittington -principles of auditing 19e
Whittington -principles of auditing 19e
Whittington -principles of auditing 19e
Whittington -principles of auditing 19e
Whittington -principles of auditing 19e
Whittington -principles of auditing 19e
Whittington -principles of auditing 19e
Whittington -principles of auditing 19e
Whittington -principles of auditing 19e
Whittington -principles of auditing 19e
Whittington -principles of auditing 19e
Whittington -principles of auditing 19e
Whittington -principles of auditing 19e
Whittington -principles of auditing 19e
Whittington -principles of auditing 19e
Whittington -principles of auditing 19e
Whittington -principles of auditing 19e
Whittington -principles of auditing 19e
Whittington -principles of auditing 19e
Whittington -principles of auditing 19e
Whittington -principles of auditing 19e
Whittington -principles of auditing 19e
Whittington -principles of auditing 19e
Whittington -principles of auditing 19e
Whittington -principles of auditing 19e
Whittington -principles of auditing 19e
Whittington -principles of auditing 19e
Whittington -principles of auditing 19e
Whittington -principles of auditing 19e
Whittington -principles of auditing 19e
Whittington -principles of auditing 19e
Whittington -principles of auditing 19e
Whittington -principles of auditing 19e
Whittington -principles of auditing 19e
Whittington -principles of auditing 19e
Whittington -principles of auditing 19e
Whittington -principles of auditing 19e
Whittington -principles of auditing 19e

More Related Content

What's hot

Lecture 10, Chapter 14, Auditing Sales and Receivables
Lecture 10, Chapter 14, Auditing Sales and ReceivablesLecture 10, Chapter 14, Auditing Sales and Receivables
Lecture 10, Chapter 14, Auditing Sales and Receivables
Sazzad Hossain, ITP, MBA, CSCA™
 
BSA - Bangladesh Standards on Auditing
BSA - Bangladesh Standards on AuditingBSA - Bangladesh Standards on Auditing
BSA - Bangladesh Standards on Auditing
Sazzad Hossain, ITP, MBA, CSCA™
 
Principles of Audit
Principles of AuditPrinciples of Audit
Principles of Audit
VadivelM9
 
Ch 4. Reporting: ISA 700, 705 & 706
Ch 4. Reporting: ISA 700, 705 & 706Ch 4. Reporting: ISA 700, 705 & 706
Ch 4. Reporting: ISA 700, 705 & 706
Sazzad Hossain, ITP, MBA, CSCA™
 
Internal control system
Internal control systemInternal control system
Internal control system
Madiha Hassan
 
Lecture slide ,chapter 2, Governance and the Auditor
Lecture slide ,chapter 2, Governance and the AuditorLecture slide ,chapter 2, Governance and the Auditor
Lecture slide ,chapter 2, Governance and the Auditor
Sazzad Hossain, ITP, MBA, CSCA™
 
Chapter 12 - Designing Substantive Procedures
Chapter 12 - Designing Substantive ProceduresChapter 12 - Designing Substantive Procedures
Chapter 12 - Designing Substantive Procedures
Sazzad Hossain, ITP, MBA, CSCA™
 
Lecture 10, chap 15, Chapter 15,Auditing Purchases, Payables and Payroll
Lecture 10,  chap 15, Chapter 15,Auditing Purchases, Payables and PayrollLecture 10,  chap 15, Chapter 15,Auditing Purchases, Payables and Payroll
Lecture 10, chap 15, Chapter 15,Auditing Purchases, Payables and Payroll
Sazzad Hossain, ITP, MBA, CSCA™
 
Ch 13. substantive procedures
Ch 13. substantive proceduresCh 13. substantive procedures
Ch 13. substantive procedures
Sazzad Hossain, ITP, MBA, CSCA™
 
ISA 200 by Sazzad Hossain ITP CSCA
ISA 200 by Sazzad Hossain ITP CSCAISA 200 by Sazzad Hossain ITP CSCA
ISA 200 by Sazzad Hossain ITP CSCA
Sazzad Hossain, ITP, MBA, CSCA™
 
Ch 1. introduction to assurance Concept & Need for Assurance
Ch 1. introduction to assurance Concept & Need for AssuranceCh 1. introduction to assurance Concept & Need for Assurance
Ch 1. introduction to assurance Concept & Need for Assurance
Sazzad Hossain, ITP, MBA, CSCA™
 
Lecture slide ,chapter 6, Overview of the audit of financial reports
Lecture slide ,chapter 6, Overview of the audit of financial reportsLecture slide ,chapter 6, Overview of the audit of financial reports
Lecture slide ,chapter 6, Overview of the audit of financial reports
Sazzad Hossain, ITP, MBA, CSCA™
 
Lecture slide, chapter 1, An Overview of Auditing
Lecture slide, chapter 1, An Overview of AuditingLecture slide, chapter 1, An Overview of Auditing
Lecture slide, chapter 1, An Overview of Auditing
Sazzad Hossain, ITP, MBA, CSCA™
 
Forensic audit & procedure
Forensic audit & procedureForensic audit & procedure
Forensic audit & procedure
VadivelM9
 
Ch 10. documentation
Ch 10. documentationCh 10. documentation
Ch 10. documentation
Sazzad Hossain, ITP, MBA, CSCA™
 
Introduction of Assurance
Introduction of AssuranceIntroduction of Assurance
Introduction of Assurance
Sazzad Hossain, ITP, MBA, CSCA™
 
Internal Audit Plan 2015
Internal Audit Plan 2015Internal Audit Plan 2015
Internal Audit Plan 2015
Mohammad Kashif
 
Lecture slide, chapter 3,Professional Ethics, Independence and Audit Quality
Lecture slide, chapter 3,Professional Ethics, Independence and Audit QualityLecture slide, chapter 3,Professional Ethics, Independence and Audit Quality
Lecture slide, chapter 3,Professional Ethics, Independence and Audit Quality
Sazzad Hossain, ITP, MBA, CSCA™
 
Client Evaluation and Planning the Audit Lecture slide chapter 8
Client Evaluation and Planning the Audit Lecture slide chapter 8Client Evaluation and Planning the Audit Lecture slide chapter 8
Client Evaluation and Planning the Audit Lecture slide chapter 8
Sazzad Hossain, ITP, MBA, CSCA™
 

What's hot (20)

Lecture 10, Chapter 14, Auditing Sales and Receivables
Lecture 10, Chapter 14, Auditing Sales and ReceivablesLecture 10, Chapter 14, Auditing Sales and Receivables
Lecture 10, Chapter 14, Auditing Sales and Receivables
 
BSA - Bangladesh Standards on Auditing
BSA - Bangladesh Standards on AuditingBSA - Bangladesh Standards on Auditing
BSA - Bangladesh Standards on Auditing
 
Principles of Audit
Principles of AuditPrinciples of Audit
Principles of Audit
 
Ch 4. Reporting: ISA 700, 705 & 706
Ch 4. Reporting: ISA 700, 705 & 706Ch 4. Reporting: ISA 700, 705 & 706
Ch 4. Reporting: ISA 700, 705 & 706
 
Internal control system
Internal control systemInternal control system
Internal control system
 
Lecture slide ,chapter 2, Governance and the Auditor
Lecture slide ,chapter 2, Governance and the AuditorLecture slide ,chapter 2, Governance and the Auditor
Lecture slide ,chapter 2, Governance and the Auditor
 
Chapter 12 - Designing Substantive Procedures
Chapter 12 - Designing Substantive ProceduresChapter 12 - Designing Substantive Procedures
Chapter 12 - Designing Substantive Procedures
 
Lecture 10, chap 15, Chapter 15,Auditing Purchases, Payables and Payroll
Lecture 10,  chap 15, Chapter 15,Auditing Purchases, Payables and PayrollLecture 10,  chap 15, Chapter 15,Auditing Purchases, Payables and Payroll
Lecture 10, chap 15, Chapter 15,Auditing Purchases, Payables and Payroll
 
Audit.planning
Audit.planningAudit.planning
Audit.planning
 
Ch 13. substantive procedures
Ch 13. substantive proceduresCh 13. substantive procedures
Ch 13. substantive procedures
 
ISA 200 by Sazzad Hossain ITP CSCA
ISA 200 by Sazzad Hossain ITP CSCAISA 200 by Sazzad Hossain ITP CSCA
ISA 200 by Sazzad Hossain ITP CSCA
 
Ch 1. introduction to assurance Concept & Need for Assurance
Ch 1. introduction to assurance Concept & Need for AssuranceCh 1. introduction to assurance Concept & Need for Assurance
Ch 1. introduction to assurance Concept & Need for Assurance
 
Lecture slide ,chapter 6, Overview of the audit of financial reports
Lecture slide ,chapter 6, Overview of the audit of financial reportsLecture slide ,chapter 6, Overview of the audit of financial reports
Lecture slide ,chapter 6, Overview of the audit of financial reports
 
Lecture slide, chapter 1, An Overview of Auditing
Lecture slide, chapter 1, An Overview of AuditingLecture slide, chapter 1, An Overview of Auditing
Lecture slide, chapter 1, An Overview of Auditing
 
Forensic audit & procedure
Forensic audit & procedureForensic audit & procedure
Forensic audit & procedure
 
Ch 10. documentation
Ch 10. documentationCh 10. documentation
Ch 10. documentation
 
Introduction of Assurance
Introduction of AssuranceIntroduction of Assurance
Introduction of Assurance
 
Internal Audit Plan 2015
Internal Audit Plan 2015Internal Audit Plan 2015
Internal Audit Plan 2015
 
Lecture slide, chapter 3,Professional Ethics, Independence and Audit Quality
Lecture slide, chapter 3,Professional Ethics, Independence and Audit QualityLecture slide, chapter 3,Professional Ethics, Independence and Audit Quality
Lecture slide, chapter 3,Professional Ethics, Independence and Audit Quality
 
Client Evaluation and Planning the Audit Lecture slide chapter 8
Client Evaluation and Planning the Audit Lecture slide chapter 8Client Evaluation and Planning the Audit Lecture slide chapter 8
Client Evaluation and Planning the Audit Lecture slide chapter 8
 

Viewers also liked

Stephen Kazman - School Trustee Ward 5
Stephen Kazman - School Trustee Ward 5Stephen Kazman - School Trustee Ward 5
Stephen Kazman - School Trustee Ward 5
Stephen Kazman
 
engineering as a career.
engineering as a career.engineering as a career.
engineering as a career.
chief essay
 
Dossier Spesa Pubblica Febbraio 2015
Dossier Spesa Pubblica Febbraio 2015Dossier Spesa Pubblica Febbraio 2015
Dossier Spesa Pubblica Febbraio 2015
Alessandro Pastacci
 
Suomen kipu ry
Suomen kipu rySuomen kipu ry
Suomen kipu ry
Suomen Kipu ry
 
Viijaesitys
ViijaesitysViijaesitys
Viijaesitys
Markku Korhonen
 
public speaking skills
public speaking skillspublic speaking skills
public speaking skillsAsido Saragih
 
Crisis
CrisisCrisis
Crisis
Praveen PC
 
Public speaking
Public speakingPublic speaking
Public speaking
Islam Abdelaziz
 
Tracking Your Health the Easy Way
Tracking Your Health the Easy WayTracking Your Health the Easy Way
Tracking Your Health the Easy Way
stylemagazinescomau
 
Internet marketing strategies
Internet marketing strategiesInternet marketing strategies
Internet marketing strategies
Rahul Kumar
 

Viewers also liked (10)

Stephen Kazman - School Trustee Ward 5
Stephen Kazman - School Trustee Ward 5Stephen Kazman - School Trustee Ward 5
Stephen Kazman - School Trustee Ward 5
 
engineering as a career.
engineering as a career.engineering as a career.
engineering as a career.
 
Dossier Spesa Pubblica Febbraio 2015
Dossier Spesa Pubblica Febbraio 2015Dossier Spesa Pubblica Febbraio 2015
Dossier Spesa Pubblica Febbraio 2015
 
Suomen kipu ry
Suomen kipu rySuomen kipu ry
Suomen kipu ry
 
Viijaesitys
ViijaesitysViijaesitys
Viijaesitys
 
public speaking skills
public speaking skillspublic speaking skills
public speaking skills
 
Crisis
CrisisCrisis
Crisis
 
Public speaking
Public speakingPublic speaking
Public speaking
 
Tracking Your Health the Easy Way
Tracking Your Health the Easy WayTracking Your Health the Easy Way
Tracking Your Health the Easy Way
 
Internet marketing strategies
Internet marketing strategiesInternet marketing strategies
Internet marketing strategies
 

Similar to Whittington -principles of auditing 19e

Auditing And Assurance Services In Australia 6th Edition Louwers Test Bank
Auditing And Assurance Services In Australia 6th Edition Louwers Test BankAuditing And Assurance Services In Australia 6th Edition Louwers Test Bank
Auditing And Assurance Services In Australia 6th Edition Louwers Test Bank
qixoxyzu
 
ACC 460 ACC460 Final EXAM MCQ`s Correct Answers 100%
ACC 460 ACC460 Final EXAM MCQ`s Correct Answers 100%ACC 460 ACC460 Final EXAM MCQ`s Correct Answers 100%
ACC 460 ACC460 Final EXAM MCQ`s Correct Answers 100%
johnMilit
 
ACC 460 ACC/460 Final EXAM MCQ\s Correct Answers 100%
ACC 460 ACC/460 Final EXAM MCQ\s Correct Answers 100%ACC 460 ACC/460 Final EXAM MCQ\s Correct Answers 100%
ACC 460 ACC/460 Final EXAM MCQ\s Correct Answers 100%johnMilit
 
ACC 460 ACC/460 Final Exam 100% Correct
ACC 460 ACC/460 Final Exam 100% CorrectACC 460 ACC/460 Final Exam 100% Correct
ACC 460 ACC/460 Final Exam 100% Correct0_0klister
 
ACC 460 ACC/460 Final Exam 100% Correct
ACC 460 ACC/460 Final Exam 100% CorrectACC 460 ACC/460 Final Exam 100% Correct
ACC 460 ACC/460 Final Exam 100% CorrectRieTian99
 
ACC 460 ACC460 Final EXAM MCQ`s Correct Answers 100%
ACC 460 ACC460 Final EXAM MCQ`s Correct Answers 100%ACC 460 ACC460 Final EXAM MCQ`s Correct Answers 100%
ACC 460 ACC460 Final EXAM MCQ`s Correct Answers 100%johnMilit
 
ACC 460 ACC/460 Final EXAM MCQ`s Correct Answers 100%
ACC 460 ACC/460 Final EXAM MCQ`s Correct Answers 100%ACC 460 ACC/460 Final EXAM MCQ`s Correct Answers 100%
ACC 460 ACC/460 Final EXAM MCQ`s Correct Answers 100%7593retzeth
 
TEST BANK For Auditing & Assurance Services A Systematic Approach, 11th Editi...
TEST BANK For Auditing & Assurance Services A Systematic Approach, 11th Editi...TEST BANK For Auditing & Assurance Services A Systematic Approach, 11th Editi...
TEST BANK For Auditing & Assurance Services A Systematic Approach, 11th Editi...
robinsonayot
 
(New) final exam for acc 460 acc 460 all correct answers 100%
(New) final exam for acc 460 acc 460 all correct answers 100%(New) final exam for acc 460 acc 460 all correct answers 100%
(New) final exam for acc 460 acc 460 all correct answers 100%ri0908O0o
 
Acc 460 final exam mcq`s correct answers 100%
Acc 460 final exam mcq`s correct answers 100%Acc 460 final exam mcq`s correct answers 100%
Acc 460 final exam mcq`s correct answers 100%liamSali
 
(New) final exam for acc 460 all correct answers 100%
(New) final exam for acc 460 all correct answers 100%(New) final exam for acc 460 all correct answers 100%
(New) final exam for acc 460 all correct answers 100%liamSali
 
(New) final exam for acc 460 all correct answers 100%
(New) final exam for acc 460 all correct answers 100%(New) final exam for acc 460 all correct answers 100%
(New) final exam for acc 460 all correct answers 100%quikly11
 
Acc 460 final exam mcq`s correct answers 100%
Acc 460 final exam mcq`s correct answers 100%Acc 460 final exam mcq`s correct answers 100%
Acc 460 final exam mcq`s correct answers 100%Austing_3
 
Acc 303 week 2 quiz – strayer new
Acc 303 week 2 quiz – strayer newAcc 303 week 2 quiz – strayer new
Acc 303 week 2 quiz – strayer new
LindaAdams2017
 
auditing and assurance services louwers 6th edition test bank
auditing and assurance services louwers 6th edition test bankauditing and assurance services louwers 6th edition test bank
auditing and assurance services louwers 6th edition test bank
AliExpress24
 
Question 1 1. The American Institute of Certified Public Account.docx
Question 1 1. The American Institute of Certified Public Account.docxQuestion 1 1. The American Institute of Certified Public Account.docx
Question 1 1. The American Institute of Certified Public Account.docx
makdul
 
Auditing and Assurance Services In Australia 7th Edition Gay Test Bank
Auditing and Assurance Services In Australia 7th Edition Gay Test BankAuditing and Assurance Services In Australia 7th Edition Gay Test Bank
Auditing and Assurance Services In Australia 7th Edition Gay Test Bank
MageeMagee
 
Acc 460 acc460 final exam correct 100%
Acc 460 acc460 final exam correct 100%Acc 460 acc460 final exam correct 100%
Acc 460 acc460 final exam correct 100%flyperhan
 
(New) acc 460 acc460 final exam entire answers with questions correct 100%
(New) acc 460 acc460 final exam entire answers with questions correct  100%(New) acc 460 acc460 final exam entire answers with questions correct  100%
(New) acc 460 acc460 final exam entire answers with questions correct 100%twiter343r
 
Acc 460 acc460 final exam correct 100%
Acc 460 acc460 final exam correct 100%Acc 460 acc460 final exam correct 100%
Acc 460 acc460 final exam correct 100%singup22
 

Similar to Whittington -principles of auditing 19e (20)

Auditing And Assurance Services In Australia 6th Edition Louwers Test Bank
Auditing And Assurance Services In Australia 6th Edition Louwers Test BankAuditing And Assurance Services In Australia 6th Edition Louwers Test Bank
Auditing And Assurance Services In Australia 6th Edition Louwers Test Bank
 
ACC 460 ACC460 Final EXAM MCQ`s Correct Answers 100%
ACC 460 ACC460 Final EXAM MCQ`s Correct Answers 100%ACC 460 ACC460 Final EXAM MCQ`s Correct Answers 100%
ACC 460 ACC460 Final EXAM MCQ`s Correct Answers 100%
 
ACC 460 ACC/460 Final EXAM MCQ\s Correct Answers 100%
ACC 460 ACC/460 Final EXAM MCQ\s Correct Answers 100%ACC 460 ACC/460 Final EXAM MCQ\s Correct Answers 100%
ACC 460 ACC/460 Final EXAM MCQ\s Correct Answers 100%
 
ACC 460 ACC/460 Final Exam 100% Correct
ACC 460 ACC/460 Final Exam 100% CorrectACC 460 ACC/460 Final Exam 100% Correct
ACC 460 ACC/460 Final Exam 100% Correct
 
ACC 460 ACC/460 Final Exam 100% Correct
ACC 460 ACC/460 Final Exam 100% CorrectACC 460 ACC/460 Final Exam 100% Correct
ACC 460 ACC/460 Final Exam 100% Correct
 
ACC 460 ACC460 Final EXAM MCQ`s Correct Answers 100%
ACC 460 ACC460 Final EXAM MCQ`s Correct Answers 100%ACC 460 ACC460 Final EXAM MCQ`s Correct Answers 100%
ACC 460 ACC460 Final EXAM MCQ`s Correct Answers 100%
 
ACC 460 ACC/460 Final EXAM MCQ`s Correct Answers 100%
ACC 460 ACC/460 Final EXAM MCQ`s Correct Answers 100%ACC 460 ACC/460 Final EXAM MCQ`s Correct Answers 100%
ACC 460 ACC/460 Final EXAM MCQ`s Correct Answers 100%
 
TEST BANK For Auditing & Assurance Services A Systematic Approach, 11th Editi...
TEST BANK For Auditing & Assurance Services A Systematic Approach, 11th Editi...TEST BANK For Auditing & Assurance Services A Systematic Approach, 11th Editi...
TEST BANK For Auditing & Assurance Services A Systematic Approach, 11th Editi...
 
(New) final exam for acc 460 acc 460 all correct answers 100%
(New) final exam for acc 460 acc 460 all correct answers 100%(New) final exam for acc 460 acc 460 all correct answers 100%
(New) final exam for acc 460 acc 460 all correct answers 100%
 
Acc 460 final exam mcq`s correct answers 100%
Acc 460 final exam mcq`s correct answers 100%Acc 460 final exam mcq`s correct answers 100%
Acc 460 final exam mcq`s correct answers 100%
 
(New) final exam for acc 460 all correct answers 100%
(New) final exam for acc 460 all correct answers 100%(New) final exam for acc 460 all correct answers 100%
(New) final exam for acc 460 all correct answers 100%
 
(New) final exam for acc 460 all correct answers 100%
(New) final exam for acc 460 all correct answers 100%(New) final exam for acc 460 all correct answers 100%
(New) final exam for acc 460 all correct answers 100%
 
Acc 460 final exam mcq`s correct answers 100%
Acc 460 final exam mcq`s correct answers 100%Acc 460 final exam mcq`s correct answers 100%
Acc 460 final exam mcq`s correct answers 100%
 
Acc 303 week 2 quiz – strayer new
Acc 303 week 2 quiz – strayer newAcc 303 week 2 quiz – strayer new
Acc 303 week 2 quiz – strayer new
 
auditing and assurance services louwers 6th edition test bank
auditing and assurance services louwers 6th edition test bankauditing and assurance services louwers 6th edition test bank
auditing and assurance services louwers 6th edition test bank
 
Question 1 1. The American Institute of Certified Public Account.docx
Question 1 1. The American Institute of Certified Public Account.docxQuestion 1 1. The American Institute of Certified Public Account.docx
Question 1 1. The American Institute of Certified Public Account.docx
 
Auditing and Assurance Services In Australia 7th Edition Gay Test Bank
Auditing and Assurance Services In Australia 7th Edition Gay Test BankAuditing and Assurance Services In Australia 7th Edition Gay Test Bank
Auditing and Assurance Services In Australia 7th Edition Gay Test Bank
 
Acc 460 acc460 final exam correct 100%
Acc 460 acc460 final exam correct 100%Acc 460 acc460 final exam correct 100%
Acc 460 acc460 final exam correct 100%
 
(New) acc 460 acc460 final exam entire answers with questions correct 100%
(New) acc 460 acc460 final exam entire answers with questions correct  100%(New) acc 460 acc460 final exam entire answers with questions correct  100%
(New) acc 460 acc460 final exam entire answers with questions correct 100%
 
Acc 460 acc460 final exam correct 100%
Acc 460 acc460 final exam correct 100%Acc 460 acc460 final exam correct 100%
Acc 460 acc460 final exam correct 100%
 

Recently uploaded

The Influence of Marketing Strategy and Market Competition on Business Perfor...
The Influence of Marketing Strategy and Market Competition on Business Perfor...The Influence of Marketing Strategy and Market Competition on Business Perfor...
The Influence of Marketing Strategy and Market Competition on Business Perfor...
Adam Smith
 
amptalk_RecruitingDeck_english_2024.06.05
amptalk_RecruitingDeck_english_2024.06.05amptalk_RecruitingDeck_english_2024.06.05
amptalk_RecruitingDeck_english_2024.06.05
marketing317746
 
Improving profitability for small business
Improving profitability for small businessImproving profitability for small business
Improving profitability for small business
Ben Wann
 
Company Valuation webinar series - Tuesday, 4 June 2024
Company Valuation webinar series - Tuesday, 4 June 2024Company Valuation webinar series - Tuesday, 4 June 2024
Company Valuation webinar series - Tuesday, 4 June 2024
FelixPerez547899
 
5 Things You Need To Know Before Hiring a Videographer
5 Things You Need To Know Before Hiring a Videographer5 Things You Need To Know Before Hiring a Videographer
5 Things You Need To Know Before Hiring a Videographer
ofm712785
 
falcon-invoice-discounting-a-premier-platform-for-investors-in-india
falcon-invoice-discounting-a-premier-platform-for-investors-in-indiafalcon-invoice-discounting-a-premier-platform-for-investors-in-india
falcon-invoice-discounting-a-premier-platform-for-investors-in-india
Falcon Invoice Discounting
 
Mastering B2B Payments Webinar from BlueSnap
Mastering B2B Payments Webinar from BlueSnapMastering B2B Payments Webinar from BlueSnap
Mastering B2B Payments Webinar from BlueSnap
Norma Mushkat Gaffin
 
20240425_ TJ Communications Credentials_compressed.pdf
20240425_ TJ Communications Credentials_compressed.pdf20240425_ TJ Communications Credentials_compressed.pdf
20240425_ TJ Communications Credentials_compressed.pdf
tjcomstrang
 
Sustainability: Balancing the Environment, Equity & Economy
Sustainability: Balancing the Environment, Equity & EconomySustainability: Balancing the Environment, Equity & Economy
Sustainability: Balancing the Environment, Equity & Economy
Operational Excellence Consulting
 
Putting the SPARK into Virtual Training.pptx
Putting the SPARK into Virtual Training.pptxPutting the SPARK into Virtual Training.pptx
Putting the SPARK into Virtual Training.pptx
Cynthia Clay
 
Bài tập - Tiếng anh 11 Global Success UNIT 1 - Bản HS.doc
Bài tập - Tiếng anh 11 Global Success UNIT 1 - Bản HS.docBài tập - Tiếng anh 11 Global Success UNIT 1 - Bản HS.doc
Bài tập - Tiếng anh 11 Global Success UNIT 1 - Bản HS.doc
daothibichhang1
 
Search Disrupted Google’s Leaked Documents Rock the SEO World.pdf
Search Disrupted Google’s Leaked Documents Rock the SEO World.pdfSearch Disrupted Google’s Leaked Documents Rock the SEO World.pdf
Search Disrupted Google’s Leaked Documents Rock the SEO World.pdf
Arihant Webtech Pvt. Ltd
 
Maksym Vyshnivetskyi: PMO Quality Management (UA)
Maksym Vyshnivetskyi: PMO Quality Management (UA)Maksym Vyshnivetskyi: PMO Quality Management (UA)
Maksym Vyshnivetskyi: PMO Quality Management (UA)
Lviv Startup Club
 
3.0 Project 2_ Developing My Brand Identity Kit.pptx
3.0 Project 2_ Developing My Brand Identity Kit.pptx3.0 Project 2_ Developing My Brand Identity Kit.pptx
3.0 Project 2_ Developing My Brand Identity Kit.pptx
tanyjahb
 
Premium MEAN Stack Development Solutions for Modern Businesses
Premium MEAN Stack Development Solutions for Modern BusinessesPremium MEAN Stack Development Solutions for Modern Businesses
Premium MEAN Stack Development Solutions for Modern Businesses
SynapseIndia
 
What is the TDS Return Filing Due Date for FY 2024-25.pdf
What is the TDS Return Filing Due Date for FY 2024-25.pdfWhat is the TDS Return Filing Due Date for FY 2024-25.pdf
What is the TDS Return Filing Due Date for FY 2024-25.pdf
seoforlegalpillers
 
Cree_Rey_BrandIdentityKit.PDF_PersonalBd
Cree_Rey_BrandIdentityKit.PDF_PersonalBdCree_Rey_BrandIdentityKit.PDF_PersonalBd
Cree_Rey_BrandIdentityKit.PDF_PersonalBd
creerey
 
Recruiting in the Digital Age: A Social Media Masterclass
Recruiting in the Digital Age: A Social Media MasterclassRecruiting in the Digital Age: A Social Media Masterclass
Recruiting in the Digital Age: A Social Media Masterclass
LuanWise
 
ikea_woodgreen_petscharity_dog-alogue_digital.pdf
ikea_woodgreen_petscharity_dog-alogue_digital.pdfikea_woodgreen_petscharity_dog-alogue_digital.pdf
ikea_woodgreen_petscharity_dog-alogue_digital.pdf
agatadrynko
 
Authentically Social Presented by Corey Perlman
Authentically Social Presented by Corey PerlmanAuthentically Social Presented by Corey Perlman
Authentically Social Presented by Corey Perlman
Corey Perlman, Social Media Speaker and Consultant
 

Recently uploaded (20)

The Influence of Marketing Strategy and Market Competition on Business Perfor...
The Influence of Marketing Strategy and Market Competition on Business Perfor...The Influence of Marketing Strategy and Market Competition on Business Perfor...
The Influence of Marketing Strategy and Market Competition on Business Perfor...
 
amptalk_RecruitingDeck_english_2024.06.05
amptalk_RecruitingDeck_english_2024.06.05amptalk_RecruitingDeck_english_2024.06.05
amptalk_RecruitingDeck_english_2024.06.05
 
Improving profitability for small business
Improving profitability for small businessImproving profitability for small business
Improving profitability for small business
 
Company Valuation webinar series - Tuesday, 4 June 2024
Company Valuation webinar series - Tuesday, 4 June 2024Company Valuation webinar series - Tuesday, 4 June 2024
Company Valuation webinar series - Tuesday, 4 June 2024
 
5 Things You Need To Know Before Hiring a Videographer
5 Things You Need To Know Before Hiring a Videographer5 Things You Need To Know Before Hiring a Videographer
5 Things You Need To Know Before Hiring a Videographer
 
falcon-invoice-discounting-a-premier-platform-for-investors-in-india
falcon-invoice-discounting-a-premier-platform-for-investors-in-indiafalcon-invoice-discounting-a-premier-platform-for-investors-in-india
falcon-invoice-discounting-a-premier-platform-for-investors-in-india
 
Mastering B2B Payments Webinar from BlueSnap
Mastering B2B Payments Webinar from BlueSnapMastering B2B Payments Webinar from BlueSnap
Mastering B2B Payments Webinar from BlueSnap
 
20240425_ TJ Communications Credentials_compressed.pdf
20240425_ TJ Communications Credentials_compressed.pdf20240425_ TJ Communications Credentials_compressed.pdf
20240425_ TJ Communications Credentials_compressed.pdf
 
Sustainability: Balancing the Environment, Equity & Economy
Sustainability: Balancing the Environment, Equity & EconomySustainability: Balancing the Environment, Equity & Economy
Sustainability: Balancing the Environment, Equity & Economy
 
Putting the SPARK into Virtual Training.pptx
Putting the SPARK into Virtual Training.pptxPutting the SPARK into Virtual Training.pptx
Putting the SPARK into Virtual Training.pptx
 
Bài tập - Tiếng anh 11 Global Success UNIT 1 - Bản HS.doc
Bài tập - Tiếng anh 11 Global Success UNIT 1 - Bản HS.docBài tập - Tiếng anh 11 Global Success UNIT 1 - Bản HS.doc
Bài tập - Tiếng anh 11 Global Success UNIT 1 - Bản HS.doc
 
Search Disrupted Google’s Leaked Documents Rock the SEO World.pdf
Search Disrupted Google’s Leaked Documents Rock the SEO World.pdfSearch Disrupted Google’s Leaked Documents Rock the SEO World.pdf
Search Disrupted Google’s Leaked Documents Rock the SEO World.pdf
 
Maksym Vyshnivetskyi: PMO Quality Management (UA)
Maksym Vyshnivetskyi: PMO Quality Management (UA)Maksym Vyshnivetskyi: PMO Quality Management (UA)
Maksym Vyshnivetskyi: PMO Quality Management (UA)
 
3.0 Project 2_ Developing My Brand Identity Kit.pptx
3.0 Project 2_ Developing My Brand Identity Kit.pptx3.0 Project 2_ Developing My Brand Identity Kit.pptx
3.0 Project 2_ Developing My Brand Identity Kit.pptx
 
Premium MEAN Stack Development Solutions for Modern Businesses
Premium MEAN Stack Development Solutions for Modern BusinessesPremium MEAN Stack Development Solutions for Modern Businesses
Premium MEAN Stack Development Solutions for Modern Businesses
 
What is the TDS Return Filing Due Date for FY 2024-25.pdf
What is the TDS Return Filing Due Date for FY 2024-25.pdfWhat is the TDS Return Filing Due Date for FY 2024-25.pdf
What is the TDS Return Filing Due Date for FY 2024-25.pdf
 
Cree_Rey_BrandIdentityKit.PDF_PersonalBd
Cree_Rey_BrandIdentityKit.PDF_PersonalBdCree_Rey_BrandIdentityKit.PDF_PersonalBd
Cree_Rey_BrandIdentityKit.PDF_PersonalBd
 
Recruiting in the Digital Age: A Social Media Masterclass
Recruiting in the Digital Age: A Social Media MasterclassRecruiting in the Digital Age: A Social Media Masterclass
Recruiting in the Digital Age: A Social Media Masterclass
 
ikea_woodgreen_petscharity_dog-alogue_digital.pdf
ikea_woodgreen_petscharity_dog-alogue_digital.pdfikea_woodgreen_petscharity_dog-alogue_digital.pdf
ikea_woodgreen_petscharity_dog-alogue_digital.pdf
 
Authentically Social Presented by Corey Perlman
Authentically Social Presented by Corey PerlmanAuthentically Social Presented by Corey Perlman
Authentically Social Presented by Corey Perlman
 

Whittington -principles of auditing 19e

  • 1. 1-1 Chapter 01 The Role of the Public Accountant in the American Economy True / False Questions 1. Independent audits of today place more emphasis on sampling than did the audits of the 19th century. True False 2. The American Institute of Certified Public Accountants issues CPA certificates and permits CPAs to practice. True False 3. A company is either audited by the GAO or internal auditors, but not both. True False 4. The SEC does not pass on the merits of the securities that are registered with the agency. True False 5. The American Institute of Certified Public Accountants has the primary authority to establish accounting standards. True False 6. An annual peer review is a requirement of the AICPA. True False 7. Many small companies elect to have their financial statements reviewed by a CPA firm, rather than incur the cost of an audit. True False 8. Staff assistants in CPA firms generally are responsible for planning and coordinating audit engagements. True False
  • 2. 9. The Sarbanes-Oxley Act requires that auditors of certain publicly traded companies in the United States perform an integrated audit that includes providing assurance on both the financial statements and on compliance with laws and regulations. True False 10. Auditing is frequently only a small part of the practice of local CPA firms. 1-2 True False Multiple Choice Questions 11. A summary of findings rather than assurance is most likely to be included in a(n): A. Agreed-upon procedures report. B. Compilation report. C. Examination report. D. Review report. 12. The Statements on Auditing Standards have been issued by the: A. Auditing Standards Board. B. Financial Accounting Standards Board. C. Securities and Exchange Commission. D. Federal Bureau of Investigation. 13. The risk associated with a company's survival and profitability is referred to as: A. Business Risk. B. Information Risk. C. Detection Risk. D. Control Risk. 14. Historically, which of the following has the AICPA been most concerned with providing? A. Professional standards for CPAs. B. Professional guidance for regulating financial markets. C. Standards guiding the conduct of internal auditors. D. Staff support to Congress.
  • 3. 15. The organization charged with protecting investors and the public by requiring full disclosure of financial information by companies offering securities to the public is the: A. Auditing Standards Board. B. Financial Accounting Standards Board. C. Government Accounting Standards Boards. D. Securities and Exchange Commission. 16. An engagement in which a CPA firm arranges for a critical review of its practices by another CPA 1-3 firm is referred to as a(n): A. Peer Review Engagement. B. Quality Control Engagement. C. Quality Assurance Engagement. D. Attestation Engagement. 17. The serially-numbered pronouncements issued by the Auditing Standards Board over a period of years are known as: A. Auditing Statements of Position (ASPs). B. Accounting Series Releases (ASRs). C. Statements on Auditing Standards (SASs). D. Statements on Auditing Principles (SAPs). 18. The Government Accountability Office (GAO): A. Is primarily concerned with rapid processing of all accounts payable incurred by the federal government. B. Conducts operational audits and reports the results to Congress. C. Is a multinational organization of professional accountants. D. Is primarily concerned with budgets and forecasts approved by the SEC. 19. The risk that information is misstated is referred to as: A. Information risk. B. Inherent risk. C. Relative risk. D. Business risk.
  • 4. 20. The risk that a company will not be able to meet its obligations when they become due is an 1-4 aspect of: A. Information risk. B. Inherent risk. C. Relative risk. D. Business risk. 21. Which of the following attributes most clearly differentiates a CPA who audits management's financial statements as contrasted to management? A. Integrity. B. Competence. C. Independence. D. Keeping informed on current professional developments. 22. The attest function: A. Is an essential part of every engagement by the CPA, whether performing auditing, tax work, or other services. B. Includes the preparation of a report of the CPA's findings. C. Requires a consideration of internal control. D. Requires a complete review of all transactions during the period under examination. 23. Attestation risk is limited to a low level in which of the following engagement(s)? A. Both examinations and reviews. B. Examinations, but not reviews. C. Reviews, but not examinations. D. Neither examinations nor reviews. 24. When compared to an audit performed prior to 1900, an audit today: A. Is more likely to include tests of compliance with laws and regulations. B. Is less likely to include consideration of the effectiveness of internal control. C. Has bank loan officers as the primary financial statement user group. D. Includes a more detailed examination of all individual transactions.
  • 5. 25. Which of the following are issued by the Securities and Exchange Commission? 1-5 A. Accounting Research Studies. B. Accounting Trends and Techniques. C. Industry Audit Guides. D. Financial Reporting Releases. 26. Which of the following is not correct relating to the Sarbanes-Oxley Act? A. It toughens penalties for corporate fraud. B. It restricts the types of consulting CPAs may perform for audit clients. C. It created the Public Company Accounting Oversight Board (PCAOB) as a replacement for the Financial Accounting Standards Board. D. It eliminates a significant portion of the accounting profession's system of self-regulation. 27. An operational audit differs in many ways from an audit of financial statements. Which of the following is the best example of one of these differences? A. The usual audit of financial statements covers the four basic statements, whereas the operational audit is usually limited to either the balance sheet or the income statement. B. The boundaries of an operational audit are often drawn from an organization chart and are not limited to a single accounting period. C. Operational audits do not ordinarily result in the preparation of a report. D. The operational audit deals with pre-tax income. 28. The review of a company's financial statements by a CPA firm: A. Is substantially less in scope of procedures than an audit. B. Requires detailed analysis of the major accounts. C. Is of similar scope as an audit and adds similar credibility to the statements. D. Culminates in issuance of a report expressing the CPA's opinion as to the fairness of the statements. 29. Which statement is correct with respect to continuing professional education (CPE) requirements of members of the AICPA? A. Only members employed by the AICPA are required to take such courses. B. Only members in public practice are required to take such courses. C. Members, regardless of whether they are in public practice, are required to meet such requirements. D. There is no requirement for members to participate in CPE.
  • 6. 30. The FDIC Improvement Act requires that management of large financial institutions engage auditors to attest to assertions by management about the effectiveness of the institution's internal controls over: 1-6 A. Compliance with laws and regulations. B. Financial reporting. C. Effectiveness of operations. D. Efficiency of operations. 31. Passage of the Sarbanes-Oxley Act led to the establishment of the: A. Auditing Standards Board. B. Accounting Enforcement Releases Board. C. Public Company Accounting Oversight Board. D. Securities and Exchange Commission. 32. Which of the following professionals has primary responsibility for the performance of an audit? A. The managing partner of the firm. B. The senior assigned to the engagement. C. The manager assigned to the engagement. D. The partner in charge of the engagement. 33. Which of the following types of services is generally provided only by CPA firms? A. Tax audits. B. Financial statement audits. C. Compliance audits. D. Operational audits. 34. The right to practice as a CPA is given by which of the following organizations? A. State Boards of Accountancy. B. The AICPA. C. The SEC. D. The General Accounting Office.
  • 7. 35. Which of the following terms best describes the audit of a taxpayer's tax return by an IRS auditor? 1-7 A. Operational audit. B. Internal audit. C. Compliance audit. D. Government audit. 36. Inquiries and analytical procedures ordinarily form the basis for which type of engagement? A. Agreed-upon procedures. B. Audit. C. Examination. D. Review. 37. Which of the following best describes the reason why independent auditors report on financial statements? A. A management fraud may exist and it is more likely to be detected by independent auditors. B. Different interests may exist between the company preparing the statements and the persons using the statements. C. A misstatement of account balances may exist and is generally corrected as the result of the independent auditors' work. D. Poorly designed internal control may be in existence. 38. Governmental auditing often extends beyond examinations leading to the expression of opinion on the fairness of financial presentation and includes audits of efficiency, economy, effectiveness, and also: A. Accuracy. B. Evaluation. C. Compliance. D. Internal control. 39. Operational auditing is primarily oriented toward: A. Future improvements to accomplish the goals of management. B. The accuracy of data reflected in management's financial records. C. The verification that a company's financial statements are fairly presented. D. Past protection provided by existing internal control.
  • 8. 40. A typical objective of an operational audit is for the auditor to: A. Determine whether the financial statements fairly present the entity's operations. B. Evaluate the feasibility of attaining the entity's operational objectives. C. Make recommendations for improving performance. D. Report on the entity's relative success in attaining profit maximization. 41. An integrated audit performed under the Sarbanes-Oxley Act requires that auditors report on: 1-8 A. Option A B. Option B C. Option C D. Option D Matching Questions
  • 9. 42. Accountants are regulated by a variety of organizations. Match the following statements with the most directly related organizations. Organizations may be used once or not at all. 1-9 1. Formed to improve standards of financial accounting for state and local government entities State Boards of Accountancy. ____ 2. Issue CPA certificates Government Accounting Standards Board. ____ 3. Develop accounting standards for public and nonpublic companies American Institute of Certified Public Accountants. ____ 4. Develop accounting standards for the U.S. Government Financial Accounting Standards Board. ____ 5. Issue auditing standards for public companies Federal Accounting Standards Advisory Board. ____ 6. Prepares the CPA exam Public Company Accounting Oversight Board. ____ Essay Questions 43. The Sarbanes-Oxley Act of 2002 made significant reforms for public companies and their auditors. a. Describe the events that led up to the passage of the Act. b. Describe the major changes made by the Act.
  • 10. 44. Many people confuse the responsibilities of the independent auditors and the client's management with respect to audited financial statements. a. Describe management's responsibility regarding audited financial statements. b. Describe the independent auditors' responsibility regarding audited financial statements. c. Evaluate the following statement: "If the auditors disagree with management regarding an accounting principle used in the financial statements the auditors should express their views in the notes to the financial statements." 45. An investor is considering investing in one of two companies. The companies have very similar reported financial position and results of operations. However, only one of the companies has its financial statements audited. a. Describe what creates the demand for an audit in this situation. Include a discussion of how audited financial statements facilitate this investment transaction, and the effect of the audit on business risk and information risk. b. Identify the potential consequences to the company of not having its financial statements audited. 1-10
  • 11. Chapter 01 The Role of the Public Accountant in the American Economy Answer Key 1-11 True / False Questions 1. Independent audits of today place more emphasis on sampling than did the audits of the 19th century. TRUE AACSB: Analytic AICPA BB: Industry AICPA FN: Decision Making Blooms: Remember Difficulty: 2 Medium Learning Objective: 01-04 Explain why audits are demanded by society. Topic: Financial Statement Audits 2. The American Institute of Certified Public Accountants issues CPA certificates and permits CPAs to practice. FALSE AACSB: Analytic AICPA BB: Industry AICPA FN: Decision Making Blooms: Remember Difficulty: 2 Medium Learning Objective: 01-07 Explain the regulatory process for auditors of public companies and auditors of nonpublic companies. Topic: Public Accounting Profession 3. A company is either audited by the GAO or internal auditors, but not both. FALSE AACSB: Analytic AICPA BB: Industry AICPA FN: Decision Making Blooms: Remember Difficulty: 1 Easy Learning Objective: 01-06 Contrast the various types of audits and types of auditors. Topic: Financial Statement Audits 4. The SEC does not pass on the merits of the securities that are registered with the agency. TRUE AACSB: Analytic
  • 12. 1-12 AICPA BB: Industry AICPA FN: Decision Making Blooms: Remember Difficulty: 2 Medium Learning Objective: 01-07 Explain the regulatory process for auditors of public companies and auditors of nonpublic companies. Topic: Public Accounting Profession 5. The American Institute of Certified Public Accountants has the primary authority to establish accounting standards. FALSE AACSB: Analytic AICPA BB: Industry AICPA FN: Decision Making Blooms: Remember Difficulty: 1 Easy Learning Objective: 01-07 Explain the regulatory process for auditors of public companies and auditors of nonpublic companies. Topic: Public Accounting Profession 6. An annual peer review is a requirement of the AICPA. FALSE AACSB: Analytic AICPA BB: Industry AICPA FN: Decision Making Blooms: Remember Difficulty: 2 Medium Learning Objective: 01-07 Explain the regulatory process for auditors of public companies and auditors of nonpublic companies. Topic: Public Accounting Profession 7. Many small companies elect to have their financial statements reviewed by a CPA firm, rather than incur the cost of an audit. TRUE AACSB: Analytic AICPA BB: Industry AICPA FN: Decision Making Blooms: Remember Difficulty: 1 Easy Learning Objective: 01-07 Explain the regulatory process for auditors of public companies and auditors of nonpublic companies. Topic: Public Accounting Profession 8. Staff assistants in CPA firms generally are responsible for planning and coordinating audit engagements. FALSE AACSB: Analytic AICPA BB: Industry AICPA FN: Decision Making Blooms: Remember Difficulty: 1 Easy
  • 13. Learning Objective: 01-08 Describe how public accounting firms are typically organized and the responsibilities of auditors at the 1-13 various levels in the organization. Topic: Public Accounting Profession 9. The Sarbanes-Oxley Act requires that auditors of certain publicly traded companies in the United States perform an integrated audit that includes providing assurance on both the financial statements and on compliance with laws and regulations. FALSE AACSB: Analytic AICPA BB: Industry AICPA FN: Decision Making Blooms: Remember Difficulty: 2 Medium Learning Objective: 01-06 Contrast the various types of audits and types of auditors. Topic: Financial Statement Audits 10. Auditing is frequently only a small part of the practice of local CPA firms. TRUE AACSB: Analytic AICPA BB: Industry AICPA FN: Decision Making Blooms: Remember Difficulty: 2 Medium Learning Objective: 01-08 Describe how public accounting firms are typically organized and the responsibilities of auditors at the various levels in the organization. Topic: Public Accounting Profession Multiple Choice Questions 11. A summary of findings rather than assurance is most likely to be included in a(n): A. Agreed-upon procedures report. B. Compilation report. C. Examination report. D. Review report. AACSB: Analytic AICPA BB: Industry AICPA FN: Decision Making Blooms: Remember Difficulty: 2 Medium Learning Objective: 01-02 Identify assurance services that involve attestation. Topic: Attest Function
  • 14. 12. The Statements on Auditing Standards have been issued by the: A. Auditing Standards Board. B. Financial Accounting Standards Board. C. Securities and Exchange Commission. D. Federal Bureau of Investigation. 1-14 AACSB: Analytic AICPA BB: Industry AICPA FN: Decision Making Blooms: Remember Difficulty: 1 Easy Learning Objective: 01-07 Explain the regulatory process for auditors of public companies and auditors of nonpublic companies. Topic: Public Accounting Profession 13. The risk associated with a company's survival and profitability is referred to as: A. Business Risk. B. Information Risk. C. Detection Risk. D. Control Risk. AACSB: Analytic AICPA BB: Industry AICPA FN: Decision Making Blooms: Remember Difficulty: 1 Easy Learning Objective: 01-04 Explain why audits are demanded by society. Topic: Financial Statement Audits 14. Historically, which of the following has the AICPA been most concerned with providing? A. Professional standards for CPAs. B. Professional guidance for regulating financial markets. C. Standards guiding the conduct of internal auditors. D. Staff support to Congress. AACSB: Analytic AICPA BB: Industry AICPA FN: Decision Making Blooms: Remember Difficulty: 2 Medium Learning Objective: 01-07 Explain the regulatory process for auditors of public companies and auditors of nonpublic companies. Topic: Public Accounting Profession
  • 15. 15. The organization charged with protecting investors and the public by requiring full disclosure of financial information by companies offering securities to the public is the: A. Auditing Standards Board. B. Financial Accounting Standards Board. C. Government Accounting Standards Boards. D. Securities and Exchange Commission. 1-15 AACSB: Analytic AICPA BB: Industry AICPA FN: Decision Making Blooms: Remember Difficulty: 2 Medium Learning Objective: 01-07 Explain the regulatory process for auditors of public companies and auditors of nonpublic companies. Topic: Public Accounting Profession 16. An engagement in which a CPA firm arranges for a critical review of its practices by another CPA firm is referred to as a(n): A. Peer Review Engagement. B. Quality Control Engagement. C. Quality Assurance Engagement. D. Attestation Engagement. AACSB: Analytic AICPA BB: Industry AICPA FN: Decision Making Blooms: Remember Difficulty: 1 Easy Learning Objective: 01-07 Explain the regulatory process for auditors of public companies and auditors of nonpublic companies. Topic: Public Accounting Profession 17. The serially-numbered pronouncements issued by the Auditing Standards Board over a period of years are known as: A. Auditing Statements of Position (ASPs). B. Accounting Series Releases (ASRs). C. Statements on Auditing Standards (SASs). D. Statements on Auditing Principles (SAPs). AACSB: Analytic AICPA BB: Industry AICPA FN: Decision Making Blooms: Remember Difficulty: 1 Easy Learning Objective: 01-07 Explain the regulatory process for auditors of public companies and auditors of nonpublic companies. Topic: Public Accounting Profession
  • 16. 18. The Government Accountability Office (GAO): A. Is primarily concerned with rapid processing of all accounts payable incurred by the federal 1-16 government. B. Conducts operational audits and reports the results to Congress. C. Is a multinational organization of professional accountants. D. Is primarily concerned with budgets and forecasts approved by the SEC. AACSB: Analytic AICPA BB: Industry AICPA FN: Decision Making Blooms: Remember Difficulty: 1 Easy Learning Objective: 01-06 Contrast the various types of audits and types of auditors. Topic: Financial Statement Audits 19. The risk that information is misstated is referred to as: A. Information risk. B. Inherent risk. C. Relative risk. D. Business risk. AACSB: Analytic AICPA BB: Industry AICPA FN: Decision Making Blooms: Remember Difficulty: 1 Easy Learning Objective: 01-04 Explain why audits are demanded by society. Topic: Financial Statement Audits 20. The risk that a company will not be able to meet its obligations when they become due is an aspect of: A. Information risk. B. Inherent risk. C. Relative risk. D. Business risk. AACSB: Analytic AICPA BB: Industry AICPA FN: Decision Making Blooms: Remember Difficulty: 1 Easy Learning Objective: 01-04 Explain why audits are demanded by society. Topic: Financial Statement Audits
  • 17. 21. Which of the following attributes most clearly differentiates a CPA who audits management's financial statements as contrasted to management? A. Integrity. B. Competence. C. Independence. D. Keeping informed on current professional developments. 1-17 AACSB: Analytic AICPA BB: Industry AICPA FN: Decision Making Blooms: Remember Difficulty: 1 Easy Learning Objective: 01-01 Describe the nature of assurance services. Topic: Assurance Services 22. The attest function: A. Is an essential part of every engagement by the CPA, whether performing auditing, tax work, or other services. B. Includes the preparation of a report of the CPA's findings. C. Requires a consideration of internal control. D. Requires a complete review of all transactions during the period under examination. AACSB: Analytic AICPA BB: Industry AICPA FN: Decision Making Blooms: Remember Difficulty: 2 Medium Learning Objective: 01-02 Identify assurance services that involve attestation. Topic: Attest Function 23. Attestation risk is limited to a low level in which of the following engagement(s)? A. Both examinations and reviews. B. Examinations, but not reviews. C. Reviews, but not examinations. D. Neither examinations nor reviews. AACSB: Analytic AICPA BB: Industry AICPA FN: Decision Making Blooms: Remember Difficulty: 1 Easy Learning Objective: 01-02 Identify assurance services that involve attestation. Topic: Attest Function
  • 18. 24. When compared to an audit performed prior to 1900, an audit today: A. Is more likely to include tests of compliance with laws and regulations. B. Is less likely to include consideration of the effectiveness of internal control. C. Has bank loan officers as the primary financial statement user group. D. Includes a more detailed examination of all individual transactions. 1-18 AACSB: Analytic AICPA BB: Industry AICPA FN: Decision Making Blooms: Remember Difficulty: 2 Medium Learning Objective: 01-04 Explain why audits are demanded by society. Topic: Financial Statement Audits 25. Which of the following are issued by the Securities and Exchange Commission? A. Accounting Research Studies. B. Accounting Trends and Techniques. C. Industry Audit Guides. D. Financial Reporting Releases. AACSB: Analytic AICPA BB: Industry AICPA FN: Decision Making Blooms: Remember Difficulty: 2 Medium Learning Objective: 01-07 Explain the regulatory process for auditors of public companies and auditors of nonpublic companies. Topic: Public Accounting Profession 26. Which of the following is not correct relating to the Sarbanes-Oxley Act? A. It toughens penalties for corporate fraud. B. It restricts the types of consulting CPAs may perform for audit clients. C. It created the Public Company Accounting Oversight Board (PCAOB) as a replacement for the Financial Accounting Standards Board. D. It eliminates a significant portion of the accounting profession's system of self-regulation. AACSB: Analytic AICPA BB: Industry AICPA FN: Decision Making Blooms: Remember Difficulty: 2 Medium Learning Objective: 01-05 Describe how the credibility of the accounting profession was affected by the large number of companies reporting accounting irregularities in the beginning of this century. Topic: Financial Statement Audits
  • 19. 27. An operational audit differs in many ways from an audit of financial statements. Which of the following is the best example of one of these differences? A. The usual audit of financial statements covers the four basic statements, whereas the operational audit is usually limited to either the balance sheet or the income statement. B. The boundaries of an operational audit are often drawn from an organization chart and are not limited to a single accounting period. C. Operational audits do not ordinarily result in the preparation of a report. D. The operational audit deals with pre-tax income. 1-19 AACSB: Analytic AICPA BB: Industry AICPA FN: Decision Making Blooms: Remember Difficulty: 2 Medium Learning Objective: 01-06 Contrast the various types of audits and types of auditors. Topic: Financial Statement Audits 28. The review of a company's financial statements by a CPA firm: A. Is substantially less in scope of procedures than an audit. B. Requires detailed analysis of the major accounts. C. Is of similar scope as an audit and adds similar credibility to the statements. D. Culminates in issuance of a report expressing the CPA's opinion as to the fairness of the statements. AACSB: Analytic AICPA BB: Industry AICPA FN: Decision Making Blooms: Remember Difficulty: 1 Easy Learning Objective: 01-02 Identify assurance services that involve attestation. Topic: Attest Function 29. Which statement is correct with respect to continuing professional education (CPE) requirements of members of the AICPA? A. Only members employed by the AICPA are required to take such courses. B. Only members in public practice are required to take such courses. C. Members, regardless of whether they are in public practice, are required to meet such requirements. D. There is no requirement for members to participate in CPE. AACSB: Analytic AICPA BB: Industry AICPA FN: Decision Making Blooms: Remember Difficulty: 2 Medium
  • 20. Learning Objective: 01-07 Explain the regulatory process for auditors of public companies and auditors of nonpublic companies. 1-20 Topic: Public Accounting Profession 30. The FDIC Improvement Act requires that management of large financial institutions engage auditors to attest to assertions by management about the effectiveness of the institution's internal controls over: A. Compliance with laws and regulations. B. Financial reporting. C. Effectiveness of operations. D. Efficiency of operations. AACSB: Analytic AICPA BB: Industry AICPA FN: Decision Making Blooms: Remember Difficulty: 2 Medium Learning Objective: 01-04 Explain why audits are demanded by society. Topic: Financial Statement Audits 31. Passage of the Sarbanes-Oxley Act led to the establishment of the: A. Auditing Standards Board. B. Accounting Enforcement Releases Board. C. Public Company Accounting Oversight Board. D. Securities and Exchange Commission. AACSB: Analytic AICPA BB: Industry AICPA FN: Decision Making Blooms: Remember Difficulty: 2 Medium Learning Objective: 01-05 Describe how the credibility of the accounting profession was affected by the large number of companies reporting accounting irregularities in the beginning of this century. Topic: Financial Statement Audits 32. Which of the following professionals has primary responsibility for the performance of an audit? A. The managing partner of the firm. B. The senior assigned to the engagement. C. The manager assigned to the engagement. D. The partner in charge of the engagement. AACSB: Analytic AICPA BB: Industry AICPA FN: Decision Making Blooms: Remember Difficulty: 2 Medium
  • 21. Learning Objective: 01-08 Describe how public accounting firms are typically organized and the responsibilities of auditors at the 1-21 various levels in the organization. Topic: Public Accounting Profession 33. Which of the following types of services is generally provided only by CPA firms? A. Tax audits. B. Financial statement audits. C. Compliance audits. D. Operational audits. AACSB: Analytic AICPA BB: Industry AICPA FN: Decision Making Blooms: Remember Difficulty: 2 Medium Learning Objective: 01-02 Identify assurance services that involve attestation. Topic: Attest Function 34. The right to practice as a CPA is given by which of the following organizations? A. State Boards of Accountancy. B. The AICPA. C. The SEC. D. The General Accounting Office. AACSB: Analytic AICPA BB: Industry AICPA FN: Decision Making Blooms: Remember Difficulty: 2 Medium Learning Objective: 01-07 Explain the regulatory process for auditors of public companies and auditors of nonpublic companies. Topic: Public Accounting Profession 35. Which of the following terms best describes the audit of a taxpayer's tax return by an IRS auditor? A. Operational audit. B. Internal audit. C. Compliance audit. D. Government audit. AACSB: Analytic AICPA BB: Industry AICPA FN: Decision Making Blooms: Remember Difficulty: 2 Medium Learning Objective: 01-06 Contrast the various types of audits and types of auditors. Topic: Financial Statement Audits
  • 22. 36. Inquiries and analytical procedures ordinarily form the basis for which type of engagement? 1-22 A. Agreed-upon procedures. B. Audit. C. Examination. D. Review. AACSB: Analytic AICPA BB: Industry AICPA FN: Decision Making Blooms: Remember Difficulty: 1 Easy Learning Objective: 01-02 Identify assurance services that involve attestation. Topic: Attest Function 37. Which of the following best describes the reason why independent auditors report on financial statements? A. A management fraud may exist and it is more likely to be detected by independent auditors. B. Different interests may exist between the company preparing the statements and the persons using the statements. C. A misstatement of account balances may exist and is generally corrected as the result of the independent auditors' work. D. Poorly designed internal control may be in existence. AACSB: Analytic AICPA BB: Industry AICPA FN: Decision Making Blooms: Remember Difficulty: 2 Medium Source: AICPA Topic: Financial Statement Audits 38. Governmental auditing often extends beyond examinations leading to the expression of opinion on the fairness of financial presentation and includes audits of efficiency, economy, effectiveness, and also: A. Accuracy. B. Evaluation. C. Compliance. D. Internal control. AACSB: Reflective Thinking AICPA BB: Critical Thinking AICPA FN: Decision Making Blooms: Understand Difficulty: 3 Hard Learning Objective: 01-06 Contrast the various types of audits and types of auditors. Source: AICPA
  • 23. 1-23 Topic: Financial Statement Audits 39. Operational auditing is primarily oriented toward: A. Future improvements to accomplish the goals of management. B. The accuracy of data reflected in management's financial records. C. The verification that a company's financial statements are fairly presented. D. Past protection provided by existing internal control. AACSB: Reflective Thinking AICPA BB: Critical Thinking AICPA FN: Decision Making Blooms: Understand Difficulty: 3 Hard Learning Objective: 01-06 Contrast the various types of audits and types of auditors. Source: AICPA Topic: Financial Statement Audits 40. A typical objective of an operational audit is for the auditor to: A. Determine whether the financial statements fairly present the entity's operations. B. Evaluate the feasibility of attaining the entity's operational objectives. C. Make recommendations for improving performance. D. Report on the entity's relative success in attaining profit maximization. AACSB: Reflective Thinking AICPA BB: Critical Thinking AICPA FN: Decision Making Blooms: Understand Difficulty: 3 Hard Learning Objective: 01-06 Contrast the various types of audits and types of auditors. Source: AICPA Topic: Financial Statement Audits 41. An integrated audit performed under the Sarbanes-Oxley Act requires that auditors report on: A. Option A B. Option B C. Option C D. Option D AACSB: Analytic
  • 24. 1-24 AICPA BB: Industry AICPA FN: Decision Making Blooms: Remember Difficulty: 2 Medium Learning Objective: 01-06 Contrast the various types of audits and types of auditors. Topic: Financial Statement Audits Matching Questions 42. Accountants are regulated by a variety of organizations. Match the following statements with the most directly related organizations. Organizations may be used once or not at all. 1. Formed to improve standards of financial accounting for state and local government entities State Boards of Accountancy. 2 2. Issue CPA certificates Government Accounting Standards Board. 1 3. Develop accounting standards for public and nonpublic companies American Institute of Certified Public Accountants. 6 4. Develop accounting standards for the U.S. Government Financial Accounting Standards Board. 3 5. Issue auditing standards for public companies Federal Accounting Standards Advisory Board. 4 6. Prepares the CPA exam Public Company Accounting Oversight Board. 5 AACSB: Analytic AICPA BB: Industry AICPA FN: Decision Making Blooms: Remember Difficulty: 2 Medium Learning Objective: 01-06 Contrast the various types of audits and types of auditors. Learning Objective: 01-07 Explain the regulatory process for auditors of public companies and auditors of nonpublic companies. Topic: Financial Statement Audits Topic: Public Accounting Profession Essay Questions
  • 25. 43. The Sarbanes-Oxley Act of 2002 made significant reforms for public companies and their auditors. a. Describe the events that led up to the passage of the Act. b. Describe the major changes made by the Act. a. The events leading up to the passage of the Sarbanes-Oxley Act include: • A large number of misstatements of financial statements, many of which resulted from fraudulent financial reporting. Notably including WorldCom and Enron. • The conviction of the Big 5 accounting firm of Arthur Andersen on charges of destroying evidence. b. The major reforms made the Act include: • Tougher penalties for fraud. • Restrictions on the types of consulting services that may be provided by auditors to their public audit clients. • The creation of the Public Company Accounting Oversight Board to create auditing standards and oversee accounting firms that audit public companies. • Requirements for management to make an assertion about the effectiveness of internal control. • Requirements for auditors of public companies to audit and report on internal control. 1-25 AACSB: Reflective Thinking AICPA BB: Critical Thinking AICPA FN: Decision Making Blooms: Understand Difficulty: 3 Hard Learning Objective: 01-05 Describe how the credibility of the accounting profession was affected by the large number of companies reporting accounting irregularities in the beginning of this century. Learning Objective: 01-06 Contrast the various types of audits and types of auditors. Topic: Financial Statement Audits
  • 26. 44. Many people confuse the responsibilities of the independent auditors and the client's management with respect to audited financial statements. a. Describe management's responsibility regarding audited financial statements. b. Describe the independent auditors' responsibility regarding audited financial statements. c. Evaluate the following statement: "If the auditors disagree with management regarding an accounting principle used in the financial statements the auditors should express their views in the notes to the financial statements." a. Management has primary responsibility for the fairness of the financial statements and internal control. b. The auditors are responsible for performing an independent audit of the financial statements and issuing a report on them in accordance with generally accepted auditing standards. c. The statement if false. The notes to the financial statements should contain only representations of management. The auditors should express their reservations in their report. 1-26 AACSB: Analytic AICPA BB: Industry AICPA FN: Decision Making Blooms: Remember Difficulty: 2 Medium Learning Objective: 01-02 Identify assurance services that involve attestation. Topic: Attest Function
  • 27. 45. An investor is considering investing in one of two companies. The companies have very similar reported financial position and results of operations. However, only one of the companies has its financial statements audited. a. Describe what creates the demand for an audit in this situation. Include a discussion of how audited financial statements facilitate this investment transaction, and the effect of the audit on business risk and information risk. b. Identify the potential consequences to the company of not having its financial statements audited. a. Audits add credibility to the financial statements of the company. The individual can invest in the company knowing that there is a low probability that the financial statements depart materially from generally accepted accounting principles. Audited financial statements facilitate this transaction by reducing risk related to the investment. Specifically, audits reduce information risk--the risk that information used to make the investment decision is misstated-- related to the financial statements. Audited financial statements do not directly affect business risk, which is the risk that the company will not be able to meet its financial obligations. b. The potential consequences of not having an audit are: • If the investor is particularly risk averse, he or she may not invest in the company at all. • If the investor decides to invest in the company, he or she will not be willing to pay as high a price because the investor will want to be compensated for the additional risk that is involved in relying upon unaudited financial statements. 1-27 AACSB: Analytic AICPA BB: Industry AICPA FN: Decision Making Blooms: Remember Difficulty: 2 Medium Learning Objective: 01-04 Explain why audits are demanded by society. Topic: Financial Statement Audits
  • 28. 1-28 Chapter 02 Professional Standards Multiple Choice Questions 1. The attestation standards of reporting do not require the attestation report to include a statement that A. Provides a conclusion whether the subject matter is presented in conformity with established or stated criteria. B. Indicates that the practitioner has significant reservations about the engagement. C. Identifies the subject matter or assertion being reported on. D. Indicates that the accountant assumes no responsibility to update the report. 2. Control risk is A. The probability that a material misstatement could not be prevented or detected by the entity's internal control policies and procedures. B. The probability that a material misstatement could occur and not be detected by auditors' procedures. C. The risk that auditors will not be able to complete the audit on a timely basis. D. The risk that auditors will not properly control the staff on the audit engagement. 3. The responsibilities principle under generally accepted auditing standards does not include which of the following? A. Competence and capabilities. B. Independent attitude. C. Due care. D. Planning and supervision. 4. Which of the following types of auditors' reports does not require an explanatory paragraph to support the opinion? A. Unqualified opinion. B. Adverse opinion. C. Qualified opinion. D. Disclaimer of opinion.
  • 29. 5. Which of the following is an element of a system of quality control that should be considered by a public accounting firm in establishing its quality control policies and procedures? A. Lending credibility to a client's financial statements. B. Using statistical sampling techniques. C. Acceptance and continuance of clients. D. Membership in the Center for Public Company Audit Firms (CPCAF). 1-29 6. Which of the following presumptions does not relate to the reliability of audit evidence? A. The more effective the client's internal control, the more assurance it provides about the accounting data and financial statements. B. The auditors' opinion, to be economically useful, is formed within reasonable time and based on evidence obtained at a reasonable cost. C. Evidence obtained from independent sources outside the entity is more reliable than evidence secured solely within the entity. D. The independent auditors' direct personal knowledge, obtained through observation and inspection, is more persuasive than information obtained indirectly. 7. An important role of the Public Company Accounting Oversight Board (PCAOB) is to oversee the A. Issuance of statements by the Financial Accounting Standards Board. B. Preparation and grading of the Uniform CPA Examination. C. Peer review of member firms of the Private Companies Practice Section. D. Regulation of firms that audit public companies. 8. Audit evidence is usually considered sufficient when A. It is reliable. B. There is enough quantity to afford a reasonable basis for an opinion on financial statements. C. It has the qualities of being relevant, objective, and free from unknown bias. D. It has been obtained through random selection methods.
  • 30. 1-30 9. Which of the following is not considered a type of audit evidence? A. The company's trial balance. B. Auditors' calculations. C. Physical observation. D. Verbal statements made by client personnel. 10. The AICPA attestation standards differ from the AICPA responsibilities principle, performance principle and reporting principle in that: A. The attestation standards contain no requirement to obtain an understanding of the entity and assess the risk of material misstatement. B. The attestation standards do not require competence and capabilities. C. The attestation standards do not require planning for attestation engagements or supervision of accountants and consultants who perform the work. D. The attestation standards do not require a report that states the character of the engagement. 11. An audit of the financial statements of Camden Corporation is being conducted by external auditors. The external auditors are expected to: A. Certify the correctness of Camden's financial statements. B. Make a complete examination of Camden's records and verify all of Camden's transactions. C. Give an opinion on the fair presentation of Camden's financial statements in conformity with the applicable financial reporting framework (e.g., GAAP, IFRS). D. Give an opinion on the attractiveness of Camden for investment purposes and critique the wisdom and legality of its business decisions. 12. Auditors try to achieve independence in appearance in order to: A. Maintain public confidence in the profession. B. Become independent in fact. C. Comply with the responsibilities principle. D. Maintain an unbiased mental attitude.
  • 31. 13. The independent auditors' plan prepared prior to the start of field work is appropriately considered documentation of A. Planning. B. Supervision. C. Information evaluation. D. Quality assurance. 14. Which of the following procedures would provide the most reliable audit evidence? A. Inquiries of the client's accounting staff held in private. B. Inspection of pre-­‐numbered client shipping documents. C. Inspection of bank statements obtained directly from the client's financial institution. D. Analytical procedures performed by auditors on the client's trial balance. 15. Which of the following is not an attestation standard? A. The practitioner must obtain sufficient evidence to provide a reasonable basis for the conclusion expressed in the report. B. The practitioner must identify the subject matter or the assertion being reported on and state the character of the engagement. C. The practitioner must adequately plan the work and must properly supervise any assistants. D. A sufficient understanding of the client's internal controls shall be obtained to plan the engagement. 16. Which of the following would most likely be a violation of the independence requirement found in the responsibilities principle under generally accepted auditing standards? A. An auditor on the engagement has a distant relative who is employed by a vendor that does a significant amount of business with clients. B. The client's Chief Executive Officer graduated from the same university as the partner in charge of the accounting firm. C. An auditor on the engagement owns a financial interest in the stock of the client. D. The client provides financial support to a number of charitable causes that also receive support from the accounting firm. 1-31
  • 32. 17. A vendor's invoice received and held by the client would be considered what type of evidence? A. External. B. Internal. C. External-­‐internal. D. Written representation. 18. Which of the following statements is generally correct about the appropriateness of audit evidence? A. Auditors' direct personal knowledge, obtained through observation and inspection, is more persuasive than information obtained indirectly from independent outside sources. B. To be reliable, audit evidence must be either valid or relevant, but need not be both. C. Client accounting data alone may be considered sufficient appropriate audit evidence to issue an unqualified opinion on client financial statements. D. Appropriateness of audit evidence refers to the amount of corroborative evidence to be obtained. 19. The standard auditors' report refers to GAAS and GAAP in which paragraph? A. GAAS: Scope only; GAAP: Opinion only B. GAAS: Introductory only; GAAP: Scope and opinion C. GAAS: Introductory and scope; GAAP: Opinion only D. GAAS: Introductory only; GAAP: All paragraphs 20. Which of the following is not included in the auditors' standard report representing an unqualified opinion? A. A brief indication of the responsibility of auditors and management for the financial statements. B. An indication that all appropriate disclosures have been made and included in the financial statements. C. An indication that the audit was conducted in accordance with standards established by the PCAOB. D. The auditors' opinion on the fairness of the financial statements. 1-32
  • 33. 21. Internal evidence A. Is obtained directly from third parties independent of the client. B. Originates outside of the client's system but has been received and processed by the client. C. Consists of documents that are produced, used, and stored within the client's information system. D. Consists of representations made by the client's officers, directors, owners, and employees. 22. Which of the following presumptions is correct about the reliability of audit evidence? A. Information obtained indirectly from outside sources is the most reliable form of audit evidence. B. To be reliable, audit evidence should be convincing rather than persuasive. C. Reliability of audit evidence refers to the amount of corroborative evidence obtained. D. An effective system of internal control provides more assurance about the reliability of audit evidence. 23. When auditors do not mention consistency in the auditors' report, a reader of the financial statements may infer A. That the applicable financial reporting framework (e.g., GAAP) has been consistently observed in the current period in relation to the preceding period. B. That no material departure from the applicable financial reporting framework (e.g., GAAP) has been detected. C. That no reclassification of items or change in classifications has occurred. D. Nothing about application of accounting principles within the period. 24. The auditors' responsibility to express an opinion on the financial statements is A. Implicitly represented in the auditors' standard report. B. Explicitly represented in the introductory paragraph of the auditors' standard report. C. Explicitly represented in the scope paragraph of the auditors' standard report. D. Explicitly represented in the opinion paragraph of the auditors' standard report. 1-33
  • 34. 25. Which of the following is not a concept from the performance principle under generally accepted auditing standards? A. The auditor must plan the work and properly supervise any assistants. B. The auditor must express an opinion in accordance with the auditor's findings. C. The auditor must obtain sufficient appropriate evidence about whether material misstatements exist. D. The auditor must determine and apply an appropriate materiality level throughout the audit. 26. Under generally accepted auditing standards, which of the following reflects a concept related to the responsibilities principle? A. The initial planning of the audit engagement should occur with the audit partner, manager, senior, and client personnel. B. The confirmation of accounts receivable should occur on each audit. C. The completion of an internal control questionnaire. D. Maintaining professional skepticism and exercising professional judgment. 27. Which of the following represent audit quality guides that remain stable over time and are applicable for all audits? A. Auditing procedures. B. Auditing standards. C. Due care. D. System of quality control. 28. Which of the following situations would most likely be in conflict with the responsibilities principle? A. Auditors perform the engagement with prudent auditors, but not expert auditors. B. Auditors obtain expertise in their client's industry as they are conducting the audit examination. C. Auditors are directly involved with a client manager in a strategic decision-­‐making capacity. D. Auditors fail to document their assessment of control risk following their study of internal control. 1-34
  • 35. 1-35 29. Which of the following statements is not true with respect to the evidence that would be gathered when assessments of control risk are high? A. Auditors would be required to rely on external (rather than internal) forms of evidence. B. Auditors would be required to perform procedures at interim periods, rather than at year end. C. Auditors would be required to confirm a larger number of customer accounts receivable balances. D. Auditors would be required to obtain more evidence through direct personal observation. 30. As it relates to audit evidence, appropriateness refers to the A. Originality of evidence gathered. B. Quality of evidence gathered. C. Quantity of evidence gathered. D. Timeliness of evidence gathered. 31. Which of the following information would not be included in the auditors' standard report? A. The names of the financial statements audited. B. A description of the nature of an audit. C. An indication that all necessary disclosures have been presented. D. An opinion on the entity's financial statements. 32. The primary purpose of the auditors' study of internal control for a nonpublic entity is: A. To provide constructive suggestions to the client for improving its internal control. B. To report on internal control as required by Auditing Standard No. 5. C. To identify and detect fraud and irregularities perpetrated by client personnel. D. To determine the nature, timing, and extent of substantive procedures. 33. Which reporting options do auditors have if the client's financial statements are not presented according to the applicable financial framework (e.g., GAAP, IFRS)? A. Unqualified or disclaimer of opinion. B. Qualified or disclaimer of opinion. C. Unqualified or adverse. D. Qualified or adverse.
  • 36. 34. Which of the following is most closely related to system of quality control regarding engagement performance? A. Requiring all of the firm's personnel to provide a summary of their investments and other financial relationships. B. Evaluating the firm's system of quality controls on a periodic basis. C. Utilizing standardized audit plans and audit documentation on engagements in a particular industry. D. Evaluating the firm's ability to provide a quality audit to a prospective client. 1-36 Question also found in textbook 35. Which of the following categories of principles is most closely related to gathering audit evidence? A. Performance. B. Reasonable assurance. C. Reporting. D. Responsibilities. 36. To exercise due care, an accountant should A. Take continuing professional education classes. B. Report whether the financial statements are in accordance with the applicable financial reporting framework (e.g., GAAP, IFRS). C. Gather enough audit evidence to have complete assurance that there is enough support for the accountant's opinion on the financial statements. D. Conduct the engagement in accordance with GAAS and ensure that the engagement is completed on a timely basis. 37. One of an accounting firm's basic objectives is to provide professional services that conform to professional standards. Reasonable assurance of achieving this objective can be obtained by following A. Generally Accepted Auditing Standards (GAAS). B. Standards within a system of quality control. C. Generally Accepted Accounting Practices (GAAP). D. International Auditing Standards.
  • 37. 38. Which of the following best demonstrates the concept of professional skepticism? A. Relying more extensively on external evidence rather than internal evidence. B. Focusing on items that have a more significant quantitative effect on the entity's financial statements. C. Critically assessing verbal evidence received from the entity's management. D. Evaluating potential financial interests held by auditors in the client. 39. The primary purpose for obtaining an understanding of the entity's environment (including its internal control) in a financial statement audit is A. To determine the nature, timing, and extent of further audit procedures to be performed. B. To make consulting suggestions to the management. C. To obtain direct sufficient appropriate audit evidence to afford a reasonable basis for an opinion on the financial statements. D. To determine whether the entity has changed any accounting principles. 40. Ordinarily, what source of evidence should least affect audit conclusions? A. External. B. Inquiry of management. C. Auditor prepared. D. Inquiry of entity legal counsel. 41. The most persuasive evidence regarding the existence of newly acquired computer equipment is A. Inquiry of management. B. Documentation prepared externally. C. Observation of auditee's procedures. D. Physical observation. 42. Which of the following procedures would provide the most reliable audit evidence? A. Inquiries of the client's internal audit staff held in private. B. Inspection of pre-­‐numbered client purchase orders filed in the vouchers payable department. C. Inspection of vendor sales invoices received from client personnel. D. Inspection of bank statements obtained directly from the client's financial institution. 1-37
  • 38. 43. Breaux & Co., CPAs require that all audit documentation contain the initials of the preparer and the reviewer in the top right-­‐hand corner. This procedure provides evidence of Breaux & Co., CPAs' professional concern regarding which of the following? A. Independence. B. Adequate competence and capabilities. C. Adequate planning and supervision. D. Gathering sufficient appropriate evidence. 1-38 44. The attestation standards do not contain a requirement that auditors obtain A. Adequate knowledge in the subject matter of the assertions being examined. B. An understanding of the auditee's internal controls. C. Sufficient evidence for the conclusions expressed in an attestation report. D. Independence in mental attitude. 45. Which of the following concepts is least related to the standard of due care? A. Independence in fact B. Professional skepticism C. Prudent auditor D. Reasonable assurance 46. The evidence considered most appropriate by auditors is best described as A. Internal documents such as sales invoice copies produced under conditions of strong internal control. B. Written representations made by the president of the entity. C. Documentary evidence obtained directly from independent external sources. D. Direct personal knowledge obtained through physical observation and mathematical recalculation.
  • 39. 47. Auditors' understanding of the internal control in an entity contributes information for A. Determining whether members of the audit team have the required competence and capabilities to perform the audit. B. Ascertaining the independence in mental attitude of members of the audit team. C. Planning the professional development courses the audit staff needs to keep up to date with new auditing standards. D. Planning the nature, timing, and extent of further audit procedures on an audit. 48. Which of the following elements of a system of quality control is related to firms receiving independence confirmations from its professionals with respect to clients? A. Acceptance and continuance of clients. B. Engagement performance. C. Monitoring. D. Relevant ethical requirements. 49. Which of the following standards is not correctly associated with its rule-­‐making body? A. Public Company Accounting Oversight Board, Auditing Standards 1-39 B. Governmental Accounting Standards Board, Government Auditing Standards C. Auditing Standards Board, Statements on Auditing Standards D. International Auditing and Assurance Standards Board, International Statements on Auditing 50. Kramer, CPA consulted with an independent appraiser regarding the valuation of fine art for a not-­‐for-­‐profit museum. Consultation with a specialist in this case would A. Be considered proper due care. B. Be considered a failure to follow GAAS because Kramer should have known how to value fine art before accepting the engagement. C. Not be considered a violation of GAAS because GAAS does not apply to not-­‐for-­‐profit entities. D. None of the above.
  • 40. 1-40 51. Which of the following topics is not addressed in the auditors' report for a public entity? A. Responsibilities of the auditor and management in the financial reporting process. B. Absolute assurance regarding the fairness of the entity's financial statements in accordance with the applicable financial reporting framework (e.g., GAAP). C. A description of an audit engagement. D. A summary of the auditors' opinion on the effectiveness of the entity's internal control over financial reporting. 52. Which of the following is a conceptual difference between attestation standards and generally accepted auditing standards? A. The attestation standards provide a framework for the attest function beyond historical financial statements. B. The requirement that the practitioner be independent is not required under attestation standards. C. The attestation standards do not permit an attestation engagement to examine prospective "what-­‐if" financial statements. D. Requirements related to evidence are not included in the attestation standards. Questions also found in Study Guide 53. The attestation standards are a general set of standards intended to guide work in A. Audits of financial statements. B. Financial forecasts and prospective financial information. C. Areas other than audits of financial statements. D. Understanding internal control. 54. Statements on Auditing Standards (SASs) are considered to be A. Specialized to obtain evidence to render an opinion. B. Detailed interpretations of the fundamental principles. C. Standards for preparation of financial statements. D. Standards to govern the quality of a specific firm's audit practice.
  • 41. 1-41 55. Which of the following is not a subject related to the performance principle of GAAS? A. Risk of material misstatement B. Planning and supervision C. Sufficient appropriate evidence D. Due care 56. Which of the following statements is true for attestation standards, but not for the fundamental principles of generally accepted auditing standards? A. The practitioner or practitioners must have reason to believe that the subject matter is capable of evaluation against criteria that are suitable and available to users. B. The work shall be adequately planned and assistants, if any, are to be properly supervised. C. Due care shall be exercised. D. A sufficient understanding of the internal control is to be obtained. 57. The quality control of personnel management in a public accounting firm includes which of the following? A. Supervision appropriate for the competencies of the personnel assigned to the work is important. B. Professional development continuing education should be provided so that personnel will have the knowledge required to enable them to fulfill their responsibilities. C. People at all organizational levels must maintain independence in fact and appearance. D. When accepting and continuing client relationships, firms should consider their own competence. 58. Which of the following is not an implicit message in the opinion paragraph in the auditors' unqualified opinion? A. The accounting principles in the financial statements have general acceptance. B. The accounting principles used by the entity are appropriate in the circumstances. C. The audit was performed in accordance with generally accepted auditing standards. D. The financial statements are accurate within practical materiality limits.
  • 42. 59. Auditors' opinions on statements "taken as a whole" would not include A. Disclaimers of opinion. B. Adverse opinions. C. Qualified opinions. D. Unqualified opinions. 60. The opinion paragraph of the auditors' standard report includes a statement that A. The financial statements are the responsibility of management. B. The audit was conducted in accordance with generally accepted auditing standards. C. The audit provides a reasonable basis for an opinion. D. The financial statements are presented in conformity with generally accepted accounting principles. 61. The auditors' standard report should be dated with the date A. The report was delivered to the client. B. When all significant procedures have been completed and auditors have gathered sufficient appropriate evidence. C. When the client's fiscal year ended. D. When the audit was completely reviewed by supervisory personnel. 62. To ensure that a public accounting firm is providing services that conform to professional standards, the firm should follow A. The performance principle of GAAS. B. Its system of quality controls. C. Generally accepted accounting principles. D. International auditing standards. 1-42
  • 43. 1-43 Matching Questions 63. For each of the matters below, indicate through the appropriate letter the fundamental principle to which the matter is most closely related. 1. Responsibilities principle Maintaining professional skepticism. ____ 2. Responsibilities An auditors' overall conclusion of the fairness of the principle client's financial statements. ____ 3. Reporting principle The use of an audit plan to identify audit procedures to be performed during the engagement. ____ 4. Performance principle Auditors' assessment of the risk of material misstatement. ____ 5. Performance principle Accounting firm policies with respect to the level of expected continuing professional education. ____ 6. Performance principle Expressing an opinion in accordance with the auditor's findings. ____ 7. Reporting principle Proper supervision of assistants on the audit. ____ Auditors' requests to obtain bank statements directly 8. Performance from financial institutions with whom the client does principle business. ____ 9. Reporting principle An expression that an opinion cannot be expressed. ____ 10. Performance Determining and applying an appropriate materiality principle level. ____ True / False Questions Question also Found in Study Guide 64. Auditors may be independent in fact but not independent in appearance. True False 65. Standards for accountants in public practice are limited to auditing services. True False
  • 44. 66. The attestation standards provide guidance for a wide variety of attestation engagements. True False 67. The AICPA's Generally Accepted Auditing Standards must be followed on all audit engagements. True False 68. The reporting principle relates to a firm's system of quality control criteria for conducting an audit. True False 69. Auditors cannot effectively satisfy the responsibilities principle requiring due care if they have not also satisfied the performance principle. True False 70. Auditing procedures are quality guides that are less specific than auditing standards. True False 71. Auditing procedures are the same as auditing standards. True False 72. The concept of due care requires auditors to observe the performance, responsibilities and reporting principles. True False 73. Attestation standards require the practitioner to obtain a sufficient understanding of the client's internal control. True False 1-44
  • 45. 74. The performance principle sets forth the quality criteria for conducting an audit. True False 75. Auditors of entities registered with the Securities and Exchange Commission are required to register with the Public Company Accounting Oversight Board (PCAOB). True False 76. Control risk is the probability that a material misstatement (error or fraud) could occur and not be prevented or detected on a timely basis by the entity's external auditors. True False 77. Evidence that is considered "appropriate" in auditing means that all underlying accounting data and corroborating information must be absolutely compelling to auditors. True False 78. Even in the audit of historical cost financial statements, auditors may have to make inferences about the future. True False 79. The contents of the auditors' report are guided exclusively by the reporting principle of GAAS. True False 80. The auditors' standard report should always make direct reference to consistency and disclosure. True False 1-45
  • 46. 81. The auditors' standard report should either contain an expression of opinion on the financial statements taken as a whole or an assertion to the effect that an opinion cannot be expressed. True False 82. Evidence is considered appropriate when it is both valid and relevant. True False 83. The statement on quality control standards No. 7 notes that the purpose of a system of quality control is to provide reasonable assurance that the firm and its personnel issue reports that are appropriate under the circumstances. True False 1-46 Fill in the Blank Questions Question also found in Study Guide 84. The _____________________________ standards are a general set of standards to guide attestation engagements in areas other than audits of financial statements. ________________________________________ 85. Audits of historical financial statements are guided by a broad set of principles referred to as _______________________ _________________________ _____________________________ _____________________________. ________________________________________
  • 47. 86. Attestation reporting is different because attestation engagements related to nonfinancial information do not require information to be presented in accordance with _____________________________ _____________________________ _____________________________ _____________________________. ________________________________________ 87. The AICPA's fundamental principles of generally accepted auditing standards are classified in three categories: _______________________ principle, _______________________ principle, and the _____________________________ principle. ________________________________________ 88. A(n) _____________________________ _____________________________ is a list of auditing procedures that will be performed during the engagement to gather sufficient appropriate evidence. ________________________________________ 89. The responsibilities principle of GAAS highlights the importance of complying with ethical requirements, including those pertaining to __________________________ and _____________________________. ________________________________________ 90. The three aspects of practical independence are _____________________________ independence, _____________________________ independence, and _____________________________ independence. ________________________________________ 91. The concept of _____________________________ relates to financial statement users' perceptions of auditors' independence. ________________________________________ 1-47
  • 48. 92. _____________________________ ________________________ reflects a level of performance that would be exercised by reasonable auditors in similar circumstances. ________________________________________ 93. Since audit samples are used, audit evidence is considered to be _____________________________, rather than _____________________________. ________________________________________ 94. The auditors' report must state whether the financial statements are presented in accordance with __________________________ _____________________________ _____________________________ _____________________________. ________________________________________ 95. Under the reporting principle of GAAS, the auditor expresses an opinion in accordance with the ___________________ ______________. ________________________________________ 96. Under the reporting principle of GAAS, the report will contain either an expression of _____________________________ regarding the financial statements, taken as a whole, or an assertion to the effect that an opinion cannot be expressed. ________________________________________ 97. An overall opinion that the financial statements present the financial condition, results of operations, and cash flows according to generally accepted accounting principles is a(n) _____________________________ opinion. ________________________________________ 98. If a material departure from GAAP is noted, auditors can choose between a(n) __________________________ opinion or a(n) ___________________________ opinion. ________________________________________ 1-48
  • 49. 99. Auditors' indication that no opinion is given is referred to as a(n) _______________________ ___________________________________________ ________________________________________ 100. The _____________________________ paragraph of the auditors' report declares that the audit was conducted in accordance with generally accepted _________________________________________________________ ________________________________________ 101. A(n) _____________________________ _____________________________ is a study of an accounting firm's quality control policies and procedures, followed by a report on the firm's quality of audit practice in accordance with the system of quality controls. ________________________________________ 102. The _____________________________ _____________________________ Act of 2002 created the Public Company Accounting Oversight Board (PCAOB). ________________________________________ 103. The PCAOB has two primary roles: _____________________________ and _____________________________. ________________________________________ 1-49
  • 50. 1-50 Essay Questions 104. Distinguish between attestation standards and the fundamental principles of generally accepted auditing standards by identifying and describing major differences between the two sets of standards.
  • 51. 105. Alan Fallon was recently promoted to senior accountant. He was put in charge of the Mellow Markets audit because of his experience with other grocery clients. Mellow Markets has a small, but growing, chain of natural food stores. This is the first year Mellow Markets has been audited. Because of their growth, Mellow Markets needs additional capital and intend to use their audited financial statements to secure a loan. Alan has been assigned two inexperienced staff assistants for the audit. Because this is his first engagement as a senior, he intends to bring the job in on budget. To save time, he provided his assistants with a copy of the audit plan for Happy Time Food Stores. He told them that this would make things go more quickly. He also told them that he could not spend much time with them at the client's place of business, because "my time is billed out at such a high rate, we'll go right over budget." However, he did call them once a day from another audit on which he was working. After beginning their work, the assistants told Alan that the audit plan did not always match up with what they found at Mellow Markets. Alan responded, "just cross out whatever is not relevant in the audit plan and don't add anything -­‐ it will only make us go over the budget." When Alan came to the client near the end of field Work, one assistant was concerned that no inventory observation was done at the out-­‐of-­‐town locations of Mellow Markets (the audit plan had stipulated that inventory should be observed for in-­‐town stores only). Happy Time had only one out-­‐of-­‐town location, while three of Mellow Markets' five stores were in other cities. Alan told the assistant to get inventory sheets from the client for the other stores and added "make sure that the inventory balance in the general ledger agrees with the total for all the inventory sheets." The next day, Alan reviewed all audit documentation and submitted the job for review by the manager. Required: 1. Describe the performance principle of GAAS. 2. Do you believe that the Mellow Markets audit complies with these standards? Explain. 1-51 Matching Questions Question also Found in Study Guide
  • 52. 106. Using I (introductory), S (scope), O (opinion), A (additional), or N (none), indicate the paragraph in which the following statements or topics would be included in the auditors' report. 1-52 1. none The titles of the financial statements examined by the auditors. ____ 2. none A description of any scope limitation(s) encountered during the audit. ____ 3. introductory A statement that auditors were independent with respect to the entity. ____ 4. additional The auditors' conclusion with respect to the fairness of the entity's financial statements. ____ 5. opinion A statement that an audit was conducted in accordance with generally accepted auditing standards. ____ 6. opinion A statement that the entity's management is responsible for the fairness of the financial statements. ____ 7. introductory A description of an audit, which includes examining evidence in support of the financial statements. ____ 8. scope Reference to generally accepted accounting principles. ____ A description of any specific departures from GAAP noted 9. scope during the audit that were material. ____ 10. additional A statement that the financial statements were consistently prepared compared to those of prior period(s). ____
  • 53. Chapter 02 Professional Standards Answer Key 1-53 Multiple Choice Questions 1. The attestation standards of reporting do not require the attestation report to include a statement that A. Provides a conclusion whether the subject matter is presented in conformity with established or stated criteria. B. Indicates that the practitioner has significant reservations about the engagement. C. Identifies the subject matter or assertion being reported on. D. Indicates that the accountant assumes no responsibility to update the report. Original AACSB: Analytic AICPA BB: Legal AICPA FN: Research Bloom's: Knowledge Difficulty: Easy 2. Control risk is A. The probability that a material misstatement could not be prevented or detected by the entity's internal control policies and procedures. B. The probability that a material misstatement could occur and not be detected by auditors' procedures. C. The risk that auditors will not be able to complete the audit on a timely basis. D. The risk that auditors will not properly control the staff on the audit engagement. Original AACSB: Analytic AICPA BB: Legal AICPA FN: Risk Analysis Bloom's: Knowledge Difficulty: Easy
  • 54. 3. The responsibilities principle under generally accepted auditing standards does 1-54 not include which of the following? A. Competence and capabilities. B. Independent attitude. C. Due care. D. Planning and supervision. Original AACSB: Analytic AICPA BB: Legal AICPA FN: Research Bloom's: Knowledge Difficulty: Easy 4. Which of the following types of auditors' reports does not require an explanatory paragraph to support the opinion? A. Unqualified opinion. B. Adverse opinion. C. Qualified opinion. D. Disclaimer of opinion. Original AACSB: Communication AICPA BB: Legal AICPA FN: Reporting Bloom's: Knowledge Difficulty: Easy
  • 55. 5. Which of the following is an element of a system of quality control that should be considered by a public accounting firm in establishing its quality control policies and procedures? A. Lending credibility to a client's financial statements. B. Using statistical sampling techniques. C. Acceptance and continuance of clients. D. Membership in the Center for Public Company Audit Firms (CPCAF). 1-55 Original AACSB: Analytic AICPA BB: Legal AICPA FN: Research Bloom's: Knowledge Difficulty: Medium 6. Which of the following presumptions does not relate to the reliability of audit evidence? A. The more effective the client's internal control, the more assurance it provides about the accounting data and financial statements. B. The auditors' opinion, to be economically useful, is formed within reasonable time and based on evidence obtained at a reasonable cost. C. Evidence obtained from independent sources outside the entity is more reliable than evidence secured solely within the entity. D. The independent auditors' direct personal knowledge, obtained through observation and inspection, is more persuasive than information obtained indirectly. AICPA AACSB: Analytic AICPA BB: Legal AICPA FN: Research Bloom's: Comprehension Difficulty: Hard
  • 56. 7. An important role of the Public Company Accounting Oversight Board (PCAOB) is to oversee the A. Issuance of statements by the Financial Accounting Standards Board. B. Preparation and grading of the Uniform CPA Examination. C. Peer review of member firms of the Private Companies Practice Section. D. Regulation of firms that audit public companies. 1-56 Original AACSB: Analytic AICPA BB: Legal AICPA FN: Research Bloom's: Knowledge Difficulty: Easy 8. Audit evidence is usually considered sufficient when A. It is reliable. B. There is enough quantity to afford a reasonable basis for an opinion on financial statements. C. It has the qualities of being relevant, objective, and free from unknown bias. D. It has been obtained through random selection methods. AICPA AACSB: Analytic AICPA BB: Legal AICPA FN: Research Bloom's: Knowledge Difficulty: Medium
  • 57. 1-57 9. Which of the following is not considered a type of audit evidence? A. The company's trial balance. B. Auditors' calculations. C. Physical observation. D. Verbal statements made by client personnel. Original AACSB: Analytic AICPA BB: Legal AICPA FN: Research Bloom's: Knowledge Difficulty: Medium 10. The AICPA attestation standards differ from the AICPA responsibilities principle, performance principle and reporting principle in that: A. The attestation standards contain no requirement to obtain an understanding of the entity and assess the risk of material misstatement. B. The attestation standards do not require competence and capabilities. C. The attestation standards do not require planning for attestation engagements or supervision of accountants and consultants who perform the work. D. The attestation standards do not require a report that states the character of the engagement. Original AACSB: Analytic AICPA BB: Legal AICPA FN: Research Bloom's: Knowledge Difficulty: Hard
  • 58. 11. An audit of the financial statements of Camden Corporation is being conducted by external auditors. The external auditors are expected to: A. Certify the correctness of Camden's financial statements. B. Make a complete examination of Camden's records and verify all of Camden's transactions. C. Give an opinion on the fair presentation of Camden's financial statements in conformity with the applicable financial reporting framework (e.g., GAAP, IFRS). D. Give an opinion on the attractiveness of Camden for investment purposes and critique the wisdom and legality of its business decisions. 1-58 Original AACSB: Communication AICPA BB: Legal AICPA FN: Reporting Bloom's: Knowledge Difficulty: Easy 12. Auditors try to achieve independence in appearance in order to: A. Maintain public confidence in the profession. B. Become independent in fact. C. Comply with the responsibilities principle. D. Maintain an unbiased mental attitude. Original AACSB: Ethics AICPA BB: Legal AICPA FN: Research Bloom's: Knowledge Difficulty: Medium
  • 59. 13. The independent auditors' plan prepared prior to the start of field work is appropriately considered documentation of A. Planning. B. Supervision. C. Information evaluation. D. Quality assurance. 1-59 Original AACSB: Analytic AICPA BB: Legal AICPA FN: Research Bloom's: Knowledge Difficulty: Easy 14. Which of the following procedures would provide the most reliable audit evidence? A. Inquiries of the client's accounting staff held in private. B. Inspection of pre-­‐numbered client shipping documents. C. Inspection of bank statements obtained directly from the client's financial institution. D. Analytical procedures performed by auditors on the client's trial balance. AICPA AACSB: Analytic AICPA BB: Legal AICPA FN: Decision Making Bloom's: Knowledge Difficulty: Hard
  • 60. 15. Which of the following is not an attestation standard? A. The practitioner must obtain sufficient evidence to provide a reasonable basis for the conclusion expressed in the report. B. The practitioner must identify the subject matter or the assertion being reported on and state the character of the engagement. C. The practitioner must adequately plan the work and must properly supervise any assistants. D. A sufficient understanding of the client's internal controls shall be obtained to plan the engagement. 1-60 AICPA AACSB: Analytic AICPA BB: Legal AICPA FN: Research Bloom's: Knowledge Difficulty: Medium 16. Which of the following would most likely be a violation of the independence requirement found in the responsibilities principle under generally accepted auditing standards? A. An auditor on the engagement has a distant relative who is employed by a vendor that does a significant amount of business with clients. B. The client's Chief Executive Officer graduated from the same university as the partner in charge of the accounting firm. C. An auditor on the engagement owns a financial interest in the stock of the client. D. The client provides financial support to a number of charitable causes that also receive support from the accounting firm. Original AACSB: Ethics AICPA BB: Legal AICPA FN: Research Bloom's: Knowledge Difficulty: Easy
  • 61. 17. A vendor's invoice received and held by the client would be considered what type of evidence? A. External. B. Internal. C. External-­‐internal. D. Written representation. 1-61 Original AACSB: Analytic AICPA BB: Legal AICPA FN: Research Bloom's: Knowledge Difficulty: Easy 18. Which of the following statements is generally correct about the appropriateness of audit evidence? A. Auditors' direct personal knowledge, obtained through observation and inspection, is more persuasive than information obtained indirectly from independent outside sources. B. To be reliable, audit evidence must be either valid or relevant, but need not be both. C. Client accounting data alone may be considered sufficient appropriate audit evidence to issue an unqualified opinion on client financial statements. D. Appropriateness of audit evidence refers to the amount of corroborative evidence to be obtained. AICPA AACSB: Analytic AICPA BB: Legal AICPA FN: Decision Making Bloom's: Comprehension Difficulty: Medium
  • 62. 19. The standard auditors' report refers to GAAS and GAAP in which paragraph? A. GAAS: Scope only; GAAP: Opinion only B. GAAS: Introductory only; GAAP: Scope and opinion C. GAAS: Introductory and scope; GAAP: Opinion only D. GAAS: Introductory only; GAAP: All paragraphs 1-62 AICPA AACSB: Communication AICPA BB: Legal AICPA FN: Reporting Bloom's: Knowledge Difficulty: Medium 20. Which of the following is not included in the auditors' standard report representing an unqualified opinion? A. A brief indication of the responsibility of auditors and management for the financial statements. B. An indication that all appropriate disclosures have been made and included in the financial statements. C. An indication that the audit was conducted in accordance with standards established by the PCAOB. D. The auditors' opinion on the fairness of the financial statements. Original AACSB: Communication AICPA BB: Legal AICPA FN: Reporting Bloom's: Knowledge Difficulty: Medium
  • 63. 21. Internal evidence A. Is obtained directly from third parties independent of the client. B. Originates outside of the client's system but has been received and processed by the client. C. Consists of documents that are produced, used, and stored within the client's information system. D. Consists of representations made by the client's officers, directors, owners, and employees. 1-63 Original AACSB: Analytic AICPA BB: Legal AICPA FN: Research Bloom's: Knowledge Difficulty: Medium 22. Which of the following presumptions is correct about the reliability of audit evidence? A. Information obtained indirectly from outside sources is the most reliable form of audit evidence. B. To be reliable, audit evidence should be convincing rather than persuasive. C. Reliability of audit evidence refers to the amount of corroborative evidence obtained. D. An effective system of internal control provides more assurance about the reliability of audit evidence. AICPA AACSB: Analytic AICPA BB: Legal AICPA FN: Decision Making Bloom's: Comprehension Difficulty: Hard
  • 64. 23. When auditors do not mention consistency in the auditors' report, a reader of the financial statements may infer A. That the applicable financial reporting framework (e.g., GAAP) has been consistently observed in the current period in relation to the preceding period. B. That no material departure from the applicable financial reporting framework (e.g., GAAP) has been detected. C. That no reclassification of items or change in classifications has occurred. D. Nothing about application of accounting principles within the period. 1-64 AICPA AACSB: Communication AICPA BB: Legal AICPA FN: Reporting Bloom's: Knowledge Difficulty: Hard 24. The auditors' responsibility to express an opinion on the financial statements is A. Implicitly represented in the auditors' standard report. B. Explicitly represented in the introductory paragraph of the auditors' standard report. C. Explicitly represented in the scope paragraph of the auditors' standard report. D. Explicitly represented in the opinion paragraph of the auditors' standard report. AICPA AACSB: Communication AICPA BB: Legal AICPA FN: Reporting Bloom's: Knowledge Difficulty: Hard
  • 65. 25. Which of the following is not a concept from the performance principle under generally accepted auditing standards? A. The auditor must plan the work and properly supervise any assistants. B. The auditor must express an opinion in accordance with the auditor's findings. C. The auditor must obtain sufficient appropriate evidence about whether material misstatements exist. D. The auditor must determine and apply an appropriate materiality level throughout the audit. 1-65 Original AACSB: Analytic AICPA BB: Legal AICPA FN: Research Bloom's: Knowledge Difficulty: Medium 26. Under generally accepted auditing standards, which of the following reflects a concept related to the responsibilities principle? A. The initial planning of the audit engagement should occur with the audit partner, manager, senior, and client personnel. B. The confirmation of accounts receivable should occur on each audit. C. The completion of an internal control questionnaire. D. Maintaining professional skepticism and exercising professional judgment. Original AACSB: Analytic AICPA BB: Legal AICPA FN: Research Bloom's: Knowledge Difficulty: Medium
  • 66. 27. Which of the following represent audit quality guides that remain stable over time and are applicable for all audits? A. Auditing procedures. B. Auditing standards. C. Due care. D. System of quality control. 1-66 Original AACSB: Analytic AICPA BB: Legal AICPA FN: Research Bloom's: Knowledge Difficulty: Easy 28. Which of the following situations would most likely be in conflict with the responsibilities principle? A. Auditors perform the engagement with prudent auditors, but not expert auditors. B. Auditors obtain expertise in their client's industry as they are conducting the audit examination. C. Auditors are directly involved with a client manager in a strategic decision-­‐making capacity. D. Auditors fail to document their assessment of control risk following their study of internal control. Original AACSB: Analytic AICPA BB: Legal AICPA FN: Research Bloom's: Application Difficulty: Medium
  • 67. 1-67 29. Which of the following statements is not true with respect to the evidence that would be gathered when assessments of control risk are high? A. Auditors would be required to rely on external (rather than internal) forms of evidence. B. Auditors would be required to perform procedures at interim periods, rather than at year end. C. Auditors would be required to confirm a larger number of customer accounts receivable balances. D. Auditors would be required to obtain more evidence through direct personal observation. Original AACSB: Analytic AICPA BB: Critical Thinking AICPA FN: Decision Making Bloom's: Application Difficulty: Hard 30. As it relates to audit evidence, appropriateness refers to the A. Originality of evidence gathered. B. Quality of evidence gathered. C. Quantity of evidence gathered. D. Timeliness of evidence gathered. Original AACSB: Analytic AICPA BB: Legal AICPA FN: Research Bloom's: Knowledge Difficulty: Easy
  • 68. 31. Which of the following information would not be included in the auditors' standard report? A. The names of the financial statements audited. B. A description of the nature of an audit. C. An indication that all necessary disclosures have been presented. D. An opinion on the entity's financial statements. 1-68 Original AACSB: Communication AICPA BB: Legal AICPA FN: Reporting Bloom's: Knowledge Difficulty: Easy 32. The primary purpose of the auditors' study of internal control for a nonpublic entity is: A. To provide constructive suggestions to the client for improving its internal control. B. To report on internal control as required by Auditing Standard No. 5. C. To identify and detect fraud and irregularities perpetrated by client personnel. D. To determine the nature, timing, and extent of substantive procedures. Original AACSB: Analytic AICPA BB: Legal AICPA FN: Risk Analysis Bloom's: Knowledge Difficulty: Easy